Está en la página 1de 282

PRESENTACIÓN

El Instituto de Ciencias y Humanidades (ICH) está orientado al ser-


vicio educativo y a la formación integral de sus estudiantes mediante el desarrollo
de sus potencialidades. Todos los esfuerzos de nuestra institución se centran en la
enseñanza, formación e investigación en matemática, ciencias naturales, ciencias
sociales y las diversas expresiones del arte.
La matemática tiene un rol protagónico en el desarrollo de las ciencias y la
tecnologı́a. Su contribución en las investigaciones de otras ciencias es invalorable.
Ası́ por ejemplo, fı́sicos, quı́micos, biólogos, sociólogos, antropólogos, por citar sólo
algunos ejemplos, hacen uso en sus trabajos de teorı́as y herramientas matemáticas.
Desde esta perspectiva, desarrollamos una polı́tica de fomento a la investigación de
nuestros docentes que tiene como resultado no sólo la elevación del servicio educativo
en las aulas sino también la publicación de textos académicos como el presente.
Mediante estas lı́neas, presentamos a la comunidad educativa del paı́s el libro
Desigualdades, olimpiadas matemáticas, obra del matemático Victor Hugo
Laurente Artola, docente de nuestra institución.
Diferentes delegaciones de estudiantes peruanos han representado al paı́s en
olimpiadas internacionales de matemáticas y han obtenido valiosos resultados que no
hacen sino renovar nuestra confianza en el gran potencial de nuestros jóvenes y en
su interés por el estudio. Asimismo, se corrobora el trabajo denodado de los docentes,
quienes a pesar de las adversidades participan en cursos, talleres, capacitaciones y
otras jornadas académicas a fin de elevar su dominio de la matemática.
El Instituto de Ciencias y Humanidades reafirma su compromiso con el trabajo
educativo y la promoción de esfuerzos de investigación individuales y colectivos a fin
de que se materialicen sus metas institucionales.

Instituto de Ciencias y Humanidades

i
ii
INTRODUCCIÓN

Cuando a fines de la década del 90 empezamos, con un grupo de colegas del


Instituto de Ciencias y Humanidades, a colaborar en la enseñanza de jóvenes que
representarı́an al Perú en las olimpiadas internacionales de matemática, nos topamos
con un grave problema: la falta de bibliografı́a sobre el tema. Con los años, fuimos
acumulando experiencias a partir de la consulta de textos que los propios estudian-
tes traı́an de sus viajes al extranjero, de algunas publicaciones que ya se podı́an
encontrar en nuestro medio y de la enseñanza en las aulas.
De este modo, fue germinando en mi persona el interés por realizar una pu-
blicación de mi especialidad, en particular sobre el tema de desigualdades. Ası́, en
diciembre de 2007 pude iniciar el proyecto que después de diez meses se materializa
en la presente publicación titulada Desigualdades, olimpiadas matemáticas.
No hay experiencia comparada a la adquirida por los estudiantes que participan
en las olimpiadas. Su participación en olimpiadas, permite que los estudiantes ga-
nen mayor conocimiento, que se formen como futuros cientı́ficos y que establezcan
relaciones de colaboración académica con otras personas e instituciones del extran-
jero. Las pruebas de olimpiadas integran problemas de gran nivel analı́tico que co-
rresponden a temas propios del álgebra, matemática discreta, teorı́a de números y
geometrı́a.
Aparte del talento, quienes participan en olimpiadas requieren dedicación, per-
severancia y un equipo de profesionales que los ayude en su preparación. Se trata de
una práctica que deberı́a recorrer toda la vida escolar. Es necesario, desde la escuela,
fomentar la organización de talleres de matemática, la formación de equipos de es-
tudiantes que participen en concursos a nivel local, regional y nacional. El presente
texto pretende contribuir con este propósito.
El texto desarrolla el tema de desigualdades, uno de los tópicos fundamentales
del álgebra. Recoge un conjunto de elementos teóricos y una serie de ejercicios or-
ganizados de lo simple a lo complejo. Asimismo, se han incorporado los problemas

iii
iv

de las pruebas de las olimpiadas internacionales de matemática (IMO, por sus siglas
en inglés) desde el año 59 al año 2008. En algunos casos, los problemas están acom-
pañados de más de una solución de modo que el lector conozca una mayor diversidad
de estrategias para encarar los ejercicios.
Para la elaboración del texto he utilizado como fuente libros editados en Estados
Unidos, Europa y Asia. Asimismo, he recurrido a valiosa información de Internet y he
sistematizado la experiencia adquirida en la enseñanza de estudiantes que participan
en olimpiadas matemáticas.
Es recomendable, para abordar el texto, que el lector tenga conocimientos básicos
de desigualdades, de manera que pueda profundizar con mayor facilidad en los temas
propuestos. Es importante que estudie con detenimiento la teorı́a, revise las aplica-
ciones y luego intente resolver los problemas antes de recurrir a la solución.
Quiero hacer público mi agradecimiento al Instituto de Ciencias y Humanidades
(ICH) por su apoyo en la investigación y edición del texto. A Jorge Tipe Villanueva,
por sus importantes sugerencias, a Juan Neyra Faustino, por su colaboración en el
digitado en LATEXdel material y, en especial, a mi esposa Magda, a mi hijo Taylor y
a mis padres Francisco y Serafina por su constante y siempre desinteresado apoyo.
A todos ellos, mi profundo agradecimiento.

Victor Hugo Laurente Artola


Lima, setiembre de 2008
ÍNDICE GENERAL

1. Números Reales 1
1.1. Cuerpos . . . . . . . . . . . . . . . . . . . . . . . . . . . . . . . . . . 1
1.1.1. Axiomas de adición. . . . . . . . . . . . . . . . . . . . . . . . 1
1.1.2. Axiomas de multiplicación. . . . . . . . . . . . . . . . . . . . . 2
1.2. Cuerpos Ordenados . . . . . . . . . . . . . . . . . . . . . . . . . . . . 3
1.3. Aplicaciones . . . . . . . . . . . . . . . . . . . . . . . . . . . . . . . . 13

2. Valor Absoluto y Máximo Entero 25


2.1. Valor Absoluto . . . . . . . . . . . . . . . . . . . . . . . . . . . . . . 25
2.2. Máximo Entero o Parte Entera . . . . . . . . . . . . . . . . . . . . . 29
2.3. Aplicaciones . . . . . . . . . . . . . . . . . . . . . . . . . . . . . . . . 32

3. Desigualdad de Cauchy-Schwarz 37
3.1. El Lema de Titu . . . . . . . . . . . . . . . . . . . . . . . . . . . . . 39
3.2. Desigualdad de Schür . . . . . . . . . . . . . . . . . . . . . . . . . . . 41
3.3. Dos sustituciones muy útiles . . . . . . . . . . . . . . . . . . . . . . . 41
3.4. Aplicaciones . . . . . . . . . . . . . . . . . . . . . . . . . . . . . . . . 43

4. Desigualdad de la Media Aritmética - Media Geométrica 55


4.1. Aplicaciones . . . . . . . . . . . . . . . . . . . . . . . . . . . . . . . . 58

5. Desigualdad de Bernoulli y la Media Potencial 69


5.1. Media Potencial . . . . . . . . . . . . . . . . . . . . . . . . . . . . . . 72
5.2. Aplicaciones . . . . . . . . . . . . . . . . . . . . . . . . . . . . . . . . 76

6. Desigualdad de Reordenamientos 79

v
vi ÍNDICE GENERAL

7. Desigualdad con Funciones Convexas 89


7.1. Función convexa . . . . . . . . . . . . . . . . . . . . . . . . . . . . . . 89

8. Espacio Métrico 97
8.1. Desigualdad de Hölder . . . . . . . . . . . . . . . . . . . . . . . . . . 102
8.2. Desigualdad de Minkonski . . . . . . . . . . . . . . . . . . . . . . . . 103

9. Método de la Suma de Cuadrados 109

10.Desigualdades Simétricas y Cı́clicas 117


10.1. Polinomios simétricos y cı́clicos . . . . . . . . . . . . . . . . . . . . . 123

11.Problemas 129

12.Soluciones 149

Bibliografı́a 275
CAPÍTULO 1
NÚMEROS REALES

1.1. Cuerpos
Un cuerpo es un conjunto K con dos operaciones llamadas adición y multipli-
cación, que satisfacen ciertas condiciones llamadas axiomas de cuerpo.

1.1.1. Axiomas de adición.


A1 . Clausura.

x + y ∈ K, ∀ x, y ∈ K.

A2 . Conmutatividad.

x + y = y + x, ∀ x, y ∈ K.

A3 . Asociatividad.

(x + y) + z = x + (y + z), ∀ x, y, z ∈ K.

A4 . Elemento neutro.
Existe 0 ∈ K tal que

x + 0 = x, ∀ x ∈ K.

1
2 1. Números Reales

A5 . Simétrico.
Para todo x ∈ K existe (−x) ∈ K tal que

x + (−x) = 0.

De la conmutatividad, se sigue que

0+x=x+0=x y (−x) + x = x + (−x) = 0.

A la suma x + (−y) se denotará x − y y se denomina la diferencia entre x e y.


La operación se llama sustracción.

1.1.2. Axiomas de multiplicación.


M1 . Clausura.

x · y ∈ K, ∀ x, y ∈ K.

M2 . Conmutatividad.

x · y = y · x, ∀ x, y ∈ K.

M3 . Asociatividad.

(x · y)z = x(y · z), ∀ x, y, z ∈ K.

M4 . Elemento neutro.
Existe 1 ∈ K tal que 1 6= 0 y

x · 1 = x, ∀ x ∈ K.

M5 . Simétrico.
Para todo x 6= 0 ∈ K existe el inverso de x denotado por x−1 ∈ K tal que

x · x−1 = 1.
1.2. Cuerpos Ordenados 3

De la conmutatividad se sigue que

1·x = x·1 =x y x−1 · x = x · x−1 = 1.


x
Dados x, y ∈ K; con y 6= 0; el número x · y −1 se denota
.
y
Estos axiomas están relacionados con un axioma llamado distributividad.

D1 . Axioma de distributividad.

x · (y + z) = x · y + x · z, ∀ x, y, z ∈ K.

Ejemplos de cuerpos
m
1. Q = { | m ∈ Z; n ∈ Z+ }.
n
2. R = Q ∪ Q′ ; Q′ : conjunto de los irracionales.

3. Z2 = {0; 1}; veamos que este conjunto es un cuerpo de sólo 2 elementos.

0+1= 1 ; 0+0= 0 ; 0·0 =0 ; 1·0 =0


1+0= 1 ; 1+1= 0 ; 0·1 =0 ; 1·1 =1

1.2. Cuerpos Ordenados


Un cuerpo K es ordenado; si tiene un subconjunto distinguido P ⊂ K, llamado
el conjunto de elementos positivos de K que satisfacen las siguientes condiciones

P1 . x + y ∈ P y x · y ∈ P, ∀ x, y ∈ P.

P2 . Dado x ∈ K, exactamente ocurre una de las tres condiciones

x = 0 ∨ x ∈ P ∨ −x ∈ P.

Indicaremos con −P al conjunto de elementos −x, donde x ∈ P. De donde

K = P ∪ (−P) ∪ {0}.

Estos conjuntos son disjuntos y −P se llama conjunto de números negativos.


4 1. Números Reales

En todo cuerpo ordenado si a 6= 0; entonces a2 ∈ P; en efecto, siendo a 6= 0


entonces a ∈ P ∨ −a ∈ P; en el primer caso

a2 = a · a ∈ P;

en el segundo caso

a2 = (−a) · (−a) ∈ P.

En un cuerpo K: x < y se lee x es menor que y, que equivale a decir y − x ∈ P.


De donde
y > 0 equivale a decir y ∈ P.
x < 0 equivale a decir −x ∈ P;
x < y ⇔ y − x > 0;
x < y ⇔ y > x.
La relación de orden x < y en un conjunto ordenado K goza de los siguientes
teoremas
Teorema 1.1 (Transitividad)

x < y ∧ y < z ⇒ x < z.

 Prueba. En efecto

x < y ⇔ y − x > 0;
y < z ⇔ z − y > 0;

significa que (y − x); (z − y) ∈ P, entonces

(y − x) + (z − y) ∈ P
(z − x) ∈ P
z−x>0
x < z.
2
Teorema 1.2 (Tricotomı́a)
Dados x, y ∈ K, sólo se cumple una de las tres condiciones

x = y ∨ x < y ∨ x > y.
1.2. Cuerpos Ordenados 5

 Prueba. En efecto

∀ x, y ∈ K, y − x = 0 ∨ y − x ∈ P ∨ y − x ∈ −P,

equivale a

y = x ∨ x < y ∨ −(y − x) ∈ P
x−y ∈P
y<x
x>y

∴ y = x ∨ x < y ∨ x > y.
2
Teorema 1.3 (Monotonicidad de la adición)

x < y ⇔ x + z < y + z, ∀ z ∈ K.

 Prueba. En efecto

x<y
⇔ y−x> 0
⇔ (y + z) − (x + z) > 0
⇔ x + z < y + z.
2
Teorema 1.4 (Monotonicidad de la multiplicación)

x < y ∧ z > 0 ⇒ xz < yz,


x < y ∧ z < 0 ⇒ xz > yz.

 Prueba. En efecto

x<y ∧ z>0
⇔ y−x>0 ∧ z >0
⇔ (y − x); z ∈ P
6 1. Números Reales

entonces

(y − x) · z ∈ P
yz − xz ∈ P
yz − xz > 0
xz < yz.

Análogamente

x<y ∧ z <0 ⇔ y−x>0 ∧ z <0


⇔ (y − x); −z ∈ P
entonces

(y − x) · (−z) ∈ P
−yz + xz ∈ P
xz − yz > 0
xz > yz.
2
Teorema 1.5
x < y ∧ a < b ⇒ x + a < y + b.

 Prueba. En efecto

x < y ⇔ x + a < y + a, (Teorema 1.3)


a < b ⇔ y + a < y + b, (Teorema 1.3)

entonces por el teorema 1.1 x + a < y + b. 2


Teorema 1.6
x < y ∧ a > b ⇒ x − a < y − b.
 Prueba. En efecto, a > b ⇔ −a < −b, y como x < y entonces por el teorema
1.5 se tiene que

x + (−a) < y + (−b)


x − a < y − b.
2
1.2. Cuerpos Ordenados 7

Teorema 1.7
0 < x < y ∧ 0 < a < b ⇒ xa < yb.

 Prueba. En efecto

x < y ⇒ xa < ya; a > 0 (teorema 1.4)


a < b ⇒ ya < yb; y > 0 (teorema 1.4)

por el teorema 1.1 se tiene xa < yb. 2

Teorema 1.8

1
x>0 ⇔ > 0.
x

 Prueba. En efecto

(⇒) Como x · x−1 = 1 > 0 y x > 0 entonces x−1 > 0.


 
1 1
(⇐) Como > 0 entonces · x2 > 0, de donde x > 0.
x x
2
Teorema 1.9
xy > 0 ⇔ [(x > 0 ∧ y > 0) ∨ (x < 0 ∧ y < 0)].

 Prueba. En efecto

(⇒) Sea x > 0 entonces x−1 > 0.


Como xy > 0 entonces x−1 (xy) > 0, efectuando tenemos y > 0.
Similarmente si x < 0 tenemos y < 0.

(⇐) Si x > 0 ∧ y > 0 ⇒ xy > 0.


Similarmente si x < 0 ∧ y < 0 ⇒ −x > 0 ∧ −y > 0, entonces (−x)(−y) >
0, de donde xy > 0.

2
Teorema 1.10
xy < 0 ⇔ (x < 0 ∧ y > 0) ∨ (x > 0 ∧ y < 0).
8 1. Números Reales

 Prueba. La demostración es análoga al Teorema 1.9. 2


Teorema 1.11
1 1
Si ab > 0 entonces a<b ⇔ > .
a b
 Prueba.

(⇒) a < b.
Como ab > 0 entonces tenemos dos casos:
1 1
i. Si a > 0 ∧ b > 0 entonces >0 ∧ > 0, luego
a b
1 1 b−a
− = > 0, pues a < b
a b ab
1 1
⇒ − >0
a b
1 1
⇒ > .
a b
ii. Para a < 0 ∧ b < 0 el procedimiento es análogo.
   
1 1 1 1
(⇐) Como > entonces ab > ab, efectuando tenemos b > a, de donde
a b a b
a < b.

2
Teorema 1.12
En un cuerpo ordenado K, las siguientes afirmaciones son equivalentes:

(i) N ⊂ K no es acotado superiormente.


(ii) Dados a, b ∈ K, con a > 0, existe n ∈ N tal que na > b.
1
(iii) Dado cualquier a > 0 en K, con a > 0, existe n ∈ N tal que 0 < < a.
n
 Prueba. (i) ⇒ (ii). Como N no es acotado superiormente, entonces dados
b
a > 0 y b en K, existe n ∈ N tal que < n, de donde b < na.
a
1
Veamos (ii) ⇒ (iii), dado a > 0, existe n ∈ N tal que na > 1, de donde < a;
n
1
luego tenemos 0 < < a.
n
1.2. Cuerpos Ordenados 9

Por último veamos (iii) ⇒ (i).


1 1
De (iii) dado cualquier b > 0 existe n ∈ N tal que < , de donde se tiene
n b
b < n, es decir n > b.
Esto significa que ningún positivo de K es cota superior de N. 2

Definición 1.1: Un cuerpo ordenado K es arquimediano cuando se cumple cualquiera


de las tres condiciones del teorema anterior.
Ejemplo 1.1
El cuerpo de los números racionales es arquimediano.

Definición 1.2 : Sea K un cuerpo ordenado y X ⊂ K un subconjunto acotado


superiormente. Un elemento x0 ∈ K se denominará supremo de X cuando x0 es la
menor de las cotas superiores de X en K.

Para que x0 sea supremo de X ⊂ K es necesario y suficiente que sean satisfechas


las siguientes condiciones:

(i) x ≤ x0 , ∀ x ∈ X.

(ii) Si y ∈ K/x ≤ y ∀ x ∈ X, entonces x0 ≤ y.

Ası́ mismo se denota sup (X) = x0 , y es único.


Similarmente un elemento y0 ∈ K se denomina ı́nfimo de Y ⊂ K, acotado infe-
riormente, cuando y0 es la mayor de todas las cotas inferiores de Y en K.
Para que y0 sea ı́nfimo de Y ⊂ K es necesario y suficiente que sean satisfechas
las siguientes condiciones:

(i) ∀ y ∈ Y ; y ≥ y0 .

(ii) Si z ∈ K/z ≤ y ∀ y ∈ Y , entonces z ≤ y0 .

Se denota ı́nf (Y ) = y0 y a la vez es único.

Definición 1.3 Sea K un cuerpo ordenado y sea X ⊂ K con X 6= ∅, un elemento


x0 ∈ X, es el máximo de X si y sólo si x ≤ x0 , para todo x ∈ X.

Definición 1.4 Sea K un cuerpo ordenado y sea Y ⊂ K con Y 6= ∅, un elemento


y0 ∈ Y , es el mı́nimo de Y si y sólo si y ≥ y0 , para todo y ∈ Y .
10 1. Números Reales

Ejemplo 1.2
Sean A = [−3, 5], B = h−3, 5i, entonces

máx (A) = 5; sup (A) = 5


mı́n (A) = −3; ı́nf (A) = −3
mı́n (B) no existe; sup (B) = 5
máx (B) no existe; ı́nf (B) = −3.

En efecto, veamos por que el supremo de B es 5.

 Prueba. Supongan que sup (B) no es 5; sea sup (B) = x0 , significa que x0 < 5:
x0 + 5 x0 + 5
Como x0 < < 5, entonces m = ∈ B, de donde
2 2
x0 < m; (1.1)
pero x ≤ x0 , ∀x ∈ B; en particular haciendo x = m, tenemos

m ≤ x0 ; (1.2)
de (1.1) y (1.2), tenemos

m ≤ x0 < m

lo cual es una contradicción.

∴ sup (B) = 5.

Similarmente, pruebe que el ı́nf (B) es −3.

Definición 1.5 : Un cuerpo ordenado K se denomina completo cuando un subcon-


junto no vacı́o acotado superiormente, X ⊂ K, posee supremo en K.
De la definición resulta que en un cuerpo ordenado completo todo subconjunto
no vacı́o acotado inferiormente, Y ⊂ K, posee ı́nfimo; en efecto, dado Y sea X =
−Y = {−y/ y ∈ Y } = 6 ∅, es acotado superiormente, entonces existe sup (X) = x0 ;
de donde ı́nf (Y ) = −x0 .

Axioma. Existe un cuerpo ordenado completo R, denominado el cuerpo de los


números reales.
1.2. Cuerpos Ordenados 11

Lema 1.1 (de Pitágoras)


No existe un número racional cuyo cuadrado sea 2.
 2
p
 Prueba. Supongamos que existe, es decir = 2, p, q ∈ Z, primos entre sı́.
q

⇒ p2 = 2q 2 ; entonces p es par, luego p = 2k, reemplazando


(2k)2 = 2q 2 ;
2k 2 = q 2 ; también q es par, como p y q son primos entre sı́,
hemos llegado a una contradicción.

Por lo tanto, no existe un racional con esas condiciones, es decir, 2 no es
racional. 2
Los números reales que no son racionales se denominan irracionales, luego √ el
conjunto de los números irracionales se denota Q′ , donde Q′ = R − Q; como 2 no
es racional, entonces es un número irracional. [1]
De aquı́ en adelante el cuerpo donde vamos a desarrollar la teorı́a es el cuerpo
de los números reales.
Emplearemos las siguientes notaciones: R+ , es el conjunto de los números reales
positivos, R− , es el conjunto de los números reales negativos, R+0 , es el conjunto

de los números reales no negativos, y R0 , es el conjunto de los números reales no
positivos.
Teorema 1.13
Sean a, b números reales, entonces:
(i) a2 + b2 ≥ 2ab,
(ii) 4ab ≤ (a + b)2 .
 Prueba.
(i) Sean a, b ∈ R, entonces a − b ∈ R.
Luego

(a − b)2 ≥ 0
⇔ a2 − 2ab + b2 ≥ 0
⇔ a2 + b2 ≥ 2ab.

La igualdad ocurre si y sólo si a = b.


12 1. Números Reales

(ii) De (i)

a2 + b2 ≥ 2ab
⇔ a2 + 2ab + b2 ≥ 4ab
⇔ (a + b)2 ≥ 4ab
⇔ 4ab ≤ (a + b)2 .

La igualdad ocurre si y sólo si a = b. 2


Teorema 1.14
Sean x, y números reales, tales que x > 0, y < 0, entonces
1
(i) x + ≥ 2,
x
1
(ii) y + ≤ −2.
y
 Prueba.
1 √
(i) En el teorema anterior, haciendo a2 = x, b2 = , vemos que x > 0 y a = x,
x
1
b = √ , entonces
x
1 √ 1 1
x+ ≥ 2 x · √ ⇒ x + ≥ 2.
x x x

La igualdad ocurre si y sólo si x = 1.


(ii) Si y < 0 ⇒ −y > 0.
Aplicando (i)

1
(−y) + ≥ 2;
(−y)
1
−y − ≥ 2; multiplicando por (−1)
y
1
y+ ≤ −2.
y

La igualdad ocurre si y sólo si y = −1.


2
1.3. Aplicaciones 13

1.3. Aplicaciones
1. Sean a, b números reales no nulos, determine el mı́nimo valor de

a8 a4 a2 b8 b4 b2
+ + + + + .
b8 b4 b2 a8 a4 a2

Solución.

     
a8 a4 a2 b8 b4 b2 a8 b8 a4 b4 a2 b2
+ + + + + = + + + + +
b8 b4 b2 a8 a4 a2 b8 a8 b4 a4 b2 a2
≥ 2 + 2 + 2 = 6.

La igualdad ocurre si y sólo si a = ±b. 2

2. Sean a, b, x, y números reales tales que a ≥ b, x ≥ y, pruebe que

ax + by ≥ ay + bx.

 Prueba. Como a ≥ b ⇔ a − b ≥ 0, luego multiplicando por (a − b) en


la desigualdad x ≥ y tenemos:

(a − b)x ≥ (a − b)y
⇔ ax − bx ≥ ay − by
⇔ ax + by ≥ ay + bx.

3. Dados a, b, c números reales, pruebe que

a2 + b2 + c2 ≥ ab + ac + bc.
14 1. Números Reales

 Prueba. Aplicando un resultado conocido:

a2 + b2 ≥ 2ab; similarmente
b2 + c2 ≥ 2bc;
c2 + a2 ≥ 2ca.

Sumando miembro a miembro:

2(a2 + b2 + c2 ) ≥ 2ab + 2bc + 2ca,


⇔ a2 + b2 + c2 ≥ ab + bc + ca.

La igualdad ocurre si y sólo si a = b = c. 2

4. Sean a, b, c números reales positivos, pruebe que

(a + b)(a + c)(b + c) ≥ 8abc.

 Prueba. √ √ √
a, b, c ∈ R+ ⇒ a, b, b ∈ R+ ,

luego
√ √
( a − b)2 ≥ 0

⇔ a − 2 ab + b ≥ 0

⇔ a + b ≥ 2 ab, similarmente

b + c ≥ 2 bc,

c + a ≥ 2 ca,

multiplicando miembro a miembro:



(a + b)(b + c)(c + a) ≥ 8( abc)2 = 8abc

de donde (a + b)(b + c)(c + a) ≥ 8abc.


La igualdad ocurre si y sólo si a = b = c. 2
1.3. Aplicaciones 15

5. Sean a, b números reales positivos, pruebe que

a2 b2
+ ≥ a + b.
b a

 Prueba. La desigualdad es equivalente a:

a3 + b3
≥a+b
ab
⇔ a3 + b3 ≥ ab(a + b)
⇔ (a + b)(a2 − ab + b2 ) ≥ ab(a + b); a + b > 0
⇔ a2 − ab + b2 ≥ ab
⇔ a2 − 2ab + b2 ≥ 0
⇔ (a − b)2 ≥ 0.

La igualdad ocurre si y sólo si a = b = c. 2

6. Sean a, b, c números reales positivos, pruebe que

a2 + b2 b2 + c2 c2 + a2
+ + ≥ 2(a + b + c).
c a b

 Prueba. Del ejercicio anterior tenemos:

a2 b2
+ ≥ a + b; similarmente
b a
b2 c2
+ ≥ b + c;
c b
c2 a2
+ ≥ a + c;
a c

sumando miembro a miembro:

a2 + b2 b2 + c2 c2 + a2
+ + ≥ 2(a + b + c).
c a b

La igualdad ocurre si y sólo si a = b = c. 2


16 1. Números Reales

7. Sean a, b, c números reales no negativos, donde dos de ellos simultáneamente


son diferentes de cero, pruebe que

a2 b2 c2
+ + ≥ 1.
a2 + ab + b2 b2 + bc + c2 c2 + ca + a2

 Prueba. Sean a2 + ab+ b2 = m, b2 + bc + c2 = n, c2 + ca+ a2 = p, entonces


debemos probar que

a2 b2 c2
+ + ≥1
m n p
a2 b2 c2
⇔ + + −1 ≥0
m n p
1 1 1 a2 b2 c2
⇔ ( + + )( + + − 1) ≥ 0
m n p m n p
2 2 2 2
a b c b c2 a2 c2 a2 b2 1 1 1
⇔ 2
+ 2
+ 2
+ + + + + + − − − ≥0
m n p mn mp mn np mp np m n p
2 2 2
a b c (a2 + b2 ) (a2 + c2 ) (b2 + c2 ) 1 1 1
⇔ + + + + + − − − ≥0
m2 n2 p2 mn mp np m n p
a2 b2 c2 ab (a2 + ab + b2 ) ac (a2 + ac + c2 ) bc
⇔ 2
+ 2
+ 2
− + − + −
m n p mn mn mp mp np
2 2
(b + bc + c ) 1 1 1
+ − − − ≥0
np m n p
a2 b2 c2 ab ac bc m p n 1 1 1
⇔ 2
+ 2
+ 2
− − − + + + − − − ≥0
m n p mn np np mn mp np m n p
 a 2  b 2  c 2  a   b   a   c   b   c 
⇔ + + ≥ + + ,
m n p m n m p n p

esta desigualdad es verdadera (ver aplicación 3).


La igualdad ocurre si y sólo si a = b = c. 2

8. Sea z un número real positivo, pruebe que


r
8z 2
+ ≤ 3.
z+1 z+1
1.3. Aplicaciones 17

 Prueba. La desigualdad es equivalente a:


r
8z 2
≤3−
z+1 z+1
r
8z 3z + 1
⇔ ≤
z+1 z+1
8z (3z + 1)2
⇔ ≤ , pues z > 0
z+1 (z + 1)2
⇔ 8z(z + 1) ≤ (3z + 1)2
⇔ 8z 2 + 8z ≤ 9z 2 + 6z + 1
⇔ 0 ≤ z 2 − 2z + 1
⇔ 0 ≤ (z − 1)2
⇔ (z − 1)2 ≥ 0.
2

9. Sean x, y números reales, pruebe que

(1 + x2 )(1 + y 2) ≥ (1 + xy)2 .

 Prueba. Sabemos que

x2 + y 2 ≥ 2xy, ∀ x, y ∈ R
⇔ 1 + x2 + y 2 + x2 y 2 ≥ 1 + 2xy + x2 y 2
⇔ (1 + x2 )(1 + y 2 ) ≥ (1 + xy)2 .

La igualdad ocurre si y sólo si x = y. 2

10. Pruebe que

(ax + by)2 ≤ (a2 + b2 )(x2 + y 2)

para todo x, y, a, b números reales.


18 1. Números Reales

 Prueba. Como (ay − bx)2 ≥ 0, ∀ x, y, a, b ∈ R, luego equivale a:

a2 y 2 − 2abxy + b2 x2 ≥ 0
⇔ a2 y 2 + b2 x2 ≥ 2abxy
⇔ a2 x2 + b2 y 2 + a2 y 2 + b2 x2 ≥ a2 x2 + b2 y 2 + 2abxy
⇔ a2 (x2 + y 2) + b2 (y 2 + x2 ) ≥ (ax)2 + (by)2 + 2(ax)(by)
⇔ (a2 + b2 )(x2 + y 2) ≥ (ax + by)2

∴ (ax + by)2 ≤ (a2 + b2 )(x2 + y 2).

La igualdad ocurre si y sólo si ay = bx. 2

x2 + y 2 + 2
11. Sea f (x, y) = p , donde x, y son números reales, pruebe que
(1 + x2 )(1 + y 2 )
el menor valor de f es 2.

 Prueba. Observamos que x2 + y 2 + 2 = (1 + x2 ) + (1 + y 2 ).


Luego
p
(1 + x2 ) + (1 + y 2) ≥ 2 (1 + x2 )(1 + y 2 )
p
⇔ x2 + y 2 + 2 ≥ 2 (1 + x2 )(1 + y 2 )
x2 + y 2 + 2
⇔ p ≥2
(1 + x2 )(1 + y 2 )
⇔ f (x, y) ≥ 2

∴ el mı́nimo valor de f es 2 y ocurre si y sólo si x = ±y. 2

12. Pruebe que

p √
x2 + y 2 + 1 > x y 2 + 1 + y x2 + 1,

para todo x, y números reales.


1.3. Aplicaciones 19

p
 Prueba. Como en el segundo miembro tenemos x y 2 + 1, entonces pode-
mos a partir de:
p
(x − y 2 + 1)2 ≥ 0
p
⇔ x2 + y 2 + 1 − 2x 1 + y 2 ≥ 0
p
⇔ x2 + y 2 + 1 ≥ 2x 1 + y 2; simirlamente

x2 + y 2 + 1 ≥ 2y 1 + x2 .

Sumando miembro a miembro tenemos:


p √
2(x2 + y 2 + 1) ≥ 2x 1 + y 2 + 2y 1 + x2
p √
⇔ x2 + y 2 + 1 ≥ x 1 + y 2 + y 1 + x2 .

La igualdad ocurre si y sólo si


( p
x = 1 + y2

y = 1 + x2

(
x2 = 1 + y 2

y 2 = 1 + x2
⇒ x2 + y 2 = x2 + y 2 + 2,

como esto es imposible, entonces no ocurre la igualdad.

p √
∴ x2 + y 2 + 1 > x y 2 + 1 + y x2 + 1.

13. Pruebe que

2(1 − a + a2 )(1 − b + b2 ) ≥ 1 + a2 b2 ,

para todo a, b números reales.


20 1. Números Reales

 Prueba. Veamos el primer miembro de la desigualdad:

2(1 − a + a2 )(1 − b + b2 ) = (2 − 2a + 2a2 )(1 − b + b2 )


= (1 − 2a + a2 + 1 + a2 )(1 − b + b2 )
= ((1 − a)2 + 1 + a2 )(1 − b + b2 )
= (1 − a)2 (1 − b + b2 ) + (1 + a2 )(1 − b + b2 )
= (1 − a)2 (1 − 2b + b2 + b) + (1 + a2 )(1 − b + b2 )
= (1 − a)2 ((1 − b)2 + b) + (1 + a2 )(1 − b + b2 )
= (1 − a)2 (1 − b)2 + b(1 − a)2
+(1 + a2 )(1 − b + b2 )
= (1 − a)2 (1 − b)2 + b − 2ab + a2 b + 1 + a2 − b
−a2 b + b2 + a2 b2
= (1 − a)2 (1 − b)2 + 1 + a2 b2 + a2 − 2ab + b2
= (1 − a)2 (1 − b)2 + 1 + a2 b2 + (a − b)2
= 1 + a2 b2 + (a − b)2 + (1 − a)2 (1 − b)2
≥ 1 + a2 b2 .

La igualdad ocurre si y sólo si a = b = 1. 2

14. Resuelva

1 − 3a + 5a2 − 11a3 + 12a4 > 0.

Solución. Ordenando en forma descendente

12a4 − 11a3 + 5a2 − 3a + 1 > 0


 
4 4 3 a2 9
⇔ a + 11 a − a + + a2 − 3a + 1 > 0
4 4
 
a 2 1 2
⇔ a4 + 11 a2 − + (9a − 12a + 4) > 0
2 4
 a  2 1
⇔ a4 + 11 a2 − + (3a − 2)2 > 0,
2 4
1.3. Aplicaciones 21

como el lado izquierdo es una suma de cuadrados, la desigualdad se verifica


para todo a ∈ R.

∴ C.S. = R = h−∞, +∞i.

15. (Rusia 1995). Pruebe que para todo x, y números reales positivos

x y 1
+ ≤ .
x4 + y 2 x2 + y 4 xy

 Prueba. Como

1 1
x4 + y 2 ≥ 2x2 y ⇔ ≤ 2
+yx4
2 2x y
x 1
⇔ 4 2

x +y 2xy

similarmente

y 1

x2 +y 4 2xy

sumando miembro a miembro tenemos:

x y 1
+ 2 ≤ .
x4 +y 2 x +y 4 xy

La igualdad ocurre si y sólo si a = b = c. 2

16. Pruebe que para todo a, b, c números reales positivos

8a2 − 4ab + 5b2 8b2 − 4bc + 5c2 8c2 − 4ac + 5a2


S= + + ≥ 3(a + b + c).
2a + b 2b + c 2c + a
22 1. Números Reales

 Prueba. Como

a2 + b2 ≥ 2ab
⇔ a2 − ab + b2 ≥ ab
⇔ 4a2 − 4ab + 4b2 ≥ 4ab
⇔ 8a2 − 4ab + 5b2 ≥ 4a2 + 4ab + b2
⇔ 8a2 − 4ab + 5b2 ≥ (2a + b)2
8a2 − 4ab + 5b2
⇔ ≥ 2a + b; similarmente
2a + b
8b2 − 4bc + 5c2
≥ 2b + c;
2b + c
8c2 − 4ac + 5a2
≥ 2c + a;
2c + a

sumando miembro a miembro, tenemos que

S ≥ 3(a + b + c).

La igualdad ocurre si y sólo si a = b = c. 2

17. Pruebe que

 
1 1 2 a2 + b2
2 2
+ 2 2

a + ab + b a − ab + b 3 a2 b2

para todo a, b números reales no nulos.

 Prueba. Efectuando el lado izquierdo de la desigualdad, tenemos:

1 1 2(a2 + b2 )
+ =
a2 + ab + b2 a2 − ab + b2 a4 + a2 b2 + b4

acotando el denominador
1.3. Aplicaciones 23

a4 + b4 ≥ 2a2 b2
⇔ a4 + a2 b2 + b4 ≥ 3a2 b2
1 1
⇔ 4 2 2 4
≤ 2 2
a +a b +b 3a b
2 2
2(a + b ) 2(a2 + b2 )
⇔ 4 ≤ .
a + a2 b2 + b4 3a2 b2

La igualdad ocurre si y sólo si a = ±b. 2

18. Pruebe que

x2 + y 2 + z 2
zx + yz ≤ √ ,
2

para todo x, y, z números reales.

 Prueba. Como en el lado izquierdo de la desigualdad, z es el término


común; entonces escogemos:
 2
z z
x2 + √ ≥ 2x √
2 2
z2 √
x2 + ≥ 2xz; similarmente
2
z2 √
y2 + ≥ 2yz;
2

sumando miembro a miembro, tenemos:



x2 + y 2 + z 2 ≥ 2(xz + yz)
x2 + y 2 + z 2
⇔ √ ≥ xz + yz
2

x2 + y 2 + z 2
∴ zx + yz ≤ √ .
2
z
La igualdad ocurre si y sólo si x = y = √ . 2
2
24 1. Números Reales

19. Sean a, b números reales positivos, pruebe que


 3
a3 + b3 a+b
≥ .
2 2

 Prueba. La desigualdad es equivalente a:

4(a3 + b3 ) ≥ (a + b)3 ⇔ 4a3 + 4b3 ≥ a3 + b3 + 3a2 b + 3ab2


⇔ 3a3 + 3b3 ≥ 3a2 b + 3ab2
⇔ a3 + b3 ≥ a2 b + ab2
⇔ a3 − a2 b + b3 − ab2 ≥ 0
⇔ a2 (a − b) + b2 (a − b) ≥ 0
⇔ (a2 + b2 )(a − b) ≥ 0
⇔ (a − b)2 (a + b) ≥ 0.

La igualdad ocurre si y sólo si a = b. 2

20. Pruebe que

2(a3 + b3 + c3 ) ≥ a2 (b + c) + b2 (a + c) + c2 (a + b),

para todo a, b, c números reales positivos.

 Prueba. Como a, b ∈ R+ , entonces

(a + b)(a − b)2 ≥ 0, efecuando


⇔ a3 + b3 ≥ a2 b + ab2 , similarmente
b3 + c3 ≥ b2 c + bc2
c3 + a3 ≥ c2 a + ca2

sumando miembro a miembro, tenemos que

2(a3 + b3 + c3 ) ≥ a2 (b + c) + b2 (a + c) + c2 (a + b).

La igualdad ocurre si y sólo si a = b = c. 2


CAPÍTULO 2
VALOR ABSOLUTO Y MÁXIMO
ENTERO

2.1. Valor Absoluto


Definición 2.1: En el conjunto de los números reales R, definimos el valor absoluto
de un elemento x, denotado por |x| para indicar el valor absoluto

x;
 para x > 0;
|x| = 0; para x = 0;


−x; para x < 0.

Ejemplo 2.1√ √ √ √ √ √
|2| = 2; | − 3| = −(− 3) = 3; |1 − 2| = −(1 − 2) = 2 − 1.

Teorema 2.1

|x| = máx {x; −x}.

 Prueba. 
x;
 para x > 0;
máx {x; −x} = 0; para x = 0; = |x|.


−x; para x < 0.
2

25
26 2. Valor Absoluto y Máximo Entero

Teorema 2.2
Para elementos arbitrarios x, y reales, se cumplen

(i) | − x| = |x|.

(ii) |x2 | = x2 = |x|2 .



(iii) |x| = x2 .

(iv) |xy| = |x| · |y|.

x |x|
(v) | | = .
y |y|

 Prueba.

(i) Aplicando el Teorema 2.1 tenemos

| − x| = máx {−x; −(−x)}


= máx {−x; x}
= |x|.

(ii) Por definición |x2 | = x2 , pues x2 ≥ 0. Similarmente


x;
 x > 0;
|x| = 0; x = 0;


−x; x < 0;

entonces


2
x ;
 x < 0;
|x|2 = 02 ; x = 0;

 2
(−x) ; x > 0;
= x2 , ∀ x ∈ R.
2.1. Valor Absoluto 27

(iii)
|x|2 = x2
p √
|x|2 = x2

|x| = x2 .

(iv)
(x · y)2 = x2 · y 2
p p
(x · y)2 = x2 · y 2
√ p
|xy| = x2 · y 2
|xy| = |x| · |y|

(v) Similar a (iv).

Teorema 2.3
Para cualesquiera x, y, z números reales, se cumplen

(i) |x| ≥ x.

(ii) |x| ≥ −x.

(iii) |x + y| ≤ |x| + |y|.

(iv) ||x| − |y|| ≤ |x − y|.

(v) |x − z| ≤ |x − y| + |y − z|.

 Prueba.

(i) |x| = máx {x; −x} ≥ x.

(ii) |x| = máx {x; −x} ≥ −x.


28 2. Valor Absoluto y Máximo Entero

(iii)
|xy| ≥ xy ⇔ 2|x||y| ≥ 2xy
⇔ x2 + y 2 + 2|x||y| ≥ x2 + y 2 + 2xy
⇔ |x|2 + |y|2 + 2|x||y| ≥ (x + y)2
⇔ (|x| + |y|)2 ≥ (x + y)2
⇔ |x| + |y| ≥ |x + y|
⇔ |x + y| ≤ |x| + |y|.

La igualdad ocurre si y sólo xy ≥ 0.

(iv)

|x| = |(x − y) + y| ≤ |x − y| + |y|


|x| − |y| ≤ |x − y| (2.1)

Similarmente

|y| = |(y − x) + x| ≤ |y − x| + |x|


−|y − x| ≤ |x| − |y|
−|x − y| ≤ |x| − |y| (2.2)

De (2.1) y (2.2)

−|x − y| ≤ |x| − |y| ≤ |x − y|


||x| − |y|| ≤ |x − y|.

(v)
|x − z| = |(x − y) + (y − z)| ≤ |x − y| + |y − z|
|x − z| ≤ |x − y| + |y − z|.

2
2.2. Máximo Entero o Parte Entera 29

2.2. Máximo Entero o Parte Entera


Definición 2.2: El máximo entero del número real x se denota [x] y se define de la
siguiente manera

[x] = n ⇔ n ≤ x < n + 1; donde n ∈ Z.

Ejemplo 2.2

[1,4] = 1; [2,7] = 2; [−1,3] = −2;



[ 7] = 2; [π] = 3; [−π] = −4;
[4] = 4; [−3] = −3.

Teorema 2.4
Para todo x número real:

(i) [x] ≤ x < [x] + 1.

(ii) 0 ≤ x − [x] < 1.

(iii) [[x]] = [x].

 Prueba.

(i) De la definición

[x] = n ⇔ n ≤ x < n + 1; n ∈ Z

reemplazando n = [x] se tiene

[x] ≤ x < [x] + 1.

(ii) En (i), restando [x] a todos los miembros:

0 ≤ x − [x] < 1.
30 2. Valor Absoluto y Máximo Entero

(iii) Sea [x] = m con m ∈ Z ⇒ [[x]] = [m] = m = [x].

∴ [[x]] = [x].
2
Teorema 2.5
Para todo x número real y n número entero, se cumple que
(i) [x + n] = [x] + n.
(
−[x]; si x = [x];
(ii) [−x] =
−[x] − 1; si x 6= [x].
h x i  [x] 
(iii) = , n ≥ 1.
n n

 Prueba.
(i) Sea m = [x + n]. De la definición

[x + n] = m ⇔ m ≤ x + n < m + 1
⇔ (m − n) ≤ x < (m − n) + 1; (m − n) ∈ Z
⇔ [x] = m − n
⇔ [x] + n = m
⇔ [x] + n = [x + n]

(ii) Se deja como ejercicio para el lector.


(iii)
x = k + p; k ∈ Z; 0 ≤ p < 1.
x k p haciendo k = nq + r; q ∈ Z; 0 ≤ r < n,
⇒ = + ;
n n n 0 ≤ r ≤ n − 1.
x r+p pero 0 ≤ r+p < n
=q+ ;
n n r+p
0 ≤ < 1
n
hxi
⇒ = q.
n
2.2. Máximo Entero o Parte Entera 31

También [x] = [k + p] = k

      h
[x] k nq + r ri
= = = q+ = q,
n n n n

hxi 
[x]
de donde = .
n n
2

Definición 2.3 : La parte residual de x se denota {x} y se define como

{x} = x − [x].

De la definición se tiene

0 ≤ {x} < 1 y {x} = 0 ⇔ x ∈ Z.

¿Cuáles son los valores posibles de {x} + {−x}?

Teorema 2.6 (De Hermite)


Sea x un número real, y n un número entero ≥ 1, entonces

n−1 
X 
k
x+ = [nx].
k=0
n

 Prueba. Definimos la función f : R → R


     
1 2 n−1
f (x) = [x] + x + + x+ +···+ x + − [nx] .
n n n

Vemos que f es nula:

f (x) = 0, ∀ x ∈ [0; 1/ni.


 
1
Veamos que f x + = f (x). (f de periodo 1/n), en efecto
n
32 2. Valor Absoluto y Máximo Entero

           
1 1 2 3 1 n−1 1
f x+ = x+ + x+ + x+ +··· x+ + − n(x + )
n n n n n n n
       
1 1 2 n−1
f x+ = x+ + x+ +···+ x + + [x + 1] − [nx + 1]
n n n n | {z }
[x]+1−([nx]+1)
     
1 2 n−1
= [x] + x + + x+ +···+ x + − [nx]
n n n
= f (x),

de donde f (x) = 0, ∀ x ∈ R.
n−1 
X 
k
∴ x+ = [nx].
k=0
n
2
Teorema 2.7
Sea a un número entero y b un número entero positivo, entonces
hai b − a − 1
+ = 0.
b b

 Prueba. La prueba se deja para el lector. 2

2.3. Aplicaciones
 
x2
1. Resuelva = 1.
16

Solución.  
x2 x2
=1 ⇔ 1≤ <2
16 16
⇔ 16 ≤ x2 < 32
√ √
⇔ 4 ≤ x < 4 2 ∨ 4 2 < x ≤ −4.
D h √ E
√ i
∴ C.S. = −4 2, −4 ∪ 4, 4 2 .

2
2.3. Aplicaciones 33

2. Resuelva la inecuación
 2  √
x − 1 > 2.

Solución. La inecuación:

[x2 − 1] > 2 ⇔ [x2 − 1] ≥ 2
⇔ x2 − 1 ≥ 2
⇔ x2 ≥ 3
√ √
⇔ x≥ 3 ∨ x≤− 3
D
√ i h√ E
∴ C.S. = −∞, 3 ∪ 3, +∞ .

3. Resuelva en Z, la ecuación
 
5−x
 
 
 2 =− x−2 .
 2  4

Solución. Por el teorema 2.5, el lado izquierdo de la ecuación se puede


escribir como:
   
5−x 5−x  
 2   2  5 − x
 = = .
 2  2 4

Luego resolveremos
   
5−x x−2
=−
4 4
   
5−x x−2
⇔ + = 0.
4 4
34 2. Valor Absoluto y Máximo Entero

En el teorema 2.7, haciendo a = 5 − x, b = 4


   
5−x 4 − (5 − x) − 1
⇒ + =0
4 4
   
5−x x−2
⇒ + = 0.
4 4

∴ La ecuación se verifica para todo x ∈ Z, pues es una aplicación directa del


teorema 2.7. 2

4. Pruebe que

√ √ √
[ n + n + 1] = [ 4n + 2]

para todo n número entero positivo.

√ √ √
 Prueba. Basta probar que entre n+ n+1 y 4n + 2 no existe un
entero.
La prueba la haremos por contradicción; en efecto, supongamos que existe
r ∈ Z+ tal que
√ √ √
n + n + 1 < r < 4n + 2
√ √ √
⇔ ( n + n + 1)2 < r 2 < ( 4n + 2)2
p
⇔ 2n + 1 + 2 n(n + 1) < r 2 < 4n + 2

pero

n< n2 + n

⇔ 2n < 2 n2 + n

⇔ 2n + 2n + 1 < 2 n2 + n + 2n + 1,

luego tenemos que


4n + 1 < 2 n2 + n + 2n + 1 < r 2 < 4n + 2,
2.3. Aplicaciones 35

como 4n + 1 y 4n + 2 son consecutivos, entonces no existe r 2 entre ellos, lo


cual es una contradicción a lo supuesto.
√ √ √
Luego n + n + 1 y 4n + 2 tienen el mismo máximo entero.

√ √ √
∴ [ n + n + 1] = [ 4n + 2].
2

5. Si n es un entero positivo, pruebe que


  
n − 17
 n − 12 −
 
8n + 13 25 
−



25 3

es independiente de n.

 Prueba. En el teorema 2.7, haciendo a = n - 17,b = 25, tenemos que:


   
n − 17 25 − (n − 17) − 1
+ =0
25 25
   
n − 17 41 − n
⇔ + =0
25 25
   
41 − n n − 17
⇔ =−
25 25
   
41 − n n − 17
⇔ n − 12 + = n − 12 −
25 25
   
41 − n n − 17
n − 12 + n − 12 −
25 25
⇔ = .
3 3

Tomando máximo entero:


     
41 − n n − 17
 n − 12 + 25   n − 12 − 25 
 = ,
 3   3 
36 2. Valor Absoluto y Máximo Entero

efectuando el lado izquierdo

    
41 − n 41 − n
 n − 12 + 25   n − 12 + 25 
 =  ; pues n − 12 ∈ Z
 3   3 

     
24n − 259 24n − 259 24n − 259
 25   25   3 
= 

=
 = 
3 3 25

    
24n − 259 −259
 
 3   8n + 3  8n − 87

=    
25 = 25 = 25
     
8n + 13 − 100 8n + 13 8n + 13
= = −4 = − 4,
25 25 25

  
n − 17
 n − 12 −
 
8n + 13 25 
luego −4 =

.

25 3

  
n − 17
 n − 12 −
 
8n + 13 25 
∴ −

 = 4.

25 3

2
CAPÍTULO 3
DESIGUALDAD DE
CAUCHY-SCHWARZ

Teorema 3.1 (Desigualdad de Cauchy-Schwarz)


Sean a1 , . . . , an , b1 , . . . , bn , números reales, entonces

(a1 b1 + a2 b2 + · · · + an bn )2 ≤ (a21 + a22 + · · · + a2n )(b21 + b22 + · · · + b2n ).

La igualdad ocurre si y sólo si las n-uplas (a1 , a2 , . . . , an ) y (b1 , b2 , . . . , bn ) son


colineales.

 Primera Prueba. Definimos el polinomio cuadrático

f (x) = (a1 x − b1 )2 + (a2 x − b2 )2 + · · · + (an x − bn )2 ; x ∈ R.

Vemos que f (x) ≥ 0, para todo x ∈ R.


Efectuando

f (x) = (a21 x2 − 2a1 b1 x + b21 ) + (a22 x2 − 2a2 b2 x + b22 ) + · · ·


+(a2n x2 − 2an bn x + b2n ) ≥ 0
f (x) = (a21 + a22 + · · · + a2n )x2 − 2(a1 b1 + a2 b2 + · · · + an bn )x
+(b21 + b22 + · · · + b2n ) ≥ 0
f (x) = Ax2 − 2Bx + C ≥ 0,

37
38 3. Desigualdad de Cauchy-Schwarz

donde

a21 + a22 + · · · + a2n = A,


(a1 b1 + a2 b2 + · · · + an bn ) = B,
b21 + b22 + · · · + b2n = C.

Luego Ax2 − 2Bx + C ≥ 0, para todo x ∈ R, y esto ocurre si ∆ ≤ 0. (∆:


Discriminante.)
En efecto

∆ = (−2B)2 − 4AC ≤ 0
4B 2 − 4AC ≤ 0
B 2 ≤ AC

Reemplazando, tenemos que

(a1 b1 + a2 b2 + · · · + an bn )2 ≤ (a21 + a22 + · · · + a2n )(b21 + b22 + · · · + b2n ).

La igualdad ocurre cuando f (x) presenta raı́ces reales iguales.

⇒ (a1 x − b1 ) = 0 ∧ (a2 x − b2 ) = 0 ∧ · · · ∧ (an x − bn ) = 0


⇔ a1 x = b1 ∧ a2 x = b2 ∧ · · · ∧ an x = bn

de donde (a1 , a2 , . . . , an ) y (b1 , b2 , . . . , bn ) son colineales o proporcionales. 2

n
! n
! n
!2
X X X
 Segunda Prueba. Haciendo M = a2i b2i ; N = ai bi , en-
i=1 i=1 i=1
tonces tenemos que
3.1. El Lema de Titu 39

n
! n
! n
!2
X X X
M −N = a2i b2i − ai bi
i=1 i=1 i=1
n
X X n
X X
= a2i b2i + a2i b2j − a2i b2i − 2 ai bj aj bi
i=1 i6=j i=1 1≤i<j≤n
X X
= a2i b2j − 2 ai bj aj bi
i6=j 1≤i<j≤n
X  X
= a2i b2j + a2j b2i − 2 ai bj aj bi
1≤i<j≤n 1≤i<j≤n
X 
= a2i b2j − 2ai bj aj bi + a2j b2i
1≤i<j≤n
X
= (ai bj − aj bi )2 ≥ 0.
1≤i<j≤n

Entonces
M −N ≥ 0
M ≥ N. [9]
2

3.1. El Lema de Titu


Sean a1 , a2 , . . . , an números reales arbitrarios y x1 , x2 , . . . , xn números reales po-
sitivos, se tiene la desigualdad

a21 a22 a2n (a1 + a2 + · · · + an )2


+ +···+ ≥ .
x1 x2 xn x1 + x2 + · · · + xn

 Primera Prueba. Aplicando Cauchy-Schwarz

s s 2
 
a21 a22 a2n a21 √ a2n √
+ +···+ (x1 + x2 + · · · + xn ) ≥  · x1 + · · · + · xn 
x1 x2 xn x1 xn
 
a21 a22 a2
+ +···+ n (x1 + x2 + · · · + xn ) ≥ (a1 + a2 + · · · + an )2 .
x1 x2 xn
40 3. Desigualdad de Cauchy-Schwarz

a21 a22 a2n (a1 + a2 + · · · + an )2


De donde tenemos que + +···+ ≥ . 2
x1 x2 xn x1 + x2 + · · · + xn
Como vemos, es una aplicación de la desigualdad de Cauchy-Schwarz y por ello
algunos afirman que simplemente es la desigualdad de Cauchy-Schwarz.

 Segunda Prueba. (Por inducción).


Veamos que la inducción se reduce al caso n = 2.

a21 a22 (a1 + a2 )2


+ ≥
x1 x2 x1 + x2
 
⇔ a1 x2 + a2 x1 (x1 + x2 ) ≥ x1 x2 a21 + a22 + 2a1 a2
2 2

⇔ a21 x2 x1 + a21 x22 + a22 x21 + a22 x1 x2 ≥ a21 x1 x2 + a22 x1 x2 + 2a1 a2 x1 x2


⇔ a21 x22 + a22 x21 − 2a1 a2 x1 x2 ≥ 0
⇔ (a1 x2 − a2 x1 )2 ≥ 0,

a1 a2
y la igualdad se tiene si y sólo si = .
x1 x2
Aplicando el resultado dos veces se tiene que

a21 a22 a23 (a1 + a2 )2 a23


+ + ≥ +
x1 x2 x3 x1 + x2 x3
(a1 + a2 + a3 )2
≥ .
x1 + x2 + x3
Supongamos que se cumple para n

a21 a22 a2n (a1 + a2 + · · · + an )2


+ +···+ ≥ .
x1 x2 xn x1 + x2 + · · · + xn
Veamos para n + 1

 
a21 a22 a2n a2n+1 (a1 + a2 + · · · + an )2 a2n+1
+ +···+ + ≥ +
x1 x2 xn xn+1 x1 + x2 + · · · + xn xn+1
(a1 + a2 + · · · + an+1 )2
≥ . [11]
x1 + x2 + · · · + xn+1

2
3.2. Desigualdad de Schür 41

3.2. Desigualdad de Schür


Si x, y, z son números reales positivos y n un entero positivo, entonces

xn (x − y)(x − z) + y n (y − z)(y − x) + z n (z − x)(z − y) ≥ 0.

 Prueba. Como el primer miembro es simétrico, entonces podemos asumir un


orden.
Sea x ≥ y ≥ z, entonces:

x − z ≥ y − z, x − y ≥ 0
(x − y)(x − z) ≥ (x − y)(y − z), xn ≥ y n
xn (x − y)(x − z) ≥ y n (x − y)(y − z). (3.1)

También

(z − x)(z − y) ≥ 0, pues x ≥ z, y ≥ z
n
⇔ z (z − x)(z − y) ≥ 0. (3.2)

Sumando (3.1) y (3.2)

xn (x − y)(x − z) + z n (z − x)(z − y) ≥ y n (x − y)(y − z)


xn (x − y)(x − z) − y n (x − y)(y − z) + z n (z − x)(z − y) ≥ 0
xn (x − y)(x − z) + y n (y − x)(y − z) + z n (z − x)(z − y) ≥ 0.

La igualdad ocurre si y sólo si x = y = z. 2

3.3. Dos sustituciones muy útiles


Si en las inecuaciones tenemos la condición abc = 1, es conocida la sustitución
x y z
a= , b= , c= ,
y z x
que hace que el problema se convierta en otro más fácil; aunque esto no es siempre
para todo problema.
42 3. Desigualdad de Cauchy-Schwarz

Veamos otras sustituciones que son muy útiles en la resolución de problemas de


desigualdades en los torneos internacionales.
Si tenemos las condiciones x, y, z > 0 y xyz = x + y + z + 2, nos preguntamos,
¿cuál serı́a la sustitución?
De la condición

xyz = x + y + z + 2

tenemos

xyz + xy + xz + yz + x + y + z + 1 = xy + xz + yz + 2(x + y + z) + 3
(x + 1)(y + 1)(z + 1) = (xy + x + y + 1) + (xz + x + z + 1)
+(yz + y + z + 1)
(x + 1)(y + 1)(z + 1) = (x + 1)(y + 1) + (x + 1)(z + 1)
+(y + 1)(z + 1)

1 1 1
⇒ 1= + + .
z+1 y+1 x+1
1 1 1
Haciendo a = , b= , c= , se tiene a + b + c = 1 y
x+1 y+1 z+1
1−a b+c c+a a+b
x= = , y= , z= .
a a b c

Ahora veamos otra condición.


Para x, y, z > 0 y xy + yz + zx + 2xyz = 1 la sustitución es

a b c
x= , y= , z= ,
b+c c+a a+b

pues xy + yz + zx + 2xyz = 1 equivale a

1 1 1 1
= + + + 2,
xyz x y z

es decir, en la primera sustitución sólo ha sido cambiada por la inversa de x, y, z.


[11]
3.4. Aplicaciones 43

3.4. Aplicaciones
1. Sean a, b números reales tales que a2 + b2 = 1; halle la variación de 2a + b.

Solución. Como nos piden la variación de 2a+b que es equivalente a escribir


2·a+1·b; entonces escogemos los pares (2, 1) y (a, b) para aplicar la desigualdad
de Cauchy-Schwarz.
En efecto, tenemos:

(2 · a + 1 · b)2 ≤ (22 + 12 )(a2 + b2 )


⇔ (2a + b)2 ≤ (5)(1)
⇔ (2a + b)2 ≤ 5
√ √
⇔ − 5 ≤ 2a + b ≤ 5.

√ √
∴ 2a + b ∈ [− 5, 5].

2. Sean x ≥ 1, y ≥ 1. Pruebe que


√ √ 2  
x−1 y−1 1 1
+ ≤ (x + y − 2) 2
+ 2 .
x y x y
 
√ √ 1 1
 Prueba. Consideremos los pares ( x − 1, y − 1), , y aplicando
x y
la desigualdad de Cauchy-Schwarz tenemos

 2  !
√ 1 √ 1 √ √   1 2  1 2
2 2
x−1· + y−1· ≤ x−1 + y−1 +
x y x y
 √ √ 2  
x−1 y−1 1 1
⇔ + ≤ (x − 1 + y − 1) +
x y x2 y 2
√ √ 2  
x−1 y−1 1 1
⇔ + ≤ (x + y − 2) + .
x y x2 y 2

La igualdad ocurre si y sólo si x = y. 2


44 3. Desigualdad de Cauchy-Schwarz

3. Pruebe que

(a2 + ab + b2 )2 ≤ (a2 + 2b2 )(2a2 + b2 ),

para todo a, b números reales.

 Prueba. Tomamos las ternas

(a, a, b), (a, b, b)

para aplicar la desigualdad de Cauchy-Schwarz.


En efecto

(a · a + a · b + b · b)2 ≤ (a2 + a2 + b2 )(a2 + b2 + b2 )

Luego tenemos

(a2 + ab + b2 )2 ≤ (2a2 + b2 )(a2 + 2b2 ).

La igualdad ocurre si y sólo si a = b. 2

4. Sean a, b, c números reales, tales que no son nulos simultáneamente, halle el


valor máximo de

|a + 3b + 9c|
f (a, b, c) = √ .
a2 + b2 + c2

Solución. Sean las ternas (a, b, c) y (1, 3, 9), entonces

(a · 1 + b · 3 + c · 9)2 ≤ (a2 + b2 + c2 )(12 + 32 + 92 )


⇔ (a + 3b + 9c)2 ≤ 91(a2 + b2 + c2 )
p √ p
⇔ (a + 3b + 9c)2 ≤ 91 (a2 + b2 + c2 )
|a + 3b + 9c| √
⇔ √ ≤ 91,
a2 + b2 + c2
3.4. Aplicaciones 45


como f (a, b, c) ≥ 0 entonces 0 ≤ f (a, b, c) ≤ 91.

∴ El máximo valor de f es 91, y esto ocurre si y sólo si (a, b, c) = (1, 3, 9).
2

5. Sean a, b números reales, halle la variación de

f (α) = (a sen α + b cos α).

Solución. Sean los pares (a, b), (sen α, cos α). Utilizando Cauchy-Schwarz
tenemos:

(a sen α + b cos α)2 ≤ (a2 + b2 )(sen α2 + cos α2 )


⇔ (a sen α + b cos α)2 ≤ (a2 + b2 )(1)
⇔ (a sen α + b cos α)2 ≤ a2 + b2
√ √
⇔ − a2 + b2 ≤ a sen α + b cos α ≤ a2 + b2 .

√ √
∴ − a2 + b2 ≤ f (α) ≤ a2 + b2 .

6. Sean a, b, c números reales tal que a + 2b + 3c = 14, halle la variación de


a2 + b2 + c2 .

Solución. Escogemos las ternas (1, 2, 3) y (a, b, c) para utilizar la desigual-


dad de Cauchy-Schwarz.
En efecto

(1 · a + 2 · b + 3 · c)2 ≤ (12 + 22 + 32 )(a2 + b2 + c2 )


⇔ 142 ≤ 14(a2 + b2 + c2 )
⇔ 14 ≤ a2 + b2 + c2

∴ a2 + b2 + c2 ≥ 14.

La igualdad ocurre si y sólo si (a, b, c) = (1, 2, 3). 2


46 3. Desigualdad de Cauchy-Schwarz

7. Pruebe que para todo x, y, z números reales positivos

 1/2  1/2  1/2


x+y x+z y+z
+ + ≤ 61/2 .
x+y+z x+y+z x+y+z

 Prueba. Tomamos las ternas

 1/2  1/2  1/2 !


x+y x+z y+z
, , ; (1, 1, 1),
x+y+z x+y+z x+y+z

aplicando Cauchy-Schwarz:

 1/2  1/2  1/2 !2


x+y x+z y+z
·1+ ·1+ ·1
x+y+z x+y+z x+y+z
 
x+y x+z y+z
≤ + + · (12 + 12 + 12 )
x+y+z x+y+z x+y+z
 
2(x + y + z)
= · (3)
x+y+z
= 6.

 1/2  1/2  1/2


x+y x+z y+z
∴ + + ≤ 61/2 .
x+y+z x+y+z x+y+z

La igualdad ocurre si y sólo si x = y = z. 2

8. Pruebe que

 
x2 y2 z2
x+y+z ≤2 + + ,
y+z x+z x+y

para todo x, y, z números reales positivos.


3.4. Aplicaciones 47

 Prueba. Como

 2
2 x √ y √ z √
(x + y + z) = √ · y+z+ √ · x+z+ √ · x+y
y+z x+z x+y
 2 
x y2 z2
≤ + + (y + z + x + z + x + y)
y+z x+z x+y
 2 
2 x y2 z2
(x + y + z) ≤ 2 (x + y + z) + + ;
y+z x+z x+y

cancelando x + y + z > 0

 
x2 y2 z2
x+y+z ≤2 + + .
y+z x+z x+y

La igualdad ocurre si y sólo si x = y = z. 2

1 1 1
9. Sean x, y, z > 1 tales que + + = 2, pruebe que
x y z
√ √ p √
x+y+z ≥ x−1+ y − 1 + z − 1.

 Prueba. Para aplicar Cauchy-Schwarz, tomamos las ternas

√ √ √ 
x−1 y−1 z−1 √ √ √ 
√ , √ , √ y x, y, z ;
x y z

en efecto tenemos que

√ √ √ 2
x−1 √y−1 √ z−1 √
√ · x+
√ · y+ √ · z
x y z
 
x−1 y−1 z−1
≤ + + · (x + y + z)
x y z
48 3. Desigualdad de Cauchy-Schwarz

√ 2  
p √ 1 1 1
⇔ x−1+ y−1+ z−1 ≤ 1− +1− +1−
x y z
· (x + y + z)
  
1 1 1
= 3− + + (x + y + z)
x y z
= (3 − 2) (x + y + z)
= (x + y + z).

√ p √ 2
Entonces x−1+ y−1+ z−1 ≤ x + y + z.
√ p √ √
∴ x − 1 + y − 1 + z − 1 ≤ x + y + z.

3
La igualdad ocurre si y sólo si x = y = z = . 2
2

10. (China 1984/1985). Sean a1 , a2 , . . . , an > 0, pruebe que

a21 a22 a2 a2
+ + · · · + n−1 + n ≥ a1 + a2 + · · · + an .
a2 a3 an a1

 Prueba. Aplicando el Lema de Titu.

a21 a22 a2 a2 (a1 + a2 + · · · + an−1 + an )2


+ + · · · + n−1 + n ≥
a2 a3 an a1 a2 + a3 + · · · + an + a1
= a1 + a2 + · · · + an .

La igualdad ocurre si y sólo si a1 = a2 = · · · = an . 2

11. (Desigualdad de Nesbit) Pruebe que

a b c 3
+ + ≥ ,
b+c a+c a+b 2

para todo a, b, c números reales positivos.


3.4. Aplicaciones 49

 Prueba.

a b c a2 b2 c2
+ + = + +
b+c a+c a+b ab + ac ba + bc ca + cb
(a + b + c)2

(ab + ac) + (ba + bc) + (ca + cb)
(a + b + c)2
= ,
2(ab + bc + ca)

basta demostrar que

(a + b + c)2 3
≥ .
2(ab + bc + ca) 2

Como

a2 + b2 + c2 ≥ ab + ac + bc
⇔ a2 + b2 + c2 + 2(ab + ac + bc) ≥ 3(ab + ac + bc)
(a + b + c)2 3
⇔ ≥ .
2(ab + bc + ca) 2

La igualdad ocurre si y sólo si a = b = c. 2

12. Pruebe que para todo a, b, c números reales positivos

a3 b3 c3 a+b+c
2 2
+ 2 2
+ 2 2
≥ .
a + ab + b b + bc + c c + ca + a 3

 Prueba. El lado izquierdo de la desigualdad se puede escribir como

a4 b4 c4
+ + ,
a3 + a2 b + ab2 b3 + b2 c + bc2 c3 + c2 a + ca2
50 3. Desigualdad de Cauchy-Schwarz

luego, aplicando el Lema de Titu, tenemos:

a4 b4 c4
+ +
a3 + a2 b + ab2 b3 + b2 c + bc2 c3 + c2 a + ca2
(a2 + b2 + c2 )2
≥ 3
a + b3 + c3 + ab(a + b) + bc(b + c) + ca(c + a)
(a2 + b2 + c2 )2
=
(a + b + c)(a2 + b2 + c2 )
a2 + b2 + c2
= ,
a+b+c

entonces

a3 b3 c3 a2 + b2 + c2
+ + ≥ .
a2 + ab + b2 b2 + bc + c2 c2 + ca + a2 a+b+c

Pero

a2 + b2 + c2 ≥ ab + ac + bc
⇔ 2(a2 + b2 + c2 ) ≥ 2(ab + ac + bc)
⇔ 3(a2 + b2 + c2 ) ≥ a2 + b2 + c2 + 2(ab + ac + bc)
⇔ 3(a2 + b2 + c2 ) ≥ (a + b + c)2
a2 + b2 + c2 a+b+c
⇔ ≥ .
a+b+c 3

De donde

a3 b3 c3 a+b+c
2 2
+ 2 2
+ 2 2
≥ .
a + ab + b b + bc + c c + ca + a 3

La igualdad ocurre si y sólo si a = b = c. 2

13. Sean a, b, c números reales positivos. Pruebe que

a3 + b3 + c3 + 3abc ≥ ab(a + b) + bc(b + c) + ca(c + a).


3.4. Aplicaciones 51

 Prueba. La desigualdad es equivalente a

a3 + b3 + c3 + 3abc − a2 b − ab2 − b2 c − bc2 − c2 a + ca2 ≥ 0


(a3 − a2 (b + c) + abc) + (b3 − b2 (a + c) + abc) + (c3 − c2 (b + a) + abc) ≥ 0
a (a2 − (b + c)a + bc) + b (b2 − (a + c)b + ac) + c (c2 − (b + a)c + ab) ≥ 0
a(a − b)(a − c) + b(b − a)(b − c) + c(c − a)(c − b) ≥ 0,

y ésta es la desigualdad de Schür para n = 1.


La igualdad ocurre si y sólo si a = b = c. 2

14. Pruebe la desigualdad de Nesbit utilizando las sustituciones útiles

a b c 3
+ + ≥ ,
b+c a+c a+b 2

para todo a, b, c números reales positivos.

a b c
 Prueba. Haciendo x = , y= , z= .
b+c c+a a+b
Entonces es suficiente probar que si x, y, z > 0 con xy + yz + zx + 2xyz = 1,
entonces x + y + z ≥ 3/2.
3
Supongamos que x + y + z < .
2
Como

(x + y + z)2 3
xy + yz + zx ≤ ⇒ xy + yz + zx <
3 4
 3
x+y+z 1
xyz ≤ ⇒ 2xyz <
3 4

de donde

3 1
1 = (xy + yz + zx) + 2xyz < + = 1.
4 4
52 3. Desigualdad de Cauchy-Schwarz

Lo cual es una contradicción, entonces

x + y + z ≥ 3/2.

15. Pruebe que si x, y, z son números reales positivos y xyz = x+y +z +2 entonces

xy + yz + zx ≥ 2(x + y + z).

 Prueba. De x, y, z > 0 y xyz = x + y + z + 2, tenemos la sustitución

b+c c+a a+b


x= , y= , z= .
a b c

Entonces será suficiente demostrar

        
b+c c+a c+a a+b a+b b+c
+ +
a b b c c a
 
b+c c+a a+b
≥2 + +
a b c
⇔ a3 + b3 + c3 + 3abc ≥ ab(a + b) + bc(b + c) + ca(c + a)
⇔ a3 + b3 + c3 + 3abc ≥ a2 (b + c) + b2 (a + c) + c2 (a + b)
⇔ (a3 − a2 (b + c) + abc) + (b3 − b2 (a + c) + abc) + (c3 − c2 (a + b) + abc) ≥ 0
⇔ a(a − b)(a − c) + b(b − a)(b − c) + c(c − a)(c − b) ≥ 0.

Esta última desigualdad es verdadera, pues es la desigualdad de Schür.


La igualdad ocurre si y sólo si x = y = z = 2. 2

16. Pruebe que para todo a, b, c números reales positivos:

(b + c − a)2 (c + a − b)2 (a + b − c)2 3


2 2
+ 2 2
+ 2 2
≥ .
(b + c) + a (c + a) + b (a + b) + c 5
3.4. Aplicaciones 53

 Prueba. En cada fracción del primer miembro dividimos por a2 , b2 , c2


respectivamente y se tiene

 2  2  2
b+c c+a a+b
−1 −1 −1
a b c 3
 2 + 2 + 2 ≥ .
b+c c+a a+b 5
+1 +1 +1
a b c

Utilizando la sustitución

b+c c+a a+b


= x, = y, = z,
a b c

tenemos la desigualdad equivalente

(x − 1)2 (y − 1)2 (y − 1)2 3


2
+ 2 + 2 ≥
x +1 y +1 y +1 5

con xyz = x + y + z + 2, x + y + z ≥ 6.
Aplicando el Lema de Titu

(x − 1)2 (y − 1)2 (y − 1)2 (x + y + z − 3)2


+ + ≥ .
x2 + 1 y2 + 1 y2 + 1 x2 + y 2 + z 2 + 3

Luego será suficiente probar

(x + y + z − 3)2 3
≥ .
x2 + y 2 + z 2 + 3 5
⇔ 5(x + y + z − 3)2 ≥ 3(x2 + y 2 + z 2 ) + 9 (3.3)

Haciendo

x + y + z = s ⇒ x2 + y 2 + z 2 = (x + y + z)2 − 2(xy + xz + yz)


= s2 − 2(xy + xz + yz).
54 3. Desigualdad de Cauchy-Schwarz

Reemplazando en (3.3)

5(s − 3)2 ≥ 3(s2 − 2(xy + xz + yz)) + 9

efectuando

5s2 − 30s + 45 ≥ 3s2 − 6(xy + xz + yz) + 9


⇔ 2s2 − 30s + 36 ≥ −6(xy + xz + yz)
⇔ s2 − 15s + 18 ≥ −3(xy + xz + yz). (3.4)

Pero de la aplicación 15

xy + yz + zx ≤ 2(x + y + z) = 2s
⇔ xy + yz + zx ≤ 2s
⇔ 3(xy + yz + zx) ≤ 6s
⇔ −6s ≤ −3(xy + yz + zx). (3.5)

De (3.4) y (3.5) es suficiente probar

s2 − 15s + 18 ≥ −6s
⇔ s2 − 9s + 18 ≥ 0
⇔ (s − 3)(s − 6) ≥ 0,

es verdadero, pues s ≥ 6.
La igualdad ocurre si y sólo si a = b = c. 2
CAPÍTULO 4
DESIGUALDAD DE LA MEDIA
ARITMÉTICA - MEDIA GEOMÉTRICA

En las olimpiadas de matemáticas, en lo que corresponde al capı́tulo de de-


sigualdades, una de las propiedades muy utilizadas es la Media Aritmética - Media
Geométrica; en tal sentido, mostraremos una serie de ejercicios que nos permitan
profundizar.
Antes de enunciar los teoremas, definiremos la Media Aritmética, Media Ge-
ométrica y Media Armónica de los números x1 , x2 , . . . , xn reales positivos, de la
siguiente manera:

x1 + x2 + · · · + xn
MA = ,
n

MG = n x1 x2 · · · xn ,
n
MH = .
1 1 1
+ +···+
x1 x2 xn

Teorema 4.1
Sean x1 , x2 , . . . , xn números reales positivos entonces su Media Aritmética es mayor
o igual que su Media Geométrica (MA ≥ MG).
La igualdad ocurre si y sólo si x1 = x2 = · · · = xn .

 Prueba. Sea Pn = {MA ≥ MG para n números}; probemos por inducción


matemática de la siguiente manera

55
56 4. Desigualdad de la Media Aritmética - Media Geométrica

(i) Probaremos que se cumple para dos números, es decir P2 es verdadero.

(ii) Tomando la hipótesis que Pn es verdadero, probaremos que Pn−1 es verdadero.

(iii) Ası́ mismo probaremos que si Pn es verdadero entonces P2n es verdadero.

Cuando (i), (ii) y (iii) se verifican, entonces Pn con n ≥ 2 es verdadero.


Veamos
√ √ √ √
(i) Como x1 , x2 ∈ R+ , entonces x1 , x2 ∈ R+ , luego ( x1 − x2 )2 ≥ 0.
Efectuando

x1 − 2 x1 x2 + x2 ≥ 0

⇔ x1 + x2 ≥ 2 x1 x2
x1 + x2 √
⇔ ≥ x1 x2 .
2

La igualdad ocurre si y sólo si x1 = x2 .


Vemos que P2 es verdadero.

(ii) Sea g = n−1 x1 x2 · · · xn−1 , entonces g n−1 = x1 x2 · · · xn−1 .
Como Pn es verdadero, entonces

x1 + x2 + · · · + xn−1 + g √
≥ n x1 x2 · · · xn−1 · g
n
x1 + x2 + · · · + xn−1 + g p
⇔ ≥ n g n−1 · g
n

⇔ x1 + x2 + · · · + xn−1 + g ≥ n · n g n = ng
⇔ x1 + x2 + · · · + xn−1 ≥ g(n − 1)
x1 + x2 + · · · + xn−1
⇔ ≥ g.
n−1

De donde

x1 + x2 + · · · + xn−1 √
≥ n−1
x1 x2 · · · xn−1 .
n−1

Vemos que Pn−1 es verdadero.


57

(iii) Sean x1 , x2 , . . . , x2n números reales positivos, entonces

x1 + x2 + · · · + x2n = (x1 + x2 ) + (x3 + x4 ) + · · · + (x2n−1 + x2n )


√ √ √
≥ 2( x1 x2 + x3 x4 + · · · + x2n−1 x2n ) (4.1)

Como Pn es verdadero, entonces

√ √ √ q√
x3 x4 + · · · + x2n−1 x2n
x1 x2 + √ √
≥ n x1 x2 · x3 x4 · · · x2n−1 x2n
n q
√ √ √ √
⇔ x1 x2 + x3 x4 + · · · + x2n−1 x2n ≥ n n x1 x2 x3 x4 · · · x2n−1 x2n
√ √ √ √
⇔ 2 x1 x2 + x3 x4 + · · · + x2n−1 x2n ≥ 2n 2n x1 x2 x3 x4 · · · x2n−1 x2n . (4.2)

De (4.1) y (4.2) tenemos que


x1 + x2 + · · · + x2n ≥ 2n 2n x1 x2 · · · x2n
x1 + x2 + · · · + x2n √
⇔ ≥ 2n x1 x2 · · · x2n ,
2n

de donde P2n es verdadero.

Colorario 4.1 (Desigualdad de la Media Geométrica - Media Armónica)


Sean x1 , x2 , . . . , xn números reales positivos, entonces su Media Geométrica es
mayor o igual que su Media Armónica (MG ≥ MH).
La igualdad ocurre si y sólo si x1 = x2 = · · · = xn .

1 1 1
 Prueba. Aplicando el teorema 4.1 a los números , ,..., reales positivos,
x1 x2 xn
tenemos:

1 1 1 r
+ +···+
x1 x2 xn 1 1 1
≥ n · ··· ,
n x1 x2 xn
58 4. Desigualdad de la Media Aritmética - Media Geométrica

1 1 1 n
⇔ + +···+ ≥ √
x1 x2 xn n
x1 x2 · · · xn
√ n
⇔ n x1 x2 · · · xn ≥ ,
1 1 1
+ +···+
x1 x2 xn

la igualdad ocurre si y sólo si x1 = x2 = · · · = xn . 2

Colorario 4.2 (Desigualdad de la Media Aritmética - Media Armónica)


Sean x1 , x2 , . . . , xn números reales positivos, entonces
x1 + x2 + · · · + xn n
≥ .
n 1 1 1
+ +···+
x1 x2 xn

 Prueba. La demostración es inmediata, pues por transitividad, se tendrı́a, ya


que

MA ≥ MG ∧ MG ≥ MH ⇒ MA ≥ MH.

Es importante observar que de MA ≥ MH


x1 + x2 + · · · + xn n
≥ ,
n 1 1 1
+ +···+
x1 x2 xn
se tiene
 
1 1 1
(x1 + x2 + · · · + xn ) + +···+ ≥ n2 .
x1 x2 xn

4.1. Aplicaciones
1. Sean a, b, c números reales positivos. Pruebe que

a b c
+ + ≥ 3.
b c a
4.1. Aplicaciones 59

 Prueba. Como a, b, c ∈ R+ , entonces podemos utilizar MA - MG.


En efecto

a b c r
+ +
b c a ≥ 3 a·b·c
3 b c a
a b c √
+ + ≥ 3· 31
b c a
a b c
+ + ≥ 3.
b c a

La igualdad ocurre si y sólo si a = b = c.


2

2. Sean a, b, c números reales positivos. Pruebe que

a4 b4 c4
+ + ≥ 3abc.
b c a

a4 b4 c4
 Prueba. Utilizando MA - MG, a los números , , .
b c a
En efecto

a4 b4 c4 r
+ + 4 4
b c a ≥ 3 a4 b · b · c
3 c a
4 4 4
a b c
+ + ≥ 3abc.
b c a

La igualdad ocurre si y sólo si a = b = c. 2

3. Sabiendo que a, b, c, d son números reales positivos con abcd = 1, pruebe que

a2 + b2 + c2 + d2 + ab + ac + ad + bc + bd + cd ≥ 10.
60 4. Desigualdad de la Media Aritmética - Media Geométrica

 Prueba. Utilizando MA ≥ MG se tiene:

a2 + b2 + c2 + d2 + ab + ac + ad + bc + bd + cd p
≥ 10 (abcd)4
10
2 2 2 2

a + b + c + d + ab + ac + ad + bc + bd + cd ≥ 10 · 10 1

∴ a2 + b2 + c2 + d2 + ab + ac + ad + bc + bd + cd ≥ 10.

La igualdad ocurre si y sólo si a = b = c = d = 1. 2

4. (Australia 2000). Sea a un número real no nulo, b un número real, pruebe que

1 b √
a2 + b2 + + ≥ 3.
a2 a

 Prueba. Agrupando convenientemente

1 b
a2 + b2 + 2 +
a a
   
2 1 b 2 3
= b + 2+ + a + 2
4a a 4a
 2  
1 3
= b+ + a2 + 2
2a 4a

y además
r √
3 3 2 3 √
a2 + 2 ≥ 2 a2 · 2 = = 3
4a 4a 2
3 √
⇔ a2 + 2 ≥ 3
4a
 2    2
1 2 3 1 √ √
⇔ b+ + a + 2 ≥ b+ + 3 ≥ 3.
2a 4a 2a

1 b √
∴ a2 + b2 + 2
+ ≥ 3.
a a
1
La igualdad ocurre si y sólo si b = − . 2
2a
4.1. Aplicaciones 61

5. Pruebe que

a2 + 2b ≥ 3(ab)2/3

para todo a, b números reales positivos.

 Prueba. Observamos que a2 +2b = a2 +b+b, luego aplicamos MA ≥ MG


a los números a2 , b, b. En efecto tenemos:

a2 + b + b √
3
≥ a2 · b · b
3
√3
⇔ a2 + 2b ≥ 3 a2 b2
⇔ a2 + 2b ≥ 3(ab)2/3 .

La igualdad ocurre si y sólo si a2 = b. 2

6. Halle el menor valor de

4
f (x) = x2 + √ , x > 0.
x

Solución. Vemos que

4 1 1 1 1
f (x) = x2 + √ = x2 + √ + √ + √ + √
x x x x x

1 1 1 1
entonces aplicamos MA ≥ MG a los números x2 , √ , √ , √ , √ , veamos:
x x x x
1 1 1 1
x2 + √ + √ + √ + √ r
x x x x 1 1 1 1
≥ 5 x2 · √ · √ · √ · √
5 x x x x
4
⇔ x2 + √ ≥ 5(1)
x

entonces f (x) ≥ 5.
Por lo tanto, el mı́nimo de f es 5 y ocurre si y sólo si x = 1. 2
62 4. Desigualdad de la Media Aritmética - Media Geométrica

7. Pruebe que
r r r
2+1 3 3+1 n n+1
M =1+ + +···+ < n + 1.
2 3 n

r
k k+1
 Prueba. El término general de la suma del primer miembro es
k
y vemos que

(k − 1) sumandos
r v k+1 z }| {
k+1 uk + 1 +1+1+···+1
u
k
= t
k · 1| · 1{z· · · 1} < k
k k k
(k − 1) factores

r k+1
k+1 +k−1 1
⇔ k
< k = 1 + 2.
k k k

En efecto
     
1 1 1
M < 1+ 1+ 2 + 1+ 2 +···+ 1+ 2
2 3 n
1 1 1
= n+ 2 + 2 +···+ 2
2 3 n
1 1 1
< n+ + +···+
1·2 2·3 (n − 1) · n
     
1 1 1 1 1 1
= n+ − + − +···+ −
1 2 2 3 n−1 n
1
= n + 1 − < n + 1. [10, pág 17]
n
2

8. Pruebe que
r √ r √
n
n
n n
n
n
1+ + 1− < 2,
n n

siendo n entero positivo.


4.1. Aplicaciones 63

 Prueba.

r √ v  √ 
n
n √
u 1 + + (n − 1)
n
n
n u n
n n
n
n
1+ = t 1+
n · 1| · 1{z· · · 1} < =1+ 2 .
n n n n
(n − 1) factores

Similarmente
r √ √
n n n n
n
1− <1− 2 ,
n n

entonces
r √ r √ √ √
n n
n
n n
n n
n n
n
1+ + 1− < 1 + 2 + 1 − 2 = 2.
n n n n

r √ r √
n
n
n n
n
n
∴ 1+ + 1− < 2. [10, pág 17]
n n
2

9. Sean a, b, c números reales positivos con a + b + c = 3, halle el máximo valor


de
p
3
p
3
p
3
f (a, b, c) = a(b + 2c) + b(c + 2a) + c(a + 2b).

 Prueba. Como f (a, b, c) es una expresión simétrica para a, b, c entonces


el máximo ocurre cuando a = b = c y como a + b + c = 3

a = b = c = 1

⇒ 3a = 3b = 3c = 3


b + 2c = c + 2a = a + 2b = 3.

Luego aplicando MA ≥ MG, de la siguiente manera:


 
p
3 1 p 3 1 3a + (b + 2c) + 3
a(b + 2c) = √ 3a(b + 2c) · 3 ≤ √
3
9 3
9 3
64 4. Desigualdad de la Media Aritmética - Media Geométrica

 
p 1 3a + b + 2c + 3
⇔ a(b + 2c) ≤ √
3
; similarmente
3
9 3
 
p 1 3b + c + 2a + 3
3
b(c + 2a) ≤ √ ;
3
9 3
 
p 1 3c + a + 2b + 3
3
c(a + 2b) ≤ √ .
3
9 3

Sumando miembro a miembro tenemos

 
p3
p p 1 6(a + b + c) + 9
a(b + 2c) + 3 b(c + 2a) + 3 c(a + 2b) ≤ √ ,
| {z } 3
9 3
f (a,b,c)

1 √
3
⇒ f (a, b, c) ≤ √
3
· 9 = 3 3.
9


∴ máx f = 3 3 3. [10, pág 16] 2

10. (APMO 1998). Sean a, b, c números reales positivos. Pruebe que


 
x  y z 2(x + y + z)
1+ 1+ 1+ ≥2+ √ .
y z x 3 xyz

     
x y z 2x y 2y z
 Prueba. Observar que 3 + + = + + + +
  y z x y z z x
2z x
+ .
x y
Utilizando MA ≥ MG, tenemos que
r
2x y x x y x x y
+ = + + ≥ 33 · · , entonces
y z y y z y y z
s
2x y x2
+ ≥ 33 , similarmente
y z yz
4.1. Aplicaciones 65

r
2
2y z 3 y
+ ≥ 3 ;
z x xz
s
2z x z2
+ ≥ 33
x y xy

de donde tenemos que:

  s r s
x y z 3 x2 3 y2 3 z2
3 + + ≥ 3· +3· +3·
y z x yz xz xy
 
x y z
= 3 √3 xyz
+ √3 xyz
+ √
3 xyz

x y z x+y+z
⇔ + + ≥ √ ; similarmente
y z x 3 xyz

x z y x+y+z
+ + ≥ √ ,
z y x 3 xyz

Sumando miembro a miembro tenemos

x y z x z y 2(x + y + z)
+ + + + + ≥ √ ,
y z x z y x 3 xyz

sumando 2 en ambos miembros

x y z x y x z y z x y z 2(x + y + z)
1+ + + + · + · + · + · · ≥2+ √ ,
y z x y z y x z x y z x 3 xyz
| {z }
1

de donde
 
x  y z 2(x + y + z)
1+ 1+ 1+ ≥2+ √ .
y z x 3 xyz

La igualdad ocurre si y sólo si x = y = z. 2


66 4. Desigualdad de la Media Aritmética - Media Geométrica

11. Sean a, b, c lados de un triángulo con perı́metro 3. Pruebe que

1 1 1 9
√ +√ +√ ≥ . (4.3)
a+b−c b+c−a c+a−b ab + bc + ca

 Prueba. Haciendo
√ √ √
x= a + b − c; y = b + c − a; z = c + a − b,

x2 + z 2 x2 + y 2
entonces x2 + y 2 + z 2 = a + b + c = 3 y a = , b = ,
2 2
y2 + z2
c= , de donde se deduce que:
2
9 + x2 y 2 + y 2z 2 + z 2 x2
ab + ac + bc = .
4

Luego la desigualdad (4.3) es equivalente a

1 1 1 36
+ + ≥
x y z 9 + x y + y 2z 2 + z 2 x2
2 2
p
⇔ (yz + xz + xy) (9 + (xy)2 + (yz)2 + (zx)2 ) ≥ 36xyz = 36 (xyz)2 .

Haciendo xy = m, xz = n, yz = p, tenemos


(m + n + p)(9 + m2 + n2 + p2 ) ≥ 36 mnp.

Pero aplicando MA ≥ MG, tenemos que


m + n + p ≥ 3 3 mnp (4.4)
9 sumandos
z }| {
m2 + n2 + p2 + 1 + 1 + · · · + 1 p
≥ 12 m2 n2 p2
12 p
m2 + n2 + p2 + 9 ≥ 12 12 (mnp)2

m2 + n2 + p2 + 9 ≥ 12 6 mnp. (4.5)
4.1. Aplicaciones 67

Multiplicando miembro a miembro (4.4) y (4.5)


√  √ 
(m + n + p)(m2 + n2 + p2 + 9) ≥ 3 3 mnp 12 6 mnp

= 36 mnp,

de donde obtenemos:


(m + n + p)(m2 + n2 + p2 + 9) ≥ 36 mnp.

La igualdad ocurre si y sólo si a = b = c = 1. 2

12. Pruebe que para x, y números reales no negativos, con x ≥ y, y n entero


positivo, se cumple que

n−1
n(x − y)(xy) 2 ≤ xn − y n .

 Prueba. Hay dos casos:

(i) Si x = y, se cumple la igualdad.


(ii) Si x 6= y

n−1 xn − y n
n(xy) 2 ≤ ; x>y
x−y
n−1
n(xy) 2 ≤ xn−1 + xn−2 y + xn−3 y 2 + · · · + xy n−2 + y n−1.

Aplicando MA ≥ MG

xn−1 + xn−2 y + · · · + y n−1 p


> n (xn−1 ) · (xn−2 y) · (xn−3 y 2 ) · · · (y n−1)
n
q
n (n−1)n (n−1)n n−1
= x 2 · y 2 = (xy) 2

Por lo tanto de (i) y (ii) tenemos


n−1
xn−1 + xn−2 y + · · · + y n−1 ≥ n(xy) 2 .

2
68 4. Desigualdad de la Media Aritmética - Media Geométrica
CAPÍTULO 5
DESIGUALDAD DE BERNOULLI Y LA
MEDIA POTENCIAL

La desigualdad de Bernoulli es muy importante, puesto que es muy utilizada en


el análisis matemático y en otras ramas de la matemática.

Teorema 5.1
Si x ≥ −1 y n entero positivo, entonces

(1 + x)n ≥ 1 + nx.

 Prueba. La demostración la haremos por inducción.

Si n = 1: 1 + x ≥ 1 + x, es verdadera.

Supongamos que se cumpla para n = k, es decir:

(1 + x)k ≥ 1 + kx (hipótesis inductiva).

Veremos que se cumple para n = k + 1.


Multiplicando por (1 + x) en la desigualdad anterior

(1 + x)k (1 + x) ≥ (1 + kx)(1 + x)
⇔ (1 + x)k+1 ≥ 1 + (k + 1)x + kx2 ≥ 1 + (k + 1)x

69
70 5. Desigualdad de Bernoulli y la Media Potencial

de donde

(1 + x)k+1 ≥ 1 + (k + 1)x.

∴ (1 + x)n ≥ 1 + nx.

2
Teorema 5.2
Si x ≥ −1 y 0 < α < 1, entonces

(1 + x)α ≤ 1 + αx.

 Prueba. Si x = −1, la desigualdad se verifica trivialmente.


m
Veamos para x > −1. Sea α racional, α = , m, n ∈ Z+ con 1 ≤ m < n.
n
Como 1 + x > 0, tenemos

m p p
(1 + x)α = (1 + x) n = n
(1 + x)m = n
(1 + x)m · 1n−m
s
= (1 + x) · (1 + x) · · · (1 + x) · 1| · 1{z· · · 1}
n | {z }
m n−m

m n−m
z }| { z }| {
(1 + x) + (1 + x) + · · · + (1 + x) + 1 + 1 + · · · + 1

m + (n − m)
m(1 + x) + n − m n + mx m
= = = 1 + x = 1 + αx,
n n n

de donde se tiene

(1 + x)α ≤ 1 + αx, α ∈ Q, 0 < α < 1.

Ahora veremos para α ∈ Q′ . (Q′ conjunto de los números irracionales.)


Sea (qk )k∈N = (q1 , q2 , q3 , . . . , qk , . . .) una sucesión de números racionales tal que
α = lı́m qk y 0 < qk < 1.
k→∞
En efecto

(1 + x)qk ≤ 1 + qk x; x ≥ −1, k = 1, 2, 3, . . .
71

luego

(1 + x)α = lı́m (1 + x)qk ≤ lı́m (1 + qk x) = 1 + αx.


k→∞ k→∞

Para completar la prueba, veamos que 0 < α < 1 y x 6= 0, se tiene

(1 + x)α < 1 + αx;

tomemos un númeroh racional iq tal que α < q < 1.


α q
Y como (1 + x)α = (1 + x) q se cumple, entonces
α α α
(1 + x) q ≤ 1 + x; pues 0 < < 1
q q
 q
α α
⇔ (1 + x)α ≤ 1 + x < 1 + q · x; o sea
q q
(1 + x)α < 1 + αx.

Luego la desigualdad ocurre si y sólo si x = 0. 2

Teorema 5.3
Si x ≥ −1 y (α < 0 ∨ α > 1), se tiene

(1 + x)α ≥ 1 + αx.

 Prueba. Si 1 + αx < 0, la desigualdad es evidente, pues el primer miembro es


no negativo.
Si 1 + αx ≥ 0, es decir αx ≥ −1, consideremos dos casos:

Sea α > 1, entonces por el teorema 5.2

1 1
(1 + αx) α ≤ 1 + · αx = 1 + x,
α

de donde

1 + αx ≤ (1 + x)α ⇔ (1 + x)α ≥ 1 + αx.


72 5. Desigualdad de Bernoulli y la Media Potencial

Sea α < 0, tomemos un entero n positivo tal que − αn < 1, luego por el teorema
5.2
 α
−α
(1 + x) n ≤ 1+ − x
n
α 1 α
⇔ (1 + x) n ≥ α ≥ 1 + n x.
1− x
n

La igualdad sólo es posible si x = 0. 2

5.1. Media Potencial


Definición 5.1 : Dados x1 , x2 , . . . , xn números reales positivos, el número
  α1
xα1 + xα2 + · · · + xαn
Mα =
n

es la media potencial de grado α de los números x1 , x2 , · · · , xn . En particular


x1 + x2 + · · · + xn
M1 = es la media aritmética,
n
 2 1
x1 + x22 + · · · + x2n 2
M2 = es la media cuadrática,
n
 −1 
−1 −1
x1 + x−1 2 + · · · + xn
M−1 = es la media armónica.
n

Teorema 5.4
Si x1 , x2 , . . . , xn números reales positivos y α < 0 < β, entonces

Mα ≤ MG ≤ Mβ .

 Prueba. Como MG ≤ MA, entonces


p xα1 + xα2 + · · · + xαn
n
xα1 · xα2 · · · xαn ≤ .
n
1 1
De α < 0 < β, tenemos α
< 0, entonces elevando a la potencia α
5.1. Media Potencial 73

α α
 1
α α
√ x 1 + x 2
1+ · · · + xn
( xα1 · xα2 · · · xαn ) ≥
n α
n
 α 1
√ x1 + xα2 + · · · + xαn α
⇔ n
x1 · x2 · · · xn ≥ = Mα ,
n

de donde Mα ≤ MG.
Ası́ mismo
q
xβ1 + xβ2 + · · · + xβn
xβ1 xβ2 · · · xβn
n
· ≤ ; β > 0,
n
1
elevando a la potencia β

q  β1 ! β1
n β β β xβ1 + xβ2
+···+ xβn
x1 · x2 · · · xn ≤
n
! β1
√ xβ1 + xβ2 + · · · + xβn
⇔ n
x1 · x2 · · · xn ≤ = Mβ ,
n

de donde MG ≤ Mβ .

∴ Mα ≤ MG ≤ Mβ .

2
Teorema 5.5
Si x1 , x2 , . . . , xn son números reales positivos y α < β, se tiene

Mα ≤ Mβ .

La igualdad ocurre si y sólo si x1 = x2 = · · · = xn .

 Prueba. El teorema 5.4 demuestra cuando α y β tienen diferentes signos. Ahora


veamos cuando tienen el mismo signo.
Supongamos que 0 < α < β y haciendo
  α1
xα1 + xα2 + · · · + xαn
Mα = = q,
n
74 5. Desigualdad de Bernoulli y la Media Potencial

entonces
 β  β  β  β1
x1 x2 xn
 + +···+ 
Mβ Mβ  q q q 
= =  .
Mα q  n 

Tomando
 α  α  α
x1 x2 xn
d1 = ; d2 = ; · · · ; dn = ,
q q q

obtenemos
 β β β
 β1
Mβ  d1 + d2 + · · · + dn 
α α α

= .
q n

Pero

 α  α  α  α1
x1 x2 xn
  α1
 q + +···+ 
d1 + d2 + · · · + dn q q
= 



n n

  α1
1 xα1 + xα2 + · · · + xαn 1
= · = · q = 1,
q n q

d1 + d2 + · · · + dn
resulta = 1, es decir
n
d1 + d2 + · · · + dn = n.

Haciendo

d1 = 1 + k1 ; d2 = 1 + k2 ; . . . ; dn = 1 + kn ,

entonces k1 + k2 + · · · + kn = 0.
β
Como > 1, pues α < β, se tiene
α
5.1. Media Potencial 75

β β β
α1α = (1 + k1 ) α ≥ 1+ k1
α
β β β
α2α = (1 + k2 ) α ≥ 1 + k2
α
.. ..
. .
β β β
αnα = (1 + kn ) α ≥ 1 + kn .
α
Sumando miembro a miembro, tenemos
β β β β
α1α + α2α + · · · + αnα ≥ n + (k1 + k2 + · · · + kn )
α | {z }
0
β β β
⇔ α1 + α2 + · · · + αn
α α α
≥ n
β β β
α α + α2α + · · · + αnα
⇔ 1 ≥ 1
n
 β β β
 β1
α + α2 + · · · + αn 
α α α

⇔  1 ≥ 1,
n

de donde


≥ 1 ⇔ Mβ ≥ q ⇔ Mβ ≥ Mα .
q
La igualdad Mβ = Mα se cumple si y sólo si

k1 = k2 = · · · = kn = 0, luego
d1 = d2 = · · · = dn = 1, y por consiguiente
x1 = x2 = · · · = xn .

β
Falta demostrar el caso α < β < 0, pero de α < β < 0 se tiene 0 < < 1, y se
α
repite el caso anterior.

∴ Mα ≤ Mβ .

2
76 5. Desigualdad de Bernoulli y la Media Potencial

5.2. Aplicaciones
1. Demuestre que si x3 + y 3 + z 3 = 2187, siendo x, y, z números reales positivos,
entonces

x + y + z ≤ 27.

 Prueba. Como M1 ≤ M3 , entonces:


 1/3
x+y+z x3 + y 3 + z 3

3 3
(x + y + z)3 x3 + y 3 + z 3
⇔ ≤
27 3
⇔ (x + y + z) ≤ 9(x + y 3 + z 3 )
3 3

⇔ (x + y + z)3 ≤ 32 · 37 = 39
⇔ x + y + z ≤ 33 .

Por lo tanto x + y + z ≤ 27.


La igualdad ocurre si y sólo si x = y = z = 9. 2

2. Si f (x, y, z) = x3 +y 3 +z 3 , con x, y, z números reales positivos y x+y+z = 3
3,
entonces el mı́nimo valor de f es:

Solución. Como x, y, z > 0 y tenemos que buscar una relación entre x+y+z
y x3 + y 3 + z 3 , entonces aplicamos la Media Potencial, veamos:
 1/3
x+y+z x3 + y 3 + z 3

3 3
(x + y + z)3 x3 + y 3 + z 3
⇔ ≤
27 3
√3 3
( 3)
⇔ ≤ x3 + y 3 + z 3
9
1
⇔ ≤ x3 + y 3 + z 3
3

3
1 3
Por lo tanto el mı́nimo de f es y ocurre si y sólo si x = y = z = . 2
3 3
5.2. Aplicaciones 77

3. Sean x, y, z reales no negativos, pruebe que

x5 + y 5 + z 5 ≥ 5(x + y + z) − 12.

 Prueba. Como en la desigualdad aparecen suma y suma de quintas,


entonces podemos aplicar la desigualdad de Bernoulli o la media potencial;
veamos utilizando la desigualdad de Bernoulli:

(1 + a)5 ≥ 1 + 5a; a ≥ −1.

Haciendo 1 + a = x ⇒ a = x − 1, entonces

x5 ≥ 1 + 5(x − 1)
⇔ x5 ≥ 5x − 4; x ≥ 0; similarmente
y 5 ≥ 5y − 4
z 5 ≥ 5z − 4

sumando miembro a miembro y efectuando tenemos

x5 + y 5 + z 5 ≥ 5x + 5y + 5z − 12.

∴ x5 + y 5 + z 5 ≥ 5(x + y + z) − 12.

La igualdad ocurre si y sólo si x = y = z = 1. 2


78 5. Desigualdad de Bernoulli y la Media Potencial
CAPÍTULO 6
DESIGUALDAD DE
REORDENAMIENTOS

Teorema 6.1 (Teorema de Abel)


Sean (x1 , x2 , . . . , xn ), (y1 , y2 , . . . , yn ), dos n-uplas de números reales y denotamos

ck = y1 + y2 + · · · + yk ; (k = 1, 2, . . . n),

entonces

x1 y1 + x2 y2 + · · · + xn yn = (x1 − x2 )c1 + (x2 − x3 )c2 + (xn−1 − xn )cn−1 + xn cn . (6.1)

 Prueba. Efectuando el segundo miembro de (6.1)

(x1 − x2 )c1 + (x2 − x3 )c2 + (xn−1 − xn )cn−1 + xn cn = c1 x1 + (c2 − c1 )x2 + · · ·


+(cn − cn−1 )xn
= y1 x1 + y2 x2 + · · · + yn xn .
2
Teorema 6.2 (Desigualdad de Abel)
Sean x1 , x2 , . . . , xn , y y1 ≥ y2 ≥ · · · ≥ yn ≥ 0 números reales y denotemos
Xk
sk = xi con M = máx {s1 , s2 , . . . , sn } y m = mı́n {s1 , s2 , . . . , sn }, entonces
i=1

my1 ≤ x1 y1 + x2 y2 + · · · + xn yn ≤ My1 .

79
80 6. Desigualdad de Reordenamientos

 Prueba. Como y1 ≥ y2 ≥ · · · ≥ yn ≥ 0, entonces aplicando el teorema de Abel


tenemos
n
X n
X
xi yi = (yi − yi+1 )si ; yn+1 = 0
i=1 i=1
Xn n
X
≥ (yi − yi+1 )m = m (yi − yi+1 ) = my1 ,
i=1 i=1

de donde
n
X
xi yi ≥ my1 .
i=1

Similarmente
n
X n
X
xi yi = (yi − yi+1 )si ; yn+1 = 0
i=1 i=1
n
X n
X
≤ (yi − yi+1 )M = M (yi − yi+1 ) = My1 ,
i=1 i=1

de donde
n
X
xi yi ≤ My1 .
i=1

n
X
∴ my1 ≤ xi yi ≤ My1 .
i=1

2
Aplicación 6.1
Sean a1 , a2 , · · · , an y b1 ≥ b2 ≥ · · · ≥ bn ≥ 0 números reales positivos tales que

a1 · a2 · · · ak ≥ b1 · b2 · · · bk , ∀ k ∈ {1, 2, . . . , n}.

Pruebe que

a1 + a2 + · · · + an ≥ b1 + b2 + · · · + bn .
81

 Prueba. Aplicando el teorema de Abel

n
X n
X n
X  
ai
ai − bi = bi −1
i=1 i=1 i=1
b i
   
a1 a1 a2
= (b1 − b2 ) − 1 + (b2 − b3 ) + −2 +···
b1 b1 b2
n−1
! n
!
X ai X ai
+(bn−1 − bn ) − n + 1 + bn −n .
b
i=1 i
b
i=1 i

k
X ai
Analizando cada suma ; k = 1, 2, . . . , n, mediante la propiedad MA ≥ MG
i=1
bi
r
a1 a2 ak a1 a2 ak
+ +···+ ≥k k
· ··· ≥ k,
b1 b2 bk b1 b2 bk

luego
k
!
X ai
−k ≥ 0.
i=1
bi

n
X n
X
∴ ai − bi ≥ 0.
i=1 i=1

2
Teorema 6.3 (Desigualdad de Reordenamientos)
Sean (a1 , a2 , . . . , an ) y (b1 , b2 , . . . , bn ) dos sucesiones crecientes de números reales y
sea (bi1 , bi2 , . . . , bin ) una permutación de (b1 , b2 , . . . , bn ), entonces

a1 b1 + a2 b2 + · · · + an bn ≥ a1 bi1 + a2 bi2 + · · · + an bin .

Si (a1 , a2 , . . . , an ) es creciente y (b1 , b2 , . . . , bn ) es decreciente, entonces

a1 b1 + a2 b2 + · · · + an bn ≤ a1 bi1 + a2 bi2 + · · · + an bin .

 Prueba. Veamos el caso

a1 ≤ a2 ≤ · · · ≤ an y b1 ≤ b2 ≤ · · · ≤ bn .
82 6. Desigualdad de Reordenamientos

Aplicando el teorema de Abel

n
X n
X n
X
ak bk − ak bik = ak (bk − bik )
k=1 i=1 k=1

= (a1 − a2 ) (b1 − bi1 ) + (a2 − a3 ) (b1 + b2 − bi1 − bi2 ) + · · ·


| {z } | {z }
(+) (+)
n−1 n−1
!
X X
+ (an−1 − an ) bk − bik
k=1 k=1
| {z }
(+)
n n
!
X X
+ an bk − bik ≥ 0,
k=1 k=1
| {z }
(+)

pues para cada k = 1, 2, . . . , n, tenemos que


k
X k
X
bj ≤ bij ,
j=1 j=1

ya que b1 ≤ b2 ≤ · · · ≤ bn .
La segunda parte del teorema se prueba similarmente. 2

Aplicación 6.2
Pruebe que
a b c 3
+ + ≥
b+c c+a a+b 2
para todo a, b, c números reales positivos.

 Prueba. Esta desigualdad ya fue probada por el Lema de Titu y aplicando


MA ≥ MG, ahora veamos aplicando reordenamientos.
En efecto, por la simetrı́a del primer miembro de la desigualdad, podemos asumir
un orden, como a ≤ b ≤ c, entonces a + b ≤ a + c ≤ b + c y
1 1 1
≤ ≤ .
b+c a+c a+b
83

Luego

a b c b c a
+ + ≥ + +
b+c c+a a+b b+c c+a a+b
a b c c a b
+ + ≥ + + ,
b+c c+a a+b b+c c+a a+b
sumando miembro a miembro, tenemos:
 
a b c b+c c+a a+b
2 + + ≥ + + = 3,
b+c c+a a+b b+c c+a a+b

de donde

a b c 3
+ + ≥ .
b+c c+a a+b 2
2

Colorario 6.1 Para toda permutación (a′1 , a′2 , . . . , a′n ) de (a1 , a2 , . . . , an ), se tiene

a21 + a22 + · · · + a2n ≥ a1 · a′1 + a2 · a′2 + · · · + an · a′n .

Colorario 6.2 Para toda permutación (a′1 , a′2 , . . . , a′n ) de (a1 , a2 , . . . , an ), se tiene

a′1 a′2 a′n


+ +···+ ≥ n.
a1 a2 an

Teorema 6.4 (Desigualdad de Chebyshev)


Sean a1 ≤ a2 ≤ · · · ≤ an y b1 ≤ b2 ≤ · · · ≤ bn , entonces

  
a1 b1 + a2 b2 + · · · + an bn a1 + a2 + · · · + an b1 + b2 + · · · + bn
≥ .
n n n

Si una de las sucesiones es creciente y la otra, decreciente, el sentido de la de-


sigualdad cambia

  
a1 b1 + a2 b2 + · · · + an bn a1 + a2 + · · · + an b1 + b2 + · · · + bn
≤ .
n n n
84 6. Desigualdad de Reordenamientos

 Prueba. Veamos el caso cuando las dos sucesiones son crecientes. En efecto,
por la desigualdad de reordenamientos, tenemos

a1 b1 + a2 b2 + · · · + an bn = a1 b1 + a2 b2 + · · · + an bn ,
a1 b1 + a2 b2 + · · · + an bn ≥ a1 b2 + a2 b3 + · · · + an b1 ,
a1 b1 + a2 b2 + · · · + an bn ≥ a1 b3 + a2 b4 + · · · + an b2 ,
.. .. ..
. . .
a1 b1 + a2 b2 + · · · + an bn ≥ a1 bn + a2 b1 + · · · + an bn−1 .

Sumando miembro a miembro, tenemos

n(a1 b1 + a2 b2 + · · · + an bn ) ≥ (a1 + a2 + · · · + an )(b1 + b2 + · · · + bn ).

El otro caso se prueba de manera similar. 2


Aplicación 6.3
Pruebe que

a3 b3 c3 (a + b + c)3
+ + ≥
x y z 3(x + y + z)

para todo a, b, c, x, y, z números reales positivos con a ≥ b ≥ c y z ≥ y ≥ x.

 Prueba. Sean las ternas


 
a2 b2 c2
, , ; (a, b, c),
x y z

a2 b2 c2
como a ≥ b ≥ c y z ≥ y ≥ x, entonces ≥ ≥ .
x y z
Aplicando el teorema de Chebyshev
 2 
a2 b2 c2 a b2 c2
·a+ ·b+ ·c  
x y z  x + y + z  a+b+c

≥  
3 3  3
  
a3 b3 c3 a2 b2 c2 a+b+c
⇔ + + ≥ + + .
x y z x y z 3
85

Luego es suficiente demostrar


 2  
a b2 c2 a+b+c (a + b + c)3
+ + ≥ .
x y z 3 3(x + y + z)

Pero sabemos por el lema de Titu

a2 b2 c2 (a + b + c)2
+ + ≥ .
x y z x+y+z
 
a+b+c
Multiplicando por , tenemos
3
 2  
a b2 c2 a+b+c (a + b + c)3
+ + ≥ .
x y z 3 3(x + y + z)
2
Aplicación 6.4
Pruebe que
xα yα zα 3
+ + ≥
y+z z+x x+y 2
para todo α, x, y, z números reales positivos tales que xyz = 1 y α ≥ 1.

 Prueba. Por la simetrı́a, podemos asumir el orden x ≥ y ≥ z, entonces


1 1
z+x≥y+z ⇔ ≤
z+x y+z
1 1
x+y ≥x+z ⇔ ≤ ,
x+y x+z
1 1 1
de donde ≤ ≤ .
x+y x+z y+z
z y x
Luego ≤ ≤ y además
x+y x+z y+z
xα−1 ≥ y α−1 ≥ z α−1 .

Utilizando Chebyshev

 
xα yα zα 1 α−1  x y z
+ + ≥ x + y α−1 + z α−1 + + .
y+z z+x x+y 3 y+z z+x x+y
86 6. Desigualdad de Reordenamientos

Pero sabemos que

xα−1 + y α−1 + z α−1 p


≥ 3 (xyz)α−1 = 1
3
x y z 3
+ + ≥
y+z z+x x+y 2

xα yα zα 3
∴ + + ≥ .
y+z z+x x+y 2
2
Aplicación 6.5
Pruebe que

a3 b3 c3 d3 1
+ + + ≥
b+c+d a+c+d a+b+d a+b+c 3
para todo a, b, c, d reales positivos con ab + bc + cd + da = 1.

 Prueba. Por la simetrı́a del primer miembro de la desigualdad, supongamos


sin pérdida de generalidad que a ≥ b ≥ c ≥ d, y haciendo

m = b + c + d,
n = a + c + d,
p = a + b + d,
q = a+b+c
1 1 1 1
⇒ m≤n≤p≤q ∧ ≥ ≥ ≥ ,
m n p q

luego, aplicando Chebyshev, tenemos:

a3 b3 c3 d3
+ + +
m n p q
 
1 3 3 3 3 1 1 1 1
≥ (a + b + c + d ) + + +
4 m n p q
 
1 2 2 2 2 1 1 1 1
≥ (a + b + c + d ) (a + b + c + d) + + +
16 m n p q
87

como m + n + p + q = 3(a + b + c + d), entonces

a3 b3 c3 d3
+ + +
m n p q
  
1 2 m + n + p + q 1 1 1 1
≥ (a + b2 + c2 + d2 ) + + +
16 3 m n p q
  
1 2 2 2 2 1 1 1 1 1
= (a + b + c + d ) (m + n + p + q) + + +
16 3 m n p q
1 2 1
≥ (a + b2 + c2 + d2 ) (16)
16 3
1 2
= (a + b2 + c2 + d2 ) .
3

Nos falta acotar a2 + b2 + c2 + d2 y como tenemos la relación ab + bc + cd + da = 1,


entonces tomamos las cuaternas (a, b, c, d) y (b, c, d, a) para aplicar Cauchy-Schwarz;
en efecto:

(a2 + b2 + c2 + d2 ) (b2 + c2 + d2 + a2 ) ≥ (ab + bc + cd + da)2


2
⇔ (a2 + b2 + c2 + d2 ) ≥ (1)2
⇔ a2 + b2 + c2 + d2 ≥ 1.

a3 b3 c3 d3 1
∴ + + + ≥ .
m n p q 3
2
88 6. Desigualdad de Reordenamientos
CAPÍTULO 7
DESIGUALDAD CON FUNCIONES
CONVEXAS

Las funciones convexas cumplen un rol muy importante en la matemática, espe-


cialmente en la lı́nea de optimización, ya que en estos tiempos se están estudiando
modelos matemáticos en ingenierı́a, economı́a, etc.
En este capı́tulo estudiaremos estas funciones para obtener una desigualdad muy
importante llamada la desigualdad de Jensen, la cual se ha utilizado en muchas
olimpiadas matemáticas internacionales.

7.1. Función convexa


Definición 7.1: Sea f una función real de variable real, definida sobre [a, b] ⊂ R. f
es llamada una función convexa sobre [a, b] si y sólo si para cada x, y ∈ [a, b] y para
todo 0 ≤ t ≤ 1, se tiene

f (tx + (1 − t)y) ≤ tf (x) + (1 − t)f (y).

A continuación utilizaremos el concepto de primera y segunda derivada (se puede


cualquier libro de Análisis Matemático).

Teorema 7.1
Si f es una función real definida sobre [a, b] ⊂ R y f ′′ (x) > 0 para todo x ∈ ha, bi,
entonces f es una función convexa sobre [a, b].

89
90 7. Desigualdad con Funciones Convexas

 Prueba. Debemos probar que para todo x ∈ [a, b] y para todo t ∈ [0, 1], se
cumple

f (tx + (1 − t)y) ≤ tf (x) + (1 − t)f (y). (7.1)

Supongamos que t y y son constantes; definimos

g(x) = tf (x) + (1 − t)f (y) − f (tx + (1 − t)y).

Derivando con respecto a x

g ′ (x) = tf ′ (x) − tf ′ (tx + (1 − t)y) = t(f ′ (x) − f ′ (tx + (1 − t)y)).

Como f ′′ (x) > 0, ∀ x ∈ [a, b], entonces f ′ es creciente en [a, b], de donde tenemos
que

g ′(x) ≥ 0 si x ≥ y; y

g ′(x) ≤ 0 si x ≤ y,

el mı́nimo de la función es g(y), evaluando en (7.1) tenemos g(y) = 0.


Luego

g(x) ≥ g(y), ∀ x ∈ [a; b],

de donde

g(x) ≥ 0 ∀ x ∈ [a; b].

Reemplazando

tf (x) + (1 − t)f (y) − f (tx + (1 − t)y) ≥ 0.

∴ f (tx + (1 − t)y) ≤ tf (x) + (1 − t)f (y), ∀ x ∈ [a; b], ∀ t ∈ [0; 1]. [9]

2
7.1. Función convexa 91

Ejemplo 7.1
La función f (x) = ǫx , con x número real, es convexa.

−4 −3 −2 −1 1 2 3 4
−1

pues f ′′ (x) = ǫx > 0, para todo x número real.


Teorema 7.2
Si f es convexa sobre [a, b], entonces para cada x, y ∈ [a, b] se tiene
 
x+y 1
f ≤ (f (x) + f (y)) .
2 2

1
 Prueba. Es suficiente tomar t = en la definición. 2
2
Definición 7.2 : Una función f real definida sobre [a, b] ⊂ R es llamada función
cóncava sobre [a, b] si y sólo si para cada x, y ∈ [a, b] y para todo 0 ≤ t ≤ 1 se tiene

f (tx + (1 − t)y) ≥ tf (x) + (1 − t)f (y).

Teorema 7.3 (Desigualdad de Jensen) n


X
Si f es convexa sobre [a, b], entonces para cada t1 , t2 , . . . , tn ∈ [0, 1] con ti = 1 y
i=1
para cada x1 , x2 , . . . , xn ∈ [a, b], se tiene

f (t1 x1 + t2 x2 + · · · + tn xn ) ≤ t1 f (x1 ) + t2 f (x2 ) + · · · + tn f (xn ).


92 7. Desigualdad con Funciones Convexas

 Prueba. Demostraremos por inducción. Para n = 2 es el teorema anterior.


Supongamos que se cumpla para (n − 1), veamos para n.
Como

h i
t1 t2 tn
t1 x1 + t2 x2 + · · · + tn−1 xn−1 + tn xn = (1 − tn ) x
1−tn 1
+ x
1−tn 2
+···+ x
1−tn−1 n−1

+tn xn ,

entonces

   
t1 tn−1
f (t1 x1 + t2 x2 + · · · + tn xn ) = f (1 − tn ) x1 + · · · + xn−1 + tn xn
1 − tn 1 − tn
 
t1 tn−1
≤ (1 − tn )f x1 + · · · + xn−1
1 − tn 1 − tn
+tn f (xn ), f convexa
 
t1 tn−1
≤ (1 − tn ) f (x1 ) + · · · + f (xn−1 )
1 − tn 1 − tn
+tn f (xn ),
= t1 f (x1 ) + t2 f (x2 ) + · · · + tn f (xn ),

de donde se tiene

f (t1 x1 + t2 x2 + · · · + tn xn ) ≤ t1 f (x1 ) + t2 f (x2 ) + · · · + tn f (xn ).

2
1
En particular, si t1 = t2 = · · · = tn = , se tiene
n
 
x1 + x2 + · · · + xn 1
f ≤ (f (x1 ) + f (x2 ) + · · · + f (xn )) .
n n
Similarmente si f es cóncava entonces

f (t1 x1 + t2 x2 + · · · + tn xn ) ≥ t1 f (x1 ) + t2 f (x2 ) + · · · + tn f (xn ).

Teorema 7.4
Si f es una función real definida sobre [a, b] ⊂ R, y f ′′ (x) < 0 para todo x ∈ ha, bi,
entonces f es una función cóncava sobre [a, b].
7.1. Función convexa 93

 Prueba. La prueba es similar al teorema 7.1. 2


Aplicación 7.1 n
X
Pruebe que dados x1 , x2 , . . . , xn , t1 , t2 , . . . , tn números reales positivos con ti = 1,
i=1
entonces

xt11 · xt22 · · · xtnn ≤ t1 x1 + t2 x2 + · · · + tn xn .

 Prueba. Sabemos que f (x) = ǫx , x ∈ R, es convexa y además


ti
xtii = ǫln xi = ǫti ln xi ,

entonces

xt11 · xt22 · · · xtnn = ǫt1 ln x1 · ǫt2 ln x2 · · · ǫtn ln xn


= ǫt1 ln x1 +t2 ln x2 +···+tn ln xn

aplicando la Desigualdad de Jensen

ǫt1 ln x1 +t2 ln x2 +···+tn ln xn ≤ t1 ǫln x1 + t2 ǫln x2 + · · · + tn ǫln xn


= t1 x1 + t2 x2 + · · · + tn xn .
2
Aplicación 7.2
1 1
Sean x, y, a, b números reales positivos, que satisfacen + = 1, entonces
a b
1 1
xy ≤ xa + y b.
a b

 Prueba. De la aplicación anterior se tiene


1  1b 1 1
xy = (xa ) a · y b ≤ xa + y b .
a b
2
Aplicación 7.3
Si x1 , x2 , . . . , xn son números reales positivos. Pruebe que
1 1 1 n
+ +···+ ≥ √ .
1 + r1 1 + r2 1 + rn n
r1 r2 · · · rn + 1
94 7. Desigualdad con Funciones Convexas

 Prueba. Por la forma de cada fracción del primer miembro, definimos la función
1
f (x) = ; x ∈ R+ .
1 + ǫx
Veamos que es convexa.
Derivando, tenemos
−ǫx
f ′ (x) = y
(1 + ǫx )2
′′ ǫx (ǫx − 1)
f (x) = > 0, ∀ x > 0.
(1 + ǫx )3
Por lo tanto f es convexa. Luego, podemos aplicar la Desigualdad de Jensen

 
x1 + x2 + · · · + xn 1
f ≤ (f (x1 ) + f (x2 ) + · · · + f (xn ))
n n
 
1 1 1 1 1
x1 +x2 +···+xn ≤ + +···+
1+ǫ n n 1 + ǫx1 1 + ǫx2 1 + ǫxn

haciendo ǫxi = ri , i = 1, 2, . . . , n, tenemos


n 1 1 1
√ ≤ + +···+ .
n
r1 r2 · · · rn + 1 1 + r1 1 + r2 1 + rn
2
Aplicación 7.4
Utilizando la desigualdad de Jensen, pruebe que
x1 + x2 + · · · + xn √
≥ n x1 x2 · · · xn ,
n
siendo x1 , x2 , · · · , xn números reales positivos.

 Prueba. Definamos f (x) = ln x, x > 0.


Derivando
1
f ′ (x) = ,
x
1
f ′′ (x) = − < 0,
x2
7.1. Función convexa 95

entonces f es cóncava. Luego


 
1 x1 + x2 + · · · + xn
(ln x1 + ln x2 + · · · + ln xn ) ≤ ln
n n
 
1 x1 + x2 + · · · + xn
⇔ ln (x1 x2 · · · xn ) ≤ ln
n n
 
√ x1 + x 2 + · · · + xn
⇔ ln n x1 x2 · · · xn ≤ ln
n
√ x1 + x2 + · · · + xn
⇔ n x1 x2 · · · xn ≤ .
n
2
Aplicación 7.5
(IMO 2001). Sean a, b, c números reales positivos, pruebe que
a b c
√ +√ +√ ≥ 1.
a2 + 8bc b2 + 8ca c2 + 8ab

1
 Prueba. Definimos la función f (x) = √ , x > 0. La función es convexa, y sin
x
pérdida de generalidad asumimos que a + b + c = 1, luego

af (a2 + 8bc) + bf (b2 + 8ca) + cf (c2 + 8ab)


 

≥ f a(a2 + 8bc) + b(b2 + 8ca) + c(c2 + 8ab) ,


| {z }
x

luego basta demostrar que


1
f (x) = √ ≥ 1
x
⇔ x≤1
⇔ a3 + b3 + c3 + 24abc ≤ 1
⇔ a3 + b3 + c3 + 24abc ≤ (a + b + c)3
⇔ a3 + b3 + c3 + 24abc ≤ a3 + b3 + c3 + 3(a + b)(b + c)(c + a)
⇔ (a + b)(b + c)(c + a) ≥ 8abc.

La igualdad ocurre si y sólo si a = b = c. 2


96 7. Desigualdad con Funciones Convexas
CAPÍTULO 8
ESPACIO MÉTRICO

Una de las operaciones principales del análisis es el paso al lı́mite. Esta operación
descansa sobre el hecho de que en la recta numérica está definida la distancia entre
dos puntos.
Es impresionante ver que muchos resultados principales del análisis no tienen
nada que ver con la naturaleza algebraica del conjunto de los números reales, es
decir, sólo se apoya en las propiedades de distancia y con ello llegamos al concepto
de Espacio Métrico, que es uno de los conceptos más importantes de la matemática
moderna.

Definición 8.1 : Un espacio métrico es un par (X, d) compuesto de un conjunto


(espacio) X 6= ∅ de elementos (puntos) y de una distancia, es decir una función

d : X × X −→ R
(x, y) 7−→ d(x, y)

no negativa, que verifica las tres condiciones siguientes:

(1) d(x, y) = 0 ⇔ x = y.

(2) d(x, y) = d(y, x), (axioma de simetrı́a.)

(3) d(x, z) ≤ d(x, y) + d(y, z), (axioma triangular.)

Muchas veces lo denotaremos simplemente como X al espacio métrico (X, d),


por comodidad en la notación.

97
98 8. Espacio Métrico

Mencionaremos algunos ejemplos que desempeñan un papel importante en el


análisis.

1. Sea X 6= ∅ un conjunto arbitrario con


(
0, x = y
d(x, y) =
6 y
1, x =

(X, d) es un espacio métrico llamado de puntos aislados.

2. El conjunto de los números reales con la distancia

d(x, y) = |x − y|

forma el espacio métrico (R, d).


Veamos que cumple los axiomas correspondientes.

 Prueba.

(1)
d(x, y) = 0 ⇔ |x − y| = 0
⇔ x−y =0
⇔ x = y.

(2) d(x, y) = |x − y| = |y − x| = d(x, y).


(3) d(x, z) = |x − z| = |(x − y) + (y − z)| ≤ |x − y| + |y − z| = d(x, y) + d(y, z)
de donde

d(x, z) ≤ d(x, y) + d(y, z).

3. El conjunto

Rn = {x = (x1 , x2 , . . . , xn )/ x1 , x2 , . . . , xn ∈ R}
99

con la distancia
v
u n
uX
d(x, y) = t (yk − xk )2
k=1

es el espacio métrico (Rn , d).


Para verificar que es espacio métrico, veamos que se cumple el axioma trian-
gular (3), ya que (1) y (2) son directos.
Sean

x = (x1 , x2 , . . . , xn ), y = (y1 , y2 , . . . , yn ), z = (z1 , z2 , . . . , zn ),

entonces para que se cumpla

d(x, z) ≤ d(x, y) + d(y, z)

se tiene que cumplir la desigualdad


v v v
u n u n u n
uX uX uX
t (zk − xk )2 ≤ t (yk − xk )2 + t (zk − yk )2 . (8.1)
k=1 k=1 k=1

Si tomamos yk − xk = ak y zk − yk = bk ⇒ zk − xk = ak + bk , entonces (8.1)


tiene la forma
v v v
u n u n u n
uX uX uX
t (ak + bk )2 ≤ t ak + t
2
b2k . (8.2)
k=1 k=1 k=1

Esta desigualdad se deduce de la desigualdad de Cauchy-Schwarz

n
!2 n n
X X X
ak bk ≤ a2k · b2k . (8.3)
k=1 k=1 k=1
100 8. Espacio Métrico

En efecto:

n
X n
X n
X n
X
2
(ak + bk ) = a2k +2 ak bk + b2k
k=1 k=1 k=1 k=1
v
n u n n n
X uX X X
≤ 2
ak + 2 t 2
ak · 2
bk + b2k
k=1 k=1 k=1 k=1
v v 2
u n u n
uX uX
= t a2k + t b2k  ,
k=1 k=1

de donde
v v v
u n u n u n
uX uX uX
t (ak + bk )2 ≤ t a2k + t b2k .
k=1 k=1 k=1

4. Considerando el mismo conjunto Rn , pero definiendo la distancia

n
X
d1 (x, y) = |yk − xk |.
k=1

También es un espacio métrico (Rn , d1 ).

5. Considerando nuevamente Rn , y definiendo una nueva distancia

d2 (x, y) = máx |yk − xk |.


1≤k≤n

También es un espacio métrico (Rn , d2 ).

6. L2 = {x = (x1 , x2 , . . . , xn , . . .)} tal que xk ∈ R para todo k ∈ N tal que


X∞
x2k < ∞ y con distancia
k=1
v
u∞
uX
d(x, y) = t (yk − xk )2 .
k=1
101

La función d(x, y) ası́ definida tiene sentido para x, y ∈ L2 ya que la serie


v
u∞ ∞ ∞
uX X X
t (yk − xk )2 converge siempre que x2k < ∞ y yk2 < ∞, esto ocurre
k=1 k=1 k=1
2
pues (xk ± yk ) ≤ 2(x2k + yk2 ).
Al mismo tiempo vemos que si (x1 , x2 , . . . , xn , . . .), (y1 , y2, . . . , yn , . . .) ∈ L2 ,
también (x1 + y1 , x2 + y2 , . . . , xn + yn , . . .) ∈ L2 .

7. El conjunto Rn , con la distancia

n
!1/p
X
dp (x, y) = |yk − xk |p ,
k=1

donde p es un número fijo arbitrario mayor que 1 (p > 1), representa el espacio
métrico (Rn , dp ). Para comprobar que es espacio métrico, veamos que cumple
los tres axiomas.
(1) y (2) son obvias, veamos el axioma triangular (3), es decir, se debe cumplir

n
!1/p n
!1/p n
!1/p
X X X
|zk − xk |p ≤ |zk − yk |p + |yk − xk |p .
k=1 k=1 k=1

Haciendo

zk − yk = ak y yk − xk = bk ⇒ zk − xk = ak + bk ,

tenemos

n
!1/p n
!1/p n
!1/p
X X X
|ak + bk |p ≤ |ak |p + |bk |p .
k=1 k=1 k=1

A continuación estudiaremos esta desigualdad, llamada desigualdad de Minkon-


ski; previamente veremos la desigualdad de Hölder.
102 8. Espacio Métrico

8.1. Desigualdad de Hölder


1
Sean x1 , x2 , . . . , xn , y1 , y2 , . . . , yn números reales positivos y a, b > 0 tal que +
a
1
= 1, entonces
b
n n
!1/a n
!1/b
X X X
xi yi ≤ xai · yib .
i=1 i=1 i=1

n
X n
X
 Prueba. Supongamos que xai = yib = 1, usando el hecho que
i=1 i=1
1 1
xi yi ≤ xai + yib (ejercicio demostrado anteriormente),
a b

entonces
n
X n n
1X a 1X b 1 1
xi yi ≤ x + y = + = 1. (8.4)
i=1
a i=1 i b i=1 i a b

n
X n
X
Ahora supongamos que xai =M y yib = N.
i=1 i=1
xi yi
Sea x′i = 1/a , yi′ = 1/b , entonces
M N
Xn

n
xai
X a i=1 M
x′ i = = = 1,
i=1
M M
n
X
n
yib
X b i=1 N
y′i = = = 1.
i=1
N N

Luego
n
X n
X Xn
′ ′ xi yi 1
x iy i = = 1/a xi yi. (8.5)
i=1 i=1
M 1/a · N 1/b M · N 1/b i=1
8.2. Desigualdad de Minkonski 103

n
X
De (8.4) se tiene que x′i yi′ ≤ 1, entonces de (8.5) tenemos
i=1
X n
1
xi yi ≤ 1
M 1/a · N 1/b i=1
n
X
xi yi ≤ M 1/a · N 1/b .
i=1

Reemplazando

n n
!1/a n
!1/b
X X X
xi yi ≤ xai · yib .
i=1 i=1 i=1

En particular si a = b = 2, tenemos la desigualdad de Cauchy-Schwarz.

8.2. Desigualdad de Minkonski


Sean a1 , a2 , . . . , an , b1 , b2 , . . . , bn números positivos y p > 1, entonces

n
!1/p n
!1/p n
!1/p
X X X
(ak + bk )p ≤ (ak )p + (bk )p .
k=1 k=1 k=1

 Prueba. Como

(ak + bk )p = (ak + bk )(ak + bk )p−1


= ak (ak + bk )p−1 + bk (ak + bk )p−1 ,

entonces
n
X n
X n
X
(ak + bk )p = ak (ak + bk )p−1 + bk (ak + bk )p−1 , (8.6)
k=1 k=1 k=1

aplicando la desigualdad de Hölder en cada término del segundo miembro, sa-


1 1
biendo que p > 1 y + = 1
p q
104 8. Espacio Métrico

n n
!1/p n
!1/q
X X X
ak (ak + bk )p−1 ≤ (ak )p · (ak + bk )q(p−1) ,
k=1 k=1 k=1
n n
!1/p n
!1/q
X X X
bk (ak + bk )p−1 ≤ (bk )p · (ak + bk )q(p−1) ,
k=1 k=1 k=1

además q(p − 1) = p y reemplazando en (8.6)

n n
!1/p n
!1/q
X X X
(ak + bk )p ≤ (ak )p · (ak + bk )p
k=1 k=1 k=1
n
!1/p n
!1/q
X X
+ (bk )p · (ak + bk )p
k=1 k=1

n n
!1/q  n !1/p n
!1/p 
X X X X
(ak + bk )p ≤ (ak + bk )p · (ak )p + (bk )p 
k=1 k=1 k=1 k=1

n
!1−1/q n
!1/p n
!1/p
X X X
(ak + bk )p ≤ (ak )p + (bk )p
k=1 k=1 k=1
n
!1/p n
!1/p n
!1/p
X X X
(ak + bk )p ≤ (ak )p + (bk )p .
k=1 k=1 k=1

2
Teorema 8.1
Dados los puntos A, B, C del plano, se tiene que

AB ≤ AC + CB,

la igualdad ocurre si C ∈ AB.


Aplicación 8.1
Sean a, b, c los lados de un triángulo. Pruebe que
a b c
+ + < 2.
b+c c+a a+b
8.2. Desigualdad de Minkonski 105

 Prueba. Como a < b + c, b < c + a, c < a + b,

a b c 2a 2b 2c
+ + = + +
b+c c+a a+b 2b + 2c 2c + 2a 2a + 2b
2a 2b 2c
= + +
(b + c) +(b + c) (c + a) +(c + a) (a + b) +(a + b)
| {z } | {z } | {z }
2a 2b 2c
< + +
a+b+c a+b+c a+b+c
2(a + b + c)
= = 2,
a+b+c

de donde

a b c
+ + < 2.
b+c c+a a+b
2
Aplicación 8.2
Pruebe que
√ p p √
2|x + y + z| ≤ x2 + y 2 + y 2 + z 2 + x2 + z 2 ,

para todo x, y, z números reales.

 Prueba. Escribiendo adecuadamente el punto

(x + y + z, x + y + z) = (x, y) + (y, z) + (z, x).

Pero

d(x + y + z, x + y + z) = d((x, y) + (y, z) + (z, x))


≤ d(x, y) + d(y, z) + d(z, x),

de donde
√ p p √
2|x + y + z| ≤ x2 + y 2 + y 2 + z 2 + x2 + z 2 .

2
106 8. Espacio Métrico

Aplicación 8.3
Pruebe que

x4/3 y 4/3
M = +
x4/3 + (x2 + y 2)1/3 (x + z)2/3 y 4/3 + (y 2 + z 2 )1/3 (y + z)2/3
z 4/3
+ ≤1
z 4/3 + (z 2 + x2 )1/3 (x + y)2/3

para todo x, y, z números reales positivos.

 Prueba. Haciendo x = a3 , y = b3 , z = c3 , por la desigualdad de Hölder tenemos

1/3 1/3
(x2 + y 2 ) (x + z)2/3 = ((a2 )3 + (b2 )3 ) ((c2 )3/2 + (a2 )3/2 )2/3
≥ a2 c2 + b2 a2 = (xy)2/3 + (xz)2/3
1/3
⇒ x4/3 + (x2 + y 2 ) (x + z)2/3 ≥ x4/3 + (xy)2/3 + (xz)2/3
x4/3 x4/3
⇒ ≤
x4/3 + (x2 + y 2 )1/3 (x + z)2/3 x4/3 + (xy)2/3 + (xz)2/3

x2/3
= 2/3
x + y 2/3 + z 2/3

similarmente

y 4/3 y 2/3
≤ ,
y 4/3 + (y 2 + z 2 )1/3 (y + z)2/3 x2/3 + y 2/3 + z 2/3

z 4/3 z 2/3
≤ ,
z 4/3 + (z 2 + x2 )1/3 (z + x)2/3 x2/3 + y 2/3 + z 2/3

sumando miembro a miembro las tres desigualdades, en el segundo miembro se


obtiene el valor de 1, de donde

M ≤ 1.

2
8.2. Desigualdad de Minkonski 107

Aplicación 8.4
Sean a, b, c, d números reales. Determine el mı́nimo de

p p
F = (a + 1)2 + 2(b − 2)2 + (c + 3)2 + (b + 1)2 + 2(c − 2)2 + (d + 3)2
p p
+ (c + 1)2 + 2(d − 2)2 + (a + 3)2 + (d + 1)2 + 2(a − 2)2 + (b + 3)2 .

 Prueba. Sea S = a + b + c + d y aplicando la desigualdad de Minkowski:

p p
S1 = (a + 1)2 + 2(b − 2)2 + (c + 3)2 + (b + 1)2 + 2(c − 2)2 + (d + 3)2
p
≥ (a + 1 + b + 1)2 + 2(b − 2 + c − 2)2 + (c + 3 + d + 3)2
p
= (a + b + 2)2 + 2(b + c − 4)2 + (c + d + 6)2
p p
S2 = (c + 1)2 + 2(d − 2)2 + (a + 3)2 + (d + 1)2 + 2(a − 2)2 + (b + 3)2
p
≥ (c + d + 2)2 + 2(a + d − 4)2 + (a + b + 6)2
p
S1 + S2 ≥ (a + b + 2)2 + 2(b + c − 4)2 + (c + d + 6)2
p
+ (c + d + 2)2 + 2(a + d − 4)2 + (a + b + 6)2
(aplicando nuevamente Minkowski)
p
F ≥ (a + b + c + d + 4)2 + 2(a + b + c + d − 8)2 + (a + b + c + d + 12)2
p
= (s + 4)2 + 2(s − 8)2 + (s + 12)2

Efectuando dentro del radical



F ≥ s2 + 8s + 16 + 2s2 − 32s + 128 + s2 + 24s + 144
√ √ √
F ≥ 4s2 + 288 ≥ 288 = 12 2

luego el mı́nimo valor de F es 12 2 y esto ocurre cuando a = b = c = d = 0. 2
108 8. Espacio Métrico
CAPÍTULO 9
MÉTODO DE LA SUMA DE
CUADRADOS

Definición 9.1 : El polinomio f (a1 , a2 , . . . , an ) es homogéneo si todos sus términos


tienen el mismo grado absoluto. Mencionaremos algunos ejemplos:

1. f (a1 , a2 ) = a21 + a22 + 5a1 a2 , es homogéneo de segundo grado.


2. f (a1 , a2 , a3 ) = a31 + a22 a3 + a1 a2 a3 , es homogéneo de tercer grado.

Sea el polinomio homogéneo f (a1 , a2 , . . . , an ), entonces f (a1 , a2 , . . . , an ) ≥ 0,


para todo a1 , a2 , . . . , an reales, significa que la función puede expresarse como una
suma f (a1 , a2 , . . . , an ) = p21 + p22 + · · · + p2n , donde pk es una función real.
Veamos la desigualdad

A ≥ B,

donde A, B son expresiones de variables a, b, c para explicar la idea central.


Si queremos demostrar A ≥ B, entonces veremos la diferencia

A − B = f (a, b, c) = Sa (b − c)2 + Sb (c − a)2 + Sc (a − b)2 . (9.1)

Si Sa , Sb , Sc ≥ 0, entonces

f (a, b, c) = Sa (b − c)2 + Sb (c − a)2 + Sc (a − b)2 ≥ 0;

esto significa que A ≥ B es una desigualdad verdadera.

109
110 9. Método de la Suma de Cuadrados

Teorema 9.1
Si a, b, c, Sa , Sb , Sc satisfacen las condiciones

1. Sa + Sb ≥ 0, Sb + Sc ≥ 0, Sc + Sa ≥ 0, y

2. (a ≤ b ≤ c ∨ a ≥ b ≥ c) y Sb ≥ 0,

entonces

Sa (b − c)2 + Sb (c − a)2 + Sc (a − b)2 ≥ 0 (9.2)

 Prueba. Sin pérdida de generalidad supongamos que a ≤ b ≤ c, luego

Sa (b − c)2 + Sb (c − a)2 + Sc (a − b)2


= Sa (b − c)2 + Sb (c − b + b − a)2 + Sc (a − b)2
= Sa (b − c)2 + Sb [(c − b)2 + (b − a)2 + 2(c − b)(b − a)] + Sc (a − b)2
= Sa (b − c)2 + Sb (c − b)2 + Sb (b − a)2 + 2(c − b)(b − a)Sb + Sc (a − b)2
= (Sa + Sb )(b − c)2 + (Sb + Sc )(a − b)2 + 2 (c − b)(b − a) Sb ≥ 0.
| {z } | {z } | {z } |{z}
≥0 ≥0 ≥0 ≥0

La igualdad ocurre si y sólo si

(Sa + Sb ) (b − c)2 = 0 ∧ (Sb + Sc ) (a − b)2 = 0 ∧ 2(c − b)(b − a)Sb = 0,

de donde se deduce que

(a = b = c) ∨ (a = b ∧ Sa = Sb = 0) ∨ (b = c ∧ Sb = Sc = 0)
∨ (Sa = Sb = Sc = 0).

2
Teorema 9.2
Si a, b, c, Sa , Sb , Sc son números reales que satisfacen las condiciones:
(a ≤ b ≤ c ∨ a ≥ b ≥ c), Sa ≥ 0, Sc ≥ 0, Sa + 2Sb ≥ 0, Sc + 2Sb ≥ 0, entonces

Sa (b − c)2 + Sb (c − a)2 + Sc (a − b)2 ≥ 0 (9.3)


111

 Prueba. Sin pérdida de generalidad supongamos que a ≤ b ≤ c, entonces


Sa ≥ 0, Sc ≥ 0, Sa + 2Sb ≥ 0, Sc + 2Sb ≥ 0.

1. Si Sb ≥ 0, entonces (9.3) siempre es verdadero.


2. Si Sb < 0, entonces

Sa (b − c)2 + Sb (c − a)2 + Sc (a − b)2


= Sa (b − c)2 + Sb (c − b + b − a)2 + Sc (a − b)2
= (Sb + Sa ) (c − b)2 + (Sb + Sc ) (b − a)2 + 2Sb (c − b)(b − a)
≥ (Sb + Sa ) (c − b)2 + (Sb + Sc ) (b − a)2 + Sb [(c − b)2 + (b − a)2 ]
= (Sa + 2Sb ) (c − b)2 + (Sc + 2Sb ) (b − a)2 ≥ 0.

Analice cuándo se da la igualdad. 2


Teorema 9.3
Si a, b, c, Sa , Sb , Sc son números reales que satisfacen las condiciones:
(a ≤ b ≤ c ∨ a ≥ b ≥ c), Sa ≥ 0, Sb ≥ 0 y b2 Sc + c2 Sb ≥ 0, entonces

Sa (b − c)2 + Sb (c − a)2 + Sc (a − b)2 ≥ 0 (9.4)

 Prueba. Sin pérdida de generalidad supongamos que a ≤ b ≤ c, y de Sa ≥ 0,


Sb ≥ 0 y b2 Sc + c2 Sb ≥ 0, entonces

Sa (b − c)2 + Sb (c − a)2 + Sc (a − b)2


"  2 #
c−a
= Sa (b − c)2 + (b − a)2 Sb + Sc ;
b−a
 2  c 2
c−a
como ≥ , entonces
b−a b
   
2 2 2 2 2 c 2
Sa (b − c) + Sb (c − a) + Sc (a − b) ≥ Sa (b − c) + (b − a) Sb + Sc
b
 
2
 2 2
 b−a 2
= Sa (b − c) + Sb c + Sc b ≥ 0.
| {z } | {z } b
≥0 ≥0

Analice cuando se da la igualdad. 2


112 9. Método de la Suma de Cuadrados

Teorema 9.4
Si a, b, c, Sa , Sb , Sc son números reales que satisfacen las condiciones

1. Sa + Sb ≥ 0 ∨ S a + Sc ≥ 0 ∨ Sb + Sc ≥ 0, y

2. Sa Sb + Sb Sc + Sc Sa ≥ 0,

entonces

Sa (b − c)2 + Sb (c − a)2 + Sc (a − b)2 ≥ 0 (9.5)

 Prueba. Sin pérdida de generalidad supongamos que Sb + Sc ≥ 0.


Haciendo u = b − a, v = c − b, tenemos

Sa (b − c)2 + Sb (c − a)2 + Sc (a − b)2


= Sa (b − c)2 + Sb (c − b + b − a)2 + Sc (a − b)2
= Sa (b − c)2 + Sb (c − b)2 + Sb (b − a)2 + +2Sb (c − b)(b − a) + Sc (a − b)2
= (Sa + Sb )(c − b)2 + (Sb + Sc )(b − a)2 + 2Sb (c − b)(b − a)
= (Sa + Sb )v 2 + (Sb + Sc )u2 + 2Sb u · v
 
 2
Sb  Sa Sb + Sb Sc + Sc Sa  2
= (Sb + Sc ) u + v + 
 v ≥ 0.

Sb + Sc Sb + Sc
| {z }
≥0

2
Aplicación 9.1
Pruebe que
   
a b c b c a 1 1 1
3 mı́n + + , + + ≥ (a + b + c) + + ,
b c a a b c a b c
√ √ √
siendo a, b, clos lados de un triángulo.

 Prueba. Sin pérdida de generalidad supongamos que


 
a b c b c a a b c
mı́n + + , + + = + + ,
b c a a b c b c a
113

entonces la desigualdad que vamos a demostrar es equivalente a


   
a b c 1 1 1
3 + + ≥ (a + b + c) + +
b c a a b c

⇔ 3(a2 c + b2 a + c2 b) ≥ (a + b + c)(ab + ac + bc)

⇔ 3(a2 c + b2 a + c2 b) ≥ a2 b + a2 c + ab2 + b2 c + ac2 + bc2 + 3abc

⇔ 2(a2 c + b2 a + c2 b) ≥ a2 b + b2 c + ac2 + 3abc

multiplicando por 2 miembro a miembro

4a2 c + 4b2 a + 4c2 b ≥ 2a2 b + 2b2 c + 2ac2 + 6abc


2 2 2 2 2 2
⇔ a
| c − 2abc
{z + b }c + |b a − 2abc
{z + c a} + |c b − 2abc
{z + a }b
+3a2 c + 3b2 a + 3c2 b − 3a2 b − 3b2 c − 3c2 a ≥ 0
 
⇔ c(a − b)2 + a(b − c)2 + b(c − a)2 + 3 a2 c + b2 a + c2 b − a2 b − b2 c − c2 a ≥ 0
⇔ c(a − b)2 + a(b − c)2 + b(c − a)2 + 3(a − b)(b − c)(c − a) ≥ 0
| {z }
⇔ c(a − b) + a(b − c) + b(c − a) + (a − b) + (b − c) + (c − a)3 ≥ 0
2 2 2 3 3

(pues (a − b) + (b − c) + (c − a) = 0)
⇔ (a − b)2 (c| + {z
a − }b) + (b − c)2 (a b − }c) + (c − a)2 (b| + {z
| +{z c − a}) ≥ 0
Sc Sa Sb
2 2 2
⇔ Sa (b − c) + Sb (c − a) + Sc (a − b) ≥ 0 (9.6)

y como

Sa + Sb = 2b > 0,

Sa + Sc = 2a > 0,

Sb + Sc = 2c > 0

entonces para que se cumpla (9.6) por el teorema 9.4 basta probar que

Sa Sb + Sa Sc + Sb Sc ≥ 0.
114 9. Método de la Suma de Cuadrados

Reemplazando, tenemos la desigualdad equivalente

(a + b − c)(b + c − a) + (b + c − a)(c + a − b) + (c + a − b)(a + b − c)


= (b + a − c)(b + c − a) + (c + b − a)(c + a − b) + (a + c − b)(a + b − c)
= b2 − (c − a)2 + c2 − (a − b)2 + a2 − (b − c)2
= 2ab + 2ac + 2bc − (a2 + b2 + c2 )
= 4ab − (a2 + b2 + c2 + 2ab − 2ac − 2ac − 2bc)
√ √
= 4ab − (a + b − c)2 = (2 ab + a + b − c)(2 ab − a − b + c)
  
√ √ 2 √ 2 √ 2 √ √ 2
= a + b − ( c) ( c) − a− b
   
√ √ √  √ √ √ √ √ √ √ √ √
= a + b + c  a + b − c  c + a − b  c + b − a > 0,
| {z } | {z } | {z }
+ + +
√ √ √
(pues a, b, c son lados de un triángulo)

entonces Sa Sb + Sb Sc + Sc Sa > 0; luego aplicando teorema 9.4, concluimos que

(c + a − b)(a − b)2 + (a + b − c)(b − c)2 + (b + c − a)(c − a)2 ≥ 0.

La igualdad ocurre si y sólo si el triángulo es equilátero. 2

Aplicación 9.2
Sean a, b, c reales positivos tales que ab + bc + ca = 1. Pruebe que

1 + a2 b2 1 + b2 c2 1 + c2 a2 5
2
+ 2
+ 2
≥ .
(a + b) (b + c) (c + a) 2

 Prueba. Veamos

1 + a2 b2 12 + a2 b2 (ab + bc + ca)2 + a2 b2
= =
(a + b)2 (a + b)2 (a + b)2
2a2 b2 + 2ab(bc + ca) + (bc + ca)2
=
(a + b)2
2ab(ab + ac + bc) + (bc + ca)2 2ab(ab + ac + bc)
= 2
= + c2 ,
(a + b) (a + b)2
115

entonces

1 + a2 b2 2ab(ab + ac + bc)
2
= + c2 ; análogamente
(a + b) (a + b)2
1 + b2 c2 2bc(ab + ac + bc)
2
= + a2 ;
(b + c) (b + c)2
1 + c2 a2 2ca(ab + ac + bc)
2
= + b2 ;
(c + a) (c + a)2

sumando miembro a miembro, tenemos

 
1 + a2 b2 1 + b2 c2 1 + c2 a2 2ab 2bc 2ac
+ + = (ab + ac + bc) + +
(a + b)2 (b + c)2 (c + a)2 (a + b)2 (b + c)2 (a + c)2
+a2 + b2 + c2 ,

luego basta demostrar

 
2ab 2bc 2ac 5
(ab + ac + bc) 2
+ 2
+ 2
+ a2 + b2 + c2 ≥
(a + b) (b + c) (a + c) 2
 
4ab 4bc 4ac
⇔ (ab + ac + bc) 2
+ 2
+ + 2(a2 + b2 + c2 )
(a + b) (b + c) (a + c)2
≥ 5 = 5(ab + bc + ac)
 
4ab 4bc 4ac
⇔ (ab + ac + bc) + + −3
(a + b)2 (b + c)2 (a + c)2
+2(a2 + b2 + c2 − ab − ac − bc) ≥ 0
     
4ab 4bc 4ac
⇔ (ab + ac + bc) −1 + −1 + −1
(a + b)2 (b + c)2 (a + c)2
+2(a2 + b2 + c2 − ab − ac − bc) ≥ 0
116 9. Método de la Suma de Cuadrados

 
(a − b)2 (b − c)2 (a − c)2
⇔ −(ab + ac + bc) + +
(a + b)2 (b + c)2 (a + c)2
+(a − b)2 + (b − c)2 + (c − a)2 ≥ 0
   
(ab + ac + bc) 2 (ab + ac + bc)
⇔ 1− (a − b) + 1 − (b − c)2
(a + b)2 (b + c)2
 
(ab + ac + bc)
+ 1− (c − a)2 ≥ 0.
(a + c)2

Haciendo

ab + ac + bc ab + ac + bc ab + ac + bc
Sa = 1 − ; S b = 1 − ; S c = 1 − .
(b + c)2 (a + c)2 (a + b)2

Supongamos sin pérdida de generalidad a ≤ b ≤ c entonces Sa ≥ Sb ≥ Sc , luego


tenemos

ab + ac + bc a2 + (a + c)(c − b)
Sb = 1 − = ≥ 0 ⇒ Sa ≥ Sb ≥ 0 y
(c + a)2 (a + c)2

   
2 2 2 ab + bc + ca 2 ab + bc + ca
b Sc + c Sb = b 1 − +c 1−
(a + b)2 (c + a)2
 2   2 
2 a + (a + b)(b − c) 2 a + (a + c)(c − b)
= b +c
(a + b)2 (c + a)2
 2 2
  2 2

b c c b
= a2 + + (c − b) −
(a + b)2 (c + a)2 (a + c) (a + b)
   
2 b2 c2 2 ab + bc + ca
= a + + (c − b) > 0,
(a + b)2 (c + a)2 (a + c)(a + b)

luego por el teorema 9.3 queda demostrado.√


3
La igualdad ocurre si y sólo si a = b = c = . [10] 2
3
CAPÍTULO 10
DESIGUALDADES SIMÉTRICAS Y
CÍCLICAS

Definición 10.1 : Sean a1 , a2 , . . . , an números reales, los coeficientes c0 , c1 , . . . , cn


del polinomio

n
X
H(x) = (x + a1 )(x + a2 ) · · · (x + an ) = ck xn−k
k=0

se denominan funciones polinomiales simétricas elementales de a1 , a2 , . . . , an , es


decir,

c0 = 1
n
X
c1 = a1 + a2 + · · · + an = ai
i=1
X
c2 = a1 a2 + a1 a3 + · · · + an−1 an = ai aj
1≤i<j≤n

..
.

cn = a1 a2 · · · an .

117
118 10. Desigualdades Simétricas y Cı́clicas

Ejemplo 10.1
Si H(x) = (x + a)(x + b)(x + c) = x3 + (a + b + c)x2 + (ab + ac + bc)x + abc, entonces

c0 = 1
c1 = a + b + c
c2 = ab + ac + bc
c3 = abc.

En base a estos polinomios simétricos elementales, daremos una definición.

Definición 10.2: Dados los polinomios simétricos elementales c0 , c1 , . . . , cn , defini-


mos

1 k!(n − k)!
Pk = 
n ck = ck ; k = 0, 1, 2, . . . , n.
k
n!

Ejemplo 10.2

P 0 = c0 = 1
1 a1 + a2 + · · · + an
P1 = c1 =
n n
2 2
P2 = c2 = (a1 a2 + a1 a3 + · · · + an−1 an ).
n(n − 1) n(n − 1)

Teorema 10.1 (Desigualdad de Newton)


Sean a1 , a2 , . . . , an números reales positivos y k ∈ {1, 2, . . . , (n − 1)} entonces, se
cumple que

Pk−1 · Pk+1 ≤ Pk2 .

La igualdad ocurre si y sólo si a1 = a2 = · · · = an .


Nota 10.1
Antes de la demostración del teorema, veamos un resultado muy importante que
usaremos en la demostración de la desigualdad de Newton.
Sea

n
X Xn  
n−k n
H(x) = (x + a1 )(x + a2 ) · · · (x + an ) = ck x = Pk xn−k .
k=0 k=0
k
119

Derivando tenemos:
n−1
X  
n

H (x) = (n − k) Pk xn−k−1 .
k=0
k

Definimos

n−1
X   n−1 
X 
1 ′ n−k n n−k−1 n−1
Q(x) = H (x) = Pk x = Pk xn−k−1 .
n k=0
n k k=0
k

Si los valores reales ak , k = 1, 2, . . . , n son elementos del intervalo [α, β], entonces
el polinomio H(x) tiene n raı́ces reales en [α, β] y por el teorema de Rolle (consúltese
cualquier libro de análisis matemático) entonces H ′(x) tiene (n − 1) raı́ces reales en
[α, β] y denotemos como −y1 , −y2 , . . . , −yn−1 ; luego
1 ′
Q(x) = H (x) = (x + y1 )(x + y1 ) · · · (x + yn−1 ).
n
Igualando coeficientes de Q(x), obtenemos:

Pk (a1 , a2 , . . . , an ) = Pk (y1, y2 , . . . , yn−1)

para todo k = 0, 1, 2, . . . , (n − 1).

 Prueba. (de la desigualdad de Newton.) Consideremos la hipótesis inductiva


Tn :

Pj−1 (a1 , a2 , . . . , an ) · Pj+1 (a1 , a2 , . . . , an ) ≤ Pj2 (a1 , a2 , . . . , an ),

donde j = 2, 3, . . . , (n − 2).
Para j = 1 y n = 2, tenemos:
 2
a1 + a2
P0 · P2 ≤ P12 ⇔ 1 · a1 a2 ≤
2
⇔ 4a1 a2 ≤ a21 + a22 + 2a1 a2
⇔ (a1 − a2 )2 ≥ 0

como vemos se cumple para n = 2.


120 10. Desigualdades Simétricas y Cı́clicas

Para j = 1 y n = 3.

P0 (a1 , a2 , a3 ) · P2 (a1 , a2 , a3 ) ≤ P12 (a1 , a2 , a3 )


 2
2 a1 + a2 + a3
⇔ (1) · (a1 a2 + a1 a3 + a2 a3 ) ≤
3(2) 3
⇔ 3(a1 a2 + a1 a3 + a2 a3 ) ≤ a21 + a22 + a23 + 2(a1 a2 + a1 a3 + a2 a3 )
⇔ a21 + a22 + a23 ≥ a1 a2 + a1 a3 + a2 a3

ésta es una desigualdad verdadera.


Para j = 2 y n = 3.

P1 (a1 , a2 , a3 ) · P3 (a1 , a2 , a3 ) ≤ P22 (a1 , a2 , a3 )


   2
a1 + a2 + a3 a1 a2 + a1 a3 + a2 a3
⇔ · (a1 a2 a3 ) ≤
3 3

Si a1 a2 a3 = 0, se cumple trivialmente.
Si a1 a2 a3 6= 0, entonces dividiendo por (a1 a2 a3 )2 , tenemos
   2
1 1 1 1 1 1 1 1
+ + ≤ + +
3 a1 a2 a2 a3 a3 a1 9 a1 a2 a3
   2
1 1 1 1 1 1
⇔ 3 + + ≤ + +
a1 a2 a2 a3 a3 a1 a1 a2 a3
 2  2  2
1 1 1 1 1 1
⇔ + + ≥ + + .
a1 a2 a3 a1 a2 a1 a3 a2 a3

Ahora veamos para j = n − 1, es decir probaremos que


2
Pn−2 (a1 , a2 , . . . , an ) · Pn (a1 , a2 , . . . , an ) ≤ Pn−1 (a1 , a2 , . . . , an )
" #
2 X
⇔ (a1 · · · abj · · · abk · · · an ) a1 a2 · · · an
n(n − 1) 1≤j<k≤n

n
!2
1X
≤ a1 a2 · · · abj · · · an
n j=1

donde los sı́mbolos abj y abk se omiten.


Si a1 a2 · · · an = 0, se verifica la desigualdad.
121

Si a1 a2 · · · an 6= 0, entonces dividiendo por (a1 a2 · · · an )2 , se tiene


n
!2
1 X 1 1X 1
n
 ≤
2
aj ak n j=1 aj
1≤j<k≤n

y esta desigualdad es equivalente a:


     
1 1 1 1 1 1 1 1 1
P0 , ,..., · P2 , ,..., ≤ P12 , ,..., . (10.1)
a1 a2 an a1 a2 an a1 a2 an

Teniendo en cuenta la nota anterior, la desigualdad (10.1) es verdadera y con


esto se completa la prueba. 2
Teorema 10.2 (Desigualdad de Mac Laurin)
Si a1 , a2 , . . . , an son números reales positivos, entonces

P1 ≥ (P2 )1/2 ≥ · · · ≥ (Pk )1/k ≥ · · · ≥ (Pn )1/n .

La igualdad ocurre si y sólo si a1 = a2 = · · · = an .


 Prueba. Como a1 , a2 , . . . , an > 0, entonces podemos aplicar la desigualdad de
Newton para 1 ≤ k < n, en efecto:

k−1
Y 2(k−1)
Pkk−1 · k
Pk+1 Pj2j = P0 · P12 · P24 · P36 · · · Pk−1 · Pkk−1 · Pk+1
k

j=0

= (P0 · P2 )(P1 · P3 )2 (P2 · P4 )3 · · · (Pk−1 · Pk+1 )2k


≤ P12 · P24 · P36 · · · Pk2k
k
Y
= Pj2j
j=1
k−1
Y k−1
Y
⇒ Pkk−1 · k
Pk+1 Pj2j ≤ Pj2j · Pk2k .
j=0 j=1

Pkk−1 · k
Pk+1 ≤ Pk2k
k
⇔ Pk+1 ≤ Pkk+1
1/k+1 1/k
⇔ Pk+1 ≤ Pk .
La igualdad ocurre si y sólo si se cumple a1 = a2 = · · · = an . 2
122 10. Desigualdades Simétricas y Cı́clicas

Aplicación 10.1
Sean a, b, c, d números reales positivos, pruebe que

27
(ab + ac + ad + bc + bd + cd)3 ≥ (abc + abd + acd + bcd)2 .
2

 Prueba. Utilizando la desigualdad de Mac Laurin:

1/2 1/3
P2 ≥ P3 ⇔ P23 ≥ P32

!3 !2
1 1
⇔ 4 (ab + ac + ad + bc + bd + cd)
 ≥ 4 (abc + abd + acd + bcd)

2 3
1 1
⇔ 3
(ab + ac + ad + bc + bd + cd)3 ≥ 2 (abc + abd + acd + bcd)2
6 4
27
⇔ (ab + ac + ad + bc + bd + cd)3 ≥ (abc + abd + acd + bcd)2 .
2

Aplicación 10.2
Dados a, b, c, d números reales positivos, pruebe que
   2
1 1 1 1 1 1 3 1 1 1 1
+ + + + + ≤ + + + .
ab ac ad bc bd cd 8 a b c d

 Prueba. Multiplicando por (abcd)2 tenemos la desigualdad equivalente:

   2
1 2 1 1 1 1 1 3 2 1 1 1 1
(abcd) + + + + + ≤ (abcd) + + +
ab ac ad bc bd cd 8 a b c d
   2
cd + bd + bc + ad + ac + ab bcd + acd + abd + abc
⇔ abcd ≤
6 4
⇔ P4 · P2 ≤ (P3 )2 ,

esta desigualdad es verdadera, pues es la desigualdad de Newton. 2


10.1. Polinomios simétricos y cı́clicos 123

10.1. Polinomios simétricos y cı́clicos


Para mayor comprensión consideremos un polinomio P (x, y, z) de variables x, y, z;
luego, para definir introducimos dos notaciones muy importantes como:
X X
y .
cı́c sim

Definición 10.3 :
X
P (x, y, z) = P (x, y, z) + P (y, z, x) + P (z, x, y)
cı́c
X
P (x, y, z) = P (x, y, z) + P (x, z, y) + P (y, z, x)
sim
+P (y, x, z) + P (z, x, y) + P (z, y, x).

Ejemplo 10.3
X
x2 y = x2 y + y 2z + z 2 x
cı́c
X
x2 y = x2 y + x2 z + y 2 x + y 2 z + z 2 x + z 2 y
sim
X
x3 = x3 + y 3 + z 3
cı́c
X
x3 = 2 (x3 + y 3 + z 3 )
sim
X
xyz = xyz + yzx + zxy = 3xyz
cı́c
X
xyz = xyz + xzy + yxz + yzx + zxy + zyx = 6xyz.
sim

A continuación enunciaremos el teorema de Muirhead para tres variables para


mayor entendimiento.
124 10. Desigualdades Simétricas y Cı́clicas

Teorema 10.3 (Teorema de Muirhead)


Sean a1 , a2 , a3 , b1 , b2 , b3 números reales no negativos tales que

a1 ≥ a2 ≥ a3 , b1 ≥ b2 ≥ b3 , a1 ≥ b1 , a1 + a2 ≥ b1 + b2 , a1 + a2 + a3 = b1 + b2 + b3 .

Sean x, y, z números reales no negativos, entonces


X X
xa1 y a2 z a3 ≥ xb1 y b2 z b3 .
sim sim

 Prueba. De las hipótesis se observa que hay dos casos para analizar:
Caso 1:

b1 ≥ a2 ⇔ a1 + b1 ≥ a1 + a2
⇔ a1 ≥ a1 + a2 − b1

y de a1 ≥ b1 se tiene que a1 ≥ máx {a1 + a2 − b1 , b1 }.


Además máx {a1 , a2 } = a1 ≥ máx {a1 + a2 − b1 , b1 }; también de a1 + a2 − b1 ≥
b1 + a3 − b1 = a3 y a1 + a2 − b1 ≥ b2 ≥ b3 , tenemos máx {a1 + a2 − b1 , a3 } ≥
máx {b2 , b3 }.
En efecto
X X
xa1 y a2 z a3 = (xa1 y a2 z a3 + xa2 y a1 z a3 )
sim cı́c
X
= z a3 (xa1 y a2 + xa2 y a1 )
cı́c
X 
≥ z a3 xa1 +a2 −b1 y b1 + xb1 y a1 +a2 −b1 ,
cı́c
pues

xa1 y a2 + xa2 y a1 ≥ xa1 +a2 −b1 y b1 + xb1 y a1 +a2 −b1


⇔ xa1 y a2 + xa2 y a1 − xa1 +a2 −b1 y b1 − xb1 y a1 +a2 −b1 ≥ 0

⇔ xa2 y a2 xa1 −a2 + y a1 −a2 − xa1 −b1 y b1−a2 − xb1 −a2 y a1 −b1 ≥ 0
 
⇔ xa2 y a2 xb1 −a2 − y b1 −a2 xa1 −b1 − y a1 −b1 ≥ 0
10.1. Polinomios simétricos y cı́clicos 125

por que para cualquier orden x ≥ y o y ≥ x, el producto siempre es no negativo.


Luego

X X 
xa1 y a2 z a3 ≥ z a3 xa1 +a2 −b1 y b1 + xb1 y a1 +a2 −b1
sim cı́c
X 
= xb1 y a1 +a2 −b1 z a3 + xb1 y a3 z a1 +a2 −b1
cı́c
X 
= xb1 y a1 +a2 −b1 z a3 + y a3 z a1 +a2 −b1
cı́c
X 
≥ xb1 y b2 z b3 + y b3 z b2
cı́c
X 
= xb1 y b2 z b3 + xb1 y b3 z b2
cı́c
X
= xb1 y b2 z b3
sim

Caso 2: Si b1 ≥ a2

⇒ 3b1 ≥ b1 + b2 + b3 = a1 + a2 + a3 ≥ b1 + a2 + a3
⇒ 3b1 ≥ b1 + a2 + a3
⇒ 2b1 ≥ a2 + a3
⇒ b1 ≥ a2 + a3 − b1

además a1 ≥ a2 ≥ b1 ⇒ a1 ≥ a2 + a3 − b1 .
Luego tenemos que máx {a2 , a3 } ≥ máx {b1 , a2 + a3 − b1 } y máx {a1 , a2 + a3 − b1 } ≥
máx {b2 , b3 }.
En efecto:

X X
xa1 y a2 z a3 = (xa1 y a2 z a3 + xa1 z a2 y a3 )
sim cı́c
X
= xa1 (y a2 z a3 + z a2 y a3 )
cı́c
126 10. Desigualdades Simétricas y Cı́clicas

X 
≥ xa1 y b1 z a2 +a3 −b1 + y a2 +a3 −b1 z b1
cı́c
X 
= xa1 y b1 z a2 +a3 −b1 + xa1 y a2 +a3 −b1 z b1
cı́c
X 
= xa1 y b1 z a2 +a3 −b1 + xa2 +a3 −b1 y b1 z a1
cı́c
X 
= y b1 xa1 z a2 +a3 −b1 + xa2 +a3 −b1 z a1
cı́c
X 
≥ y b1 xb2 z b3 + xb3 z b2
cı́c
X
= xb1 y b2 z b3
sim
2
Aplicación 10.3
Pruebe para todo a, b, c números reales positivos la desigualdad
a b c 3
+ + ≥ .
b+c a+c a+b 2
 Prueba. La desigualdad a demostrar es equivalente a

2 [a(a + c)(a + b) + b(b + c)(a + b) + c(b + c)(a + c)] ≥ 3(b + c)(a + c)(a + b),

efectuando

2 [a(a2 + (b + c)a + bc) + b(b2 + (a + c)b + ac) + c(c2 + (a + b)c + ab)]


≥ 3 [(b + c)(a2 + (b + c)a + bc)]
⇔ 2(a3 + b3 + c3 ) + 2 (a2 (b + c) + b2 (a + c) + c2 (a + b)) + 6abc
≥ 3 [a2 (b + c) + b2 (a + c) + c2 (a + b) + 2abc]
⇔ 2(a3 + b3 + c3 ) ≥ a2 (b + c) + b2 (a + c) + c2 (a + b)
X X
⇔ a3 ≥ a2 b.
sim sim
2
10.1. Polinomios simétricos y cı́clicos 127

Aplicación 10.4
(IMO 1995). Sean a, b, c números reales positivos tales que abc = 1, pruebe que
1 1 1 3
+ 3 + 3 ≥ .
a3 (b + c) b (c + a) c (a + b) 2

 Prueba. Como los denominadores del primer miembro son de grado 4 y abc = 1,
entonces la desigualdad es equivalente a
1 1 1 3
+ 3 + 3 ≥ ,
a3 (b + c) b (c + a) c (a + b) 2(abc)4/3

es conveniente eliminar el exponente 4/3, para ello hacemos

a = x3 , b = y 3 , c = z 3 , con x, y, z > 0,

luego tenemos:

1 1 1 3 3x5 y 5 z 5
+ + ≥ =
x9 (y 3 + z 3 ) y 9(z 3 + x3 ) z 9 (x3 + y 3) 2(xyz)4 2(x9 y 9 z 9 )
⇔ 2 [y 9z 9 (z 3 + x3 )(x3 + y 3 ) + x9 z 9 (y 3 + z 3 )(x3 + y 3) + x9 z 9 (y 3 + z 3 )(z 3 + x3 )]
≥ 3 [x5 y 5z 5 (y 3 + z 3 )(z 3 + x3 )(x3 + y 3 )] ,

efectuando

X X X X
x12 y 12 + 2 x12 y 9 z 3 + x9 y 9 z 6 ≥ 3 x11 y 8z 5 + 6x8 y 8 z 8
sim sim sim sim
   
X X X X
⇔  x12 y 12 − x11 y 8 z 5  + 2  x12 y 9z 3 − x11 y 8z 5 
sim sim sim sim
 
X X
+ x9 y 9 z 6 − x8 y 8 z 8  ≥ 0.
sim sim

Para enunciar en forma general el teorema de Muirhead, es necesaria la siguiente


definición.
128 10. Desigualdades Simétricas y Cı́clicas

Definición 10.4 : Sean α = (α1 , α2 , . . . , αn ), β = (β1 , β2 , . . . , βn ) elementos en Rn ,


de componentes no negativas; diremos que α > β si y sólo si

(i) α1 ≥ α2 ≥ · · · ≥ αn y β1 ≥ β2 ≥ · · · ≥ βn .

(ii) α1 + α2 + · · · + αk ≥ β1 + β2 + · · · + βk ; k = 1, 2, . . . , (n − 1).

(iii) α1 + α2 + · · · + αn = β1 + β2 + · · · + βn .

Teorema 10.4 (Generalizado de Muirhead)


Si α > β y x1 , x2 , . . . , xn números reales no negativos, entonces
X X β
xα1 1 · xα2 2 · · · xαnn ≥ x1 1 · xβ2 2 · · · xβnn .
sim sim
La igualdad ocurre si y sólo si α = β o x1 = x2 = · · · = xn .
Aplicación 10.5
Sean x1 , x2 , x3 , x4 números reales positivos, pruebe que

3 3 √ √ √
(x1 + x32 + x33 + x34 ) ≥ x1 x4 x2 x3 + x2 x4 x1 x3 + x3 x4 x1 x2
2
√ √ √
+x2 x3 x1 x4 + x1 x2 x3 x4 + x1 x3 x2 x4 .
X X
 Prueba. Como x31 = x31 · (x02 x03 x04 ), entonces tenemos 6 permutaciones
sim √sim
para (x2 , x3 , x4 ) y para x1 x2 x3 x4 , tenemos 2 permutaciones para x1 x2 y 2 per-

mutaciones para x3 x4 , en total 4; luego, la desigualdad se puede escribir equiva-
lentemente como

√ √ √
6(x31 + x32 + x33 + x34 ) ≥ 4[x1 x4 x2 x3 + x2 x4 x1 x3 + x3 x4 x1 x2
√ √ √
+x2 x3 x1 x4 + x1 x2 x3 x4 + x1 x3 x2 x4 ].
X X √
⇔ x31 ≥ x1 x2 x3 x4 ,
sim sim

como α = (3, 0, 0, 0) y β = (1, 1, 1/2, 1/2) entonces esta última desigualdad es


verdadera.
La igualdad ocurre si y sólo si x1 = x2 = x3 = x4 . 2
CAPÍTULO 11
PROBLEMAS

1. Sean x, y números reales positivos tales que a y b son las medias aritmética y
geométrica respectivamente, pruebe que

(1 + b)2 ≤ (1 + x)(1 + y) ≤ (1 + a)2 .

2. Pruebe que

    2
1 1 2
1+ 1+ ≥ 1+
x y x+y

para todo x, y números reales positivos.

3. Sean x, y, z números reales positivos, pruebe que al menos una de las siguientes
desigualdades es verdadera:

xy ≥ 0
yz ≥ 0
zx ≥ 0.

4. Si a2 + b2 = 1, halle la variación de 2a + 3b.

5. Sabiendo que 2a + 3b + 4c = 29, halle el menor valor de a2 + b2 + c2 .

129
130 11. Problemas

6. Pruebe que
p √ p
m2 + y 2 + n2 + x2 ≥ (m + n)2 + (x + y)2

para todo x, y, m, n números reales.


7. Si a, b, c son números reales positivos que satisfacen abc = 1, pruebe que

1 + ab 1 + bc 1 + ac
+ + ≥ 3.
1+a 1+b 1+c

8. Sean a, b, c números reales positivos, pruebe que

a b c 3
+ + ≥ .
b+c c+a a+b 2

(Desigualdad de Nesbit).
9. Pruebe que si x, y, z son números reales positivos y x + y + z = 1, entonces
   
1 1 1
1+ 1+ 1+ ≥ 64.
x y z

10. Sean a, b, c, d números reales positivos tales que a2 + b2 = (c2 + d2 )3 , pruebe


que

c3 d 3
+ ≥ 1.
a b

11. Sean a, b, c números reales positivos, pruebe que

a b c
+ + ≥ 1.
b + 2c c + 2a a + 2b

12. Pruebe que

x2 y2
+ ≥ 8,
y−1 x−1

para todo x, y > 1.


131

13. Sean a, b, c números reales positivos tales que a + b + c = 2, pruebe que

a+b b+c c+a 9


+ + ≥ .
2a + bc 2b + ac 2c + ab 4

14. Si a, b son números reales tales que a2 + b2 = 1, pruebe que



2 2
a b + ab2 ≤ .
2

15. Sean x, y números reales positivos, pruebe que


√ √
x y
√ +√ ≥ 1.
x + 3y y + 3x

16. Sean a, b, c números reales positivos tales que ab + bc + ca ≤ 3abc, pruebe que

a + b + c ≤ a3 + b3 + c3 .

17. Sean a, b, c números reales positivos tales que a2 + b2 + c2 = 3, pruebe que

1 1 1
+ + ≥ 1.
1 + 2ab 1 + 2bc 1 + 2ca

18. Sean a, b, c números reales positivos tales que a2 + b2 + c2 = 3, pruebe que

1 1 1 3
+ + ≥ .
1 + ab 1 + bc 1 + ca 2

19. Pruebe que



(a + b + c)4 ≥ 8 a2 + b2 + c2 (ab + ac + bc).

para todo a, b, c números reales.


20. Dados a, b, c números reales positivos con abc = 8, pruebe que

1 1 1
√ +√ +√ ≥ 1.
1+a3 1+b3 1 + c3
132 11. Problemas

21. Sean a, b números reales positivos, pruebe que

1 1 1
2
+ 2
≥ .
(1 + a) (1 + b) 1 + ab

22. Sean a, b, c, d números reales positivos tales que abcd = 1, pruebe que

1 1 1 1
2
+ 2
+ 2
+ ≥ 1.
(1 + a) (1 + b) (1 + c) (1 + d)2

23. Sean a, b, c números reales positivos, pruebe que

2 (a3 + b3 + c3 ) + (3a + 1) (b2 + c2 ) + (3b + 1) (a2 + c2 ) + (3c + 1) (a2 + b2 )


≥ 2(ab + ac + bc) + 3(a + b)(a + c)(b + c)

24. (IMO 1995). Sean a, b, c números reales positivos tales que abc = 1, pruebe
que

1 1 1 3
+ 3 + 3 ≥ .
a3 (b + c) b (c + a) c (a + b) 2

25. Demuestre que


     
2 1 1 2 1 1 2 1 1
a + +b + +c + ≥ 2(a + b + c)
b c a c a b

para todo a, b, c números reales positivos.

26. Sean x, y, z números reales que pertenecen al intervalo [−1, 1] tales que x +
y + z = 0, pruebe que
p p √
1 + x + y 2 + 1 + y + z 2 + 1 + z + x2 ≥ 3.

27. Sean a, b, c números reales, pruebe que



a+b+c+ a2 + b2 + c2 − ab − bc − ca ≤ 3 máx {a, b, c}.
133

28. Dados x, y, z números reales positivos tales que x + y + z = 1, pruebe que


r
x y z 3
√ +√ +√ ≥ .
y+z z+x x+y 2

29. (IMO 1964). Sean a, b, c los lados de un triángulo, pruebe que

a2 (b + c − a) + b2 (a + c − b) + c2 (a + b − c) ≤ 3abc.

30. Dados a, b números reales tales que a2 + b2 = 1, pruebe que



3 3
ab + máx {a; b} ≤ .
4
31. Dados a, b, c, d números reales no negativos, pruebe que

a b c d
+ 2 + 2 + 2 ≥ 2.
b2 2
+c +d 2 2
c +d +a2 2
d +a +b2 a + b2 + c2

32. Sean a, b, c, d números reales no negativos tales que a2 +b2 +c2 +d2 = 4, pruebe
que

a3 + b3 + c3 + d3 ≤ 8.

33. Sean a, b, c los lados de un triángulo, pruebe que

a b c
+ + ≥ 1.
3a − b + c 3b − c + a 3c − a + b

34. Pruebe que si a, b, c son números reales positivos tales que abc = 1,entonces

1 1 1 1 1 1
+ + ≤ + + .
1+a+b 1+b+c 1+c+a 2+a 2+b 2+c
35. (IMO 2005). Dados a, b, c números reales positivos que satisfacen abc ≥ 1,
pruebe que

a5 − a2 b5 − b2 c5 − c2
+ + ≥ 0.
a5 + b2 + c2 b5 + c2 + a2 c5 + a2 + b2
134 11. Problemas

36. Sean a, b, c números reales tales que a + b + c = 3, pruebe que

√ √ √
a+ b+ c ≥ ab + bc + ca.

37. (APMO 1998). Sean x, y, z números reales positivos, pruebe que

 
x  y z 2(x + y + z)
1+ 1+ 1+ ≥2+ √ .
y z x 3 xyz

38. (USAMO 1998). Sean a, b, c números reales positivos, pruebe que

1 1 1 1
+ 3 + 3 ≤ .
a3 b3 3 3
+ + 3abc b + c + 3abc c + a + 3abc abc

39. Sean a, b, c reales positivos con suma 3, pruebe que

a b c 3
+ + ≥ .
1 + b2 1 + c2 1 + a2 2

40. Sean a, b, c, d números reales positivos, pruebe que

a3 b3 c3 d3 a+b+c+d
+ + + ≥ .
a2 + b2 b2 + c2 c2 + d2 d2 + a2 2

41. Sean a, b, c, d números reales positivos tales que abcd = 1, pruebe que

1 1 1
S = + +
1 + ab + bc + ca 1 + bc + cd + ab 1 + cd + da + ac
1
+ ≤ 1.
1 + da + ab + bd

42. Sean a, b, c números no negativos tales que a + b + c = 3, pruebe que

1 1 1
+ + ≥ 1.
2ab2 +1 2bc2 2
+ 1 2ca + 1
135

43. Sean a, b, c, x, y, z números reales no negativos tales que a + b + c = x + y + z,


pruebe que

ax(a + x) + by(b + y) + cz(c + z) ≥ 3(abc + xyz).

44. Sean a, b, c números reales positivos, pruebe que

a2 (b + c − a) b2 (c + a − b) c2 (a + b − c) ab + bc + ca
+ + ≤ .
b+c c+a a+b 2

45. Si a, b, c reales positivos, pruebe que


s s s
a3 b3 c3
+ + ≥ 1.
a3 + (b + c)3 b3 + (c + a)3 c3 + (a + b)3

46. Si a, b, c, d son números reales no negativos, pruebe que

a−b b−c c−d d−a


+ + + ≥ 0.
a + 2b + c b + 2c + d c + 2d + a d + 2a + b

47. Si a, b, c son números reales no negativos, entonces

√ √ √
a2 − bc b + c + b2 − ca c + a + c2 − ab a + b ≥ 0.

48. Dados a, b, c números reales positivos, pruebe que

 2  2  2
a b c 3
+ + ≥ .
a+b b+c c+a 4

49. Sean a, b, c, d números reales positivos que satisfacen

1 1 1 1
+ + + = 1,
1+a 1+b 1+c 1+d

pruebe que abcd ≥ 81.


136 11. Problemas

50. Sean a, b, c los lados de un triángulo, pruebe que

a b c
+ + ≥ 1.
3a − b + c 3b − c + a 3c − a + b

51. Dados a, b, c, d números reales positivos, pruebe que


21
a a 16

3
≥ 21 21 21 21 .
a3 + 63bcd a 16 + b 16 + c 16 + d 16

52. Dados a, b, c, d números reales positivos, pruebe que

a b c d
A= √
3
+√
3
+√
3
+√
3
≥ 1.
a3 + 63bcd b3 + 63acd c3 + 63bda d3 + 63abc

53. Sean a, b, c los lados de un triángulo, pruebe que


r
3 a3 + b3 + c3 + 3abc
≥ máx {a, b, c}.
2

54. Sean a, b, c números reales positivos, pruebe que

(a + b)2 (b + c)2 (c + a)2


S= + + > 2.
a2 + b2 + c2 + ab a2 + b2 + c2 + bc a2 + b2 + c2 + ca

55. Dados a, b, c números reales tales que a, b, c ∈ [0; 1], pruebe que

a b c
+ + + (1 − a)(1 − b)(1 − c) ≤ 1.
b+c+1 c+a+1 a+b+1

56. Sean a, b, c números reales positivos, pruebe que

a b c 3
+ + ≤ .
2a + b + c 2b + c + a 2c + a + b 4

57. Sean a, b, c números reales tales que a2 + b2 + c2 = 1, pruebe que


√ √ √ √
1 − ab + 1 − bc + 1 − ca ≥ 6.
137

58. Pruebe que

a4 + b4 + c4 + abc(a + b + c) ≥ ab(a2 + b2 ) + bc(b2 + c2 ) + ca(c2 + a2 ),

se cumple para todo a, b, c números reales.

59. Pruebe que

a3 + b3 + c3 + 4(a + b + c) + 9abc ≥ 8(ab + bc + ca),

se cumple para todo a, b, c números reales no negativos.

60. Pruebe que



p p p 3 2
M= a2 + (1 − b)2 + b2 + (1 − c)2 + c2 + (1 − a)2 ≥ ,
2

se cumple para todo a, b, c números reales.

61. Pruebe que


s s s s
3 3 3
a b c d3
3
+ 3
+ 3
+ ≥ 1,
a + 15bcd b + 15acd c + 15abd d3 + 15abc

se cumple para todo a, b, c números reales positivos.

62. Sean a, b, c números reales no negativos tales que ab + bc + ca = 3, pruebe que

1 1 1
+ 2 + 2 ≤ 1.
a2 +2 b +2 c +2

63. Sean a, b, c números reales positivos, pruebe que

a3 b3 c3 3 2  1
+ + ≥ a + b2 + c2 − (ab + bc + ca).
a+b b+c c+a 4 4
138 11. Problemas

64. Sean a, b, c números reales positivos, pruebe que

ab bc ca 1
S= + + ≤ (a + b + c).
2c + a + b 2a + b + c 2b + c + a 4

65. Si a ≥ b ≥ c ≥ d ≥ e, entonces

(a + b + c + d + e)2 ≥ 8(ac + bd + ce).

¿Cuándo ocurre la igualdad?

66. Sean x, y, z números reales positivos tales que xyz ≥ xy + yz + zx.


Pruebe que:

xyz ≥ 3(x + y + z).

67. Sean x, y números reales positivos tales que x + y = 1, pruebe que

x y 2
√ +p ≥√ .
1−x2 1 − y2 3

68. Sean a1 , a2 , . . . , an , b1 , b2 , . . . , bn números reales positivos, tales que a1 + a2 +


· · · + an = b1 + b2 + · · · + bn . Pruebe que

a21 a22 a2n 1


+ + ···+ ≥ (a1 + a2 + · · · + an ).
a1 + b1 a2 + b2 an + bn 2

69. Sean a, b, c números reales positivos, pruebe que

2ab 2bc 2ca


+ + ≤ a + b + c.
a+b b+c c+a

70. (Rusia 1992). Sean x, y, z números reales positivos, pruebe que


x4 + y 4 + z 2 ≥ 8xyz.
139

71. Sean a, b, c números reales positivos con a + b + c = 1, pruebe que


   
1 1 1
+1 +1 +1 ≥ 64.
a b c

72. Sean a, b, c números reales, y


√ √ √
x= b2 − bc + c2 ; y = c2 − ca + a2 ; z = a2 − ab + b2

pruebe que

xy + yz + zx ≥ a2 + b2 + c2 .

73. Sean x, y, z números reales positivos tales que x + y + z = 3, pruebe que

(1 − x)2 (1 − y)2 (1 − z)2


+ + ≥ 0.
1 − x4 1 − y4 1 − z4

74. Si a1 , a2 , . . . , an son números reales, pruebe que

n X
X n
i · j · cos (ai − aj ) ≥ 0.
i=1 j=1

75. Sean a, b, c números reales, tales que


√ √ √
x= b2 + bc + c2 ; y = c2 + ca + a2 ; z = a2 + ab + b2

pruebe que

xy + yz + zx ≥ (a + b + c)2 .

76. Sean a, b, c, d, e, f números reales positivos, pruebe que

a b c d e f
+ + + + + ≥ 3.
b+c c+d d+e e+f f +a a+b
140 11. Problemas

77. Sean a, b, c, x, y, z, t, v, u números reales positivos, entonces

  
a3 + b3 + c3 x3 + y 3 + z 3 t3 + u3 + v 3 ≥ (axt + byu + czv)3 .

78. (IMO 2001). Pruebe para a, b, c números reales positivos

a b c
√ +√ +√ ≥ 1.
a2 + 8bc 2
b + 8ca 2
c + 8ab

1 1 1
79. (APMO 2002). Sean a, b y c números reales positivos que satisfacen + + =
a b c
1, pruebe que

√ √ √ √ √ √ √
a + bc + b + ca + c + ab ≥ abc + a+ b+ c.

80. Sean a, b, c números reales positivos que satisfacen abc = 1, pruebe que

3 6
1+ ≥ .
a+b+c ab + ac + bc

81. Sean a, b, c, x, y, z números reales positivos tales que x + a = y + b = z + c = 1,


pruebe que
 
1 1 1
(abc + xyz) + + ≥ 3.
ay bz cx

82. Sean x, y, z números reales positivos, pruebe que

x+y+z xy + yz + zx
√ ≥p p √ .
3 3 x2 + xy + y 2 + y 2 + yz + z 2 + z 2 + zx + x2

83. Sean x, y, z, t números reales tales que x2 + y 2 ≤ 1 y z 2 + t2 ≤ 1, pruebe que


p p √
(x + z)2 + (y + t)2 + (x − z)2 + (y − t)2 ≤ 2 2.

84. Dados a, b, c números reales tales que a2 +b2 +c2 ≤ 8, pruebe que ab+bc+2ca ≥
−8.
141

85. Dados a, b, c números reales no negativos tales que a + b + c = 1, pruebe que


256
a4 b + b4 c + c4 a ≤ .
3125
86. Si a, b, y c son los lados de un triángulo, pruebe que

a b c
+ + ≥ 3.
b+c−a c+a−b a+b−c

87. Sea n un número entero ≥ 2, pruebe que


!4
X  X
8 xi xj x2i + x2j ≤ xi .
1≤i<j≤n 1≤i≤n

88. Sean a1 , a2 , . . . , an números reales positivos y S = a1 + a2 + · · · + an , pruebe


que

S S S n2
+ +···+ ≥ .
S − a1 S − a2 S − an n−1

89. Sean a, b, c números reales no negativos tales que a + b + c = 3, pruebe que


√ √ √
a 1 + b3 + b 1 + c3 + c 1 + a3 ≤ 5.

90. Sean a, b, c, d números reales no negativos, pruebe que


    
2a 2b 2c 2d
1+ 1+ 1+ 1+ ≥ 9.
b+c c+d d+a a+b

91. Si a, b, c son números reales positivos, entonces


r r r
2a 2b 2c
+ + ≤ 3.
a+b b+c c+a

92. Si x1 , x2 , . . . , xn números reales positivos tales que

1 1 1
+ +···+ = 1,
1 + x1 1 + x2 1 + xn
142 11. Problemas

pruebe que

x1 x2 · · · xn ≥ (n − 1)n .

93. Sean a, b, c números reales positivos tales que a2 + b2 + c2 = 3, pruebe que

ab bc ca
+ + ≥ 3.
c a b

94. Sean a, b, c números reales positivos, pruebe que

a+b+c
aa · bb · cc ≥ (abc) 3 .

95. Sean a, b, c números reales positivos, pruebe que

a b c c+a a+b b+c


+ + ≥ + + .
b c a c+b a+c b+a

96. Si a, b, c, d son números reales positivos, entonces

a−b b−c c−d d−a


+ + + ≥ 0.
b+c c+d d+a a+b

97. Sean a, b, c números reales positivos, pruebe que

a3 b3 c3 3(ab + ac + bc)
2 2
+ 2 2
+ 2 2
≥ .
b − bc + b c − ac + a a − ab + b a+b+c

98. Si a, b, c, d son números reales positivos, entonces


 2  2  2  2
a b c d
+ + + ≥ 1.
a+b b+c c+d d+a

99. Si a, b, c son números reales no negativos, entonces

  
3 1 − a + a2 1 − b + b2 1 − c + c2 ≥ 1 + abc + a2 b2 c2 .
143

100. Sean a, b, c son números reales no negativos, pruebe que

a2 + b2 + c2 + 2abc + 1 ≥ 2(ab + ac + bc).

101. Sean a, b, c números reales positivos tales que abc ≥ 1, pruebe que

a3 + b3 + c3 ≥ ab + ac + bc.

102. Sean a, b, c números reales positivos tales que a + b + c = 1, pruebe que

√ √ √ √ √ √
ab + c + bc + a + ca + b ≥ 1 + ab + bc + ca.

103. Sean a, b números reales positivos tales que a + b ≤ 1, halle el mı́nimo valor
de f (a, b) = ab + (ab)−1 .

3
104. Sean a, b, c números reales positivos tales que a + b + c ≤ , halle el mı́nimo
r r r 2
1 1 1
de f (a, b, c) = a2 + 2 + b2 + 2 + c2 + 2 .
b c a

105. Sean a, b, c números reales tales que a ≥ 2, b ≥ 6 y c ≥ 12, halle el máximo


valor de

√ √ √
bc a − 2 + ca 3 b − 6 + ab 4 c − 12
f (a, b, c) = .
abc

a3 b3 c3
106. Sean a, b, c números reales positivos, pruebe que + + ≥ ab + bc + ca.
b c a
107. Sean a, b, c, d números reales positivos, halle el mı́nimo valor de la función

a b c d
f (a, b, c, d) = + + +
b+c+d c+d+a d+a+b a+b+c
b+c+d c+d+a d+a+b a+b+c
+ + + + .
a b c d
144 11. Problemas

108. Halle el máximo valor de

a3 (b + c + d) + b3 (c + d + a) + c3 (d + a + b) + d3 (a + b + c)

donde a, b, c, d son números reales tales que la suma de cuadrados es 1.

109. Sean a, b, c números reales positivos, pruebe que

 
a+b+c √
3 √ √ 2 √ √ 2 √ √ 2
− abc ≤ máx a− b , b− c , c− a .
3

110. Si a, b, c son números reales no negativos, pruebe que

 3
3 3 b+c
3
a +b +c +2 a− ≥ 3abc.
2

111. Sean x1 , x2 , x3 , x4 , x5 números reales tales que x1 + x2 + x3 + x4 + x5 = 0,


pruebe que

|cos x1 | + |cos x2 | + |cos x3 | + |cos x4 | + |cos x5 | ≥ 1.

112. Sean a, b, c números reales diferentes, pruebe que

a2 b2 c2
+ + ≥ 2.
(b − c)2 (c − a)2 (a − b)2

113. (IMO 1960). Resuelva

4x2
√ 2 < 2x + 9.
1 − 1 + 2x

114. (IMO 1964). Si a, b, c son los lados de un triángulo, pruebe que

a2 (b + c − a) + b2 (a + c − b) + c2 (a + b − c) ≤ 3abc.
145

115. (IMO 1975). Considere las sucesiones (x1 , x2 , . . . , xn , . . .) y (y1 , y2, . . . , yn , . . .)


tales que x1 ≤ x2 ≤ · · · ≤ xn , y1 ≤ y2 ≤ · · · ≤ yn , y (z1 , z2 , . . . , zn ) es una
permutación de (y1 , y2 , . . . , yn ), pruebe que

(x1 − y1 )2 + · · · + (xn − yn )2 ≤ (x1 − z1 )2 + · · · + (xn − zn )2 .

116. (IMO 1978). Sean x1 , x2 , . . . , xn números enteros diferentes, pruebe que

x1 x2 xn 1 1 1
2
+ 2 +···+ 2 ≥ + +···+ .
1 2 n 1 2 n

117. (IMO 1997). Sean x1 , x2 , . . . , xn números reales que satisfacen

n+1
|x1 + x2 + · · · + xn | = 1 y |xi | ≤ , para todo i = 1, 2, . . . , n,
2

pruebe que existe la permutación (y1 , y2 , . . . , yn ) de (x1 , x2 , . . . , xn ) tal que

n+1
|y1 + 2y2 + · · · + nyn | ≤ .
2

118. (IMO 2004). Sea n un número entero positivo ≥ 3 y sean t1 , t2 , . . . , tn números


reales positivos tales que
 
2 1 1 1
n + 1 > (t1 + t2 + · · · + tn ) + +···+ ,
t1 t2 tn

pruebe que ti , tj , tk son los lados de un triángulo para todo i, j, k con 1 ≤ i <
j < k ≤ n.

119. (IMO 2000). Sean a, b, c números reales positivos tales que abc = 1, pruebe
que
   
1 1 1
a−1+ b−1+ c−1+ ≤ 1.
b c a
146 11. Problemas

120. Pruebe que

n
X nX
|i − j| |xi − xj | = |xi − xj |
i,j=1
2 i,j

si x1 ≤ x2 ≤ · · · ≤ xn .

121. Pruebe que

n
X n2 (n2 − 1)
(i − j)2 = .
i,j=1
6

122. (IMO 2003). Sea n un número entero ≥ 2 y los números reales x1 , x2 , . . . , xn


con x1 ≤ x2 ≤ · · · ≤ xn , pruebe que

n
!2 n
X 2 2 X
|xi − xj | ≤ n −1 (xi − xj )2 .
i,j=1
3 i,j=1

123. (IMO 2006). Determine el menor valor de M para que la desigualdad


ab(a2 − b2 ) + bc(b2 − c2 ) + ca(c2 − a2 ) ≤ M a2 + b2 + c2 2

se verifique para cualesquiera a, b, c números reales.

124. (IMO 1983). Sean a, b, c los lados de un triángulo, pruebe que

a2 b(a − b) + b2 c(b − c) + c2 a(c − a) ≥ 0.

125. Sean a, b, c, d, e números reales positivos tales que abcde = 1, pruebe que

a + abc b + bcd c + cde d + dea


+ + +
1 + ab + abcd 1 + bc + bcde 1 + cd + cdea 1 + de + deab
e + eab 10
+ ≥ .
1 + ea + eabc 3
147

126. Sean a, b, c que satisfacen a2 + b2 + c2 = 1, halle el máximo valor de

P = (a − b)(a − c)(b − c)(a + b + c).

127. (IMO 2008).

(a) Demuestre que

x2 x2 x2
+ + ≥1 (11.1)
(x − 1)2 (x − 1)2 (x − 1)2

para todos los números reales x, y, z, distintos de 1, con xyz = 1.


(b) Demuestre que existen infinitas ternas de números racionales x, y, z, dis-
tintos de 1, con xyz = 1 para los cuales la expresión (11.1) es una igual-
dad.
148 11. Problemas
CAPÍTULO 12
SOLUCIONES

1. Sean x, y números reales positivos tales que a y b son las medias aritmética y
geométrica respectivamente, pruebe que

(1 + b)2 ≤ (1 + x)(1 + y) ≤ (1 + a)2 .

x+y √
 Prueba. Sean a = , b = xy, y como se cumple:
2


2 xy ≤ x + y

⇔ 1 + 2 xy + xy ≤ 1 + x + y + xy

⇔ (1 + xy)2 ≤ (1 + x)(1 + y)
⇔ (1 + b)2 ≤ (1 + x)(1 + y). (12.1)

También se cumple

4xy ≤ (x + y)2
(x + y)2
⇔ xy ≤
4
(x + y)2
⇔ 1 + x + y + xy ≤ 1 + x + y +
4
149
150 12. Soluciones

 2
x+y
⇔ (1 + x)(1 + y) ≤ 1 +
2
⇔ (1 + x)(1 + y) ≤ (1 + a)2 (12.2)

Luego de (12.1) y (12.2) se tiene:

(1 + b)2 ≤ (1 + x)(1 + y) ≤ (1 + a)2 .

2. Pruebe que
    2
1 1 2
1+ 1+ ≥ 1+
x y x+y

para todo x, y números reales positivos.

 Prueba. La desigualdad es equivalente a:

1 1 1 4 4
1+ + + ≥ 1+ +
x y xy x + y (x + y)2
⇔ (x + y + 1)(x + y)2 ≥ 4(x + y)xy + 4xy
⇔ (x + y + 1)(x + y)2 ≥ 4xy(x + y + 1)
⇔ (x + y)2 ≥ 4xy
⇔ (x − y)2 ≥ 0.
2

3. Sean x, y, z números reales positivos, pruebe que al menos una de las siguientes
desigualdades es verdadera:

xy ≥ 0
yz ≥ 0
zx ≥ 0.
151

 Prueba. Supongamos que no se cumple ninguna de las desigualdades,


esto es

xy < 0
yz < 0
zx < 0.

Luego tendrı́amos que −xy > 0, −yz > 0, −zx > 0, y multiplicando −x2 y 2 z 2 >
0, lo cual es una contradicción. Por lo tanto una de las desigualdades debe ser
verdadera. 2

4. Si a2 + b2 = 1, halle la variación de 2a + 3b.

Solución. Escogemos los pares (a, b) y (2, 3) para utilizar la desigualdad de


Cauchy-Schwarz.
En efecto

(2a + 3b)2 ≤ (a2 + b2 ) (22 + 32 )


⇒ (2a + 3b)2 ≤ (1)(13)
⇒ (2a + 3b)2 ≤ 13

√ √
de donde − 13 ≤ 2a + 3b ≤ 13. 2

5. Sabiendo que 2a + 3b + 4c = 29, halle el menor valor de a2 + b2 + c2 .

Solución. Tomamos las ternas (2, 3, 4) y (a, b, c), luego aplicamos Cauchy-
Schwarz:

(2a + 3b + 4c)2 ≤ (22 + 32 + 42 ) (a2 + b2 + c2 )


⇔ (29)2 ≤ (29) (a2 + b2 + c2 )
⇔ a2 + b2 + c2 ≥ 29

∴ El mı́nimo de a2 + b2 + c2 es 29 y ocurre si y sólo si a = 2, b = 3, c = 4. 2


152 12. Soluciones

6. Pruebe que
p √ p
m2 + y 2 + n2 + x2 ≥ (m + n)2 + (x + y)2

para todo x, y, m, n números reales.

 Prueba. Aplicando Cauchy-Schwarz a (m, y) y (n, x), tenemos

(m2 + y 2)(n2 + x2 ) ≥ (mn + xy)2


p
⇔ (m2 + y 2 )(n2 + x2 ) ≥ (mn + xy)
p
⇔ (m2 + y 2) + (n2 + x2 ) + 2 (m2 + y 2 )(n2 + x2 ) ≥ (m2 + n2 + 2mn)

+(x2 + y 2 + 2xy)
p √ 2
⇔ m2 + y2 + n2 + x2 ≥ (m + n)2 + (x + y)2
p √ p
⇔ m2 + y 2 + n2 + x2 ≥ (m + n)2 + (x + y)2.

7. Si a, b, c son números reales positivos que satisfacen abc = 1, pruebe que

1 + ab 1 + bc 1 + ac
+ + ≥ 3.
1+a 1+b 1+c

 Prueba.

1 + ab c + abc c+1
= = ; similarmente
1+a c(1 + a) c(1 + a)
1 + bc a+1
=
1+b a(1 + b)
1 + ac b+1
=
1+c b(1 + c)
153

1
vemos que al multiplicar las tres últimas fracciones obtenemos = 1, en-
abc
tonces aplicando MA ≥ MG, tenemos:

c+1 a+1 b+1


+ + r
c(1 + a) a(1 + b) b(1 + c) c+1 a+1 b+1
≥ 3
· ·
3 c(1 + a) a(1 + b) b(1 + c)
r
3 1
= =1
cab

entonces

c+1 a+1 b+1


+ + ≥3
c(1 + a) a(1 + b) b(1 + c)

1 + ab 1 + bc 1 + ac
∴ + + ≥ 3.
1+a 1+b 1+c
2

8. Sean a, b, c números reales positivos, pruebe que

a b c 3
+ + ≥ .
b+c c+a a+b 2

(Desigualdad de Nesbit).

 Prueba. La desigualdad es equivalente a:


     
a b c 3
+1 + +1 + +1 ≥ +3
b+c c+a a+b 2
a+b+c a+b+c a+b+c 9
⇔ + + ≥
b+c c+a a+b 2
 
1 1 1 9
⇔ (a + b + c) + + ≥
b+c c+a a+b 2
154 12. Soluciones

 
1 1 1
⇔ 2(a + b + c) + + ≥9
b+c c+a a+b
 
1 1 1
⇔ ((b + c) + (c + a) + (a + b)) + + ≥9
b+c c+a a+b
(b + c) + (c + a) + (a + b) 3
⇔ ≥
3 1 1 1
+ +
b+c c+a a+b

esta última desigualdad es verdadera, pues MA ≥ MH. 2

9. Pruebe que si x, y, z son números reales positivos y x + y + z = 1, entonces


   
1 1 1
1+ 1+ 1+ ≥ 64.
x y z

 Prueba. La desigualdad es equivalente a:


   
x+1 y+1 z+1
≥ 64
x y z
⇔ (x + x + y + z)(y + x + y + z)(z + x + y + z) ≥ 64xyz
⇔ (2x + y + z)(x + 2y + z)(x + y + 2z) ≥ 64xyz

aplicando MA ≥ MG:

x+x+y+z √
≥ 4x·x·y·z
4
p
2x + y + z ≥ 4 4 x2 yz; similarmente
p
x + 2y + z ≥ 4 4 xy 2 z
p
x + y + 2z ≥ 4 4 xyz 2

multiplicando miembro a miembro:

(2x + y + z)(x + 2y + z)(x + y + 2z) ≥ 64xyz.

2
155

10. Sean a, b, c, d números reales positivos tales que a2 + b2 = (c2 + d2 )3 , pruebe


que

c3 d 3
+ ≥ 1.
a b

 Prueba. El primer miembro de la desigualdad

2 2
c3 d 3 c4 d4 (c2 ) (d2 )
+ = + = +
a b ac bd ac bd
luego, utilizando el Lema de Titu, tenemos

 2 !2 !2
2 2 2
c3 d 3 (c2 ) (d2 ) (c2 + d2 )
+ = + ≥
a b ac bd ac + bd
4
(c2 + d2 )
=
(ac + bd)2
3
(c2 + d2 ) (c2 + d2 )
=
(ac + bd)2
(a2 + b2 ) (c2 + d2 )
=
(ac + bd)2

entonces
 2
c3 d 3 (a2 + b2 ) (c2 + d2 )
+ ≥ ≥1
a b (ac + bd)2

pues por Cauchy-Schwarz


 
a2 + b2 c2 + d2 ≥ (ac + bd)2 .

11. Sean a, b, c números reales positivos, pruebe que

a b c
+ + ≥ 1.
b + 2c c + 2a a + 2b
156 12. Soluciones

 Prueba. Vemos que

a b c a2 b2 c2
+ + = + +
b + 2c c + 2a a + 2b ab + 2ac bc + 2ab ac + 2bc

aplicando el Lema de Titu, tenemos

a2 b2 c2 (a + b + c)2
+ + ≥ ,
ab + 2ac bc + 2ab ac + 2bc 3(ab + ac + bc)

luego bastarı́a demostrar que

(a + b + c)2
≥ 1
3(ab + ac + bc)
⇔ a2 + b2 + c2 + 2(ab + bc + ca) ≥ 3(ab + ac + bc)
⇔ a2 + b2 + c2 ≥ ab + ac + bc,

pero esta última desigualdad es verdadera.

a b c
∴ + + ≥ 1.
b + 2c c + 2a a + 2b
2

12. Pruebe que

x2 y2
+ ≥ 8,
y−1 x−1

para todo x, y > 1.

 Prueba. Como (x − 2)2 ≥ 0, para todo x real, en particular para x > 1,


efectuando

x2 − 4x + 4 ≥ 0
⇔ x2 ≥ 4(x − 1)

⇔ x≥2 x−1
x
⇔ √ ≥2
x−1
157

similarmente

y
√ ≥2
y−1

x y
como √ ,√ ∈ R+ ; entonces aplicando MA ≥ MG
x−1 y−1
x2 y2 s
+
y−1 x−1 x2 y2
≥ ·
2 y−1 x−1
x2 y2 xy x y
+ ≥ 2· √ √ = 2· √ ·√
y−1 x−1 x−1· y−1 x−1 y−1
≥ 2(2)(2) = 8

x2 y2
∴ + ≥ 8.
y−1 x−1
2

13. Sean a, b, c números reales positivos tales que a + b + c = 2, pruebe que

a+b b+c c+a 9


+ + ≥ .
2a + bc 2b + ac 2c + ab 4

 Prueba. Veamos la fracción

a+b a+b a+b a+b 1


= = 2 = = .
2a + bc (a + b + c)a + bc a + (b + c)a + bc (a + b)(a + c) a+c

Similarmente para las otras dos fracciones y la desigualdad se escribe ası́:

a+b b+c c+a 9


+ + ≥
(a + b)(a + c) (b + a)(b + c) (c + a)(c + b) 4
1 1 1 9
⇔ + + ≥ .
a+c b+a c+b 4
158 12. Soluciones

Pero
 
1 1 1
[(a + c) + (b + a) + (c + b)] + + ≥9
a+c b+a c+b
 
1 1 1
⇔ 2(a + b + c) + + ≥9
a+c b+a c+b
 
1 1 1
⇔ 4 + + ≥9
a+c b+a c+b
1 1 1 9
⇔ + + ≥
a+c b+a c+b 4

a+b b+c c+a 9


∴ + + ≥ .
2a + bc 2b + ac 2c + ab 4
2

14. Si a, b son números reales tales que a2 + b2 = 1, pruebe que



2 2
a b + ab2 ≤ .
2

 Prueba. 2
a b + ab2 = |ab(a + b)| = |ab||a + b|

pero

√  √ √
|a + b| ≤ 2 a2 + b2 = 2 · 1 = 2

|a + b| ≤ 2 (12.3)

además

a2 + b2 ≥ 2|a||b|
1
⇔ ≥ |ab|
2
1
⇔ |ab| ≤ (12.4)
2
159

multiplicando miembro a miembro:



2
|ab||a + b| ≤
2


2
∴ a2 b + ab2 ≤ .
2
2

15. Sean x, y números reales positivos, pruebe que


√ √
x y
√ +√ ≥ 1.
x + 3y y + 3x

 Prueba. Usando la identidad (a + b)2 = a2 + 2ab + b2 , en efecto

(a + b)2 = a2 + 2ab + b2

3
= a2 + ab + ab + b2 ≥ a2 + 3 a2 b4
⇒ (a + b)2 ≥ a2 + 3a2/3 · b4/3

= a2/3 a4/3 + 3b4/3
p √
⇒ (a + b) ≥ a2/3 (a4/3 + 3b4/3 ) = a1/3 a4/3 + 3b4/3

Haciendo a4/3 = x, b4/3 = y, tenemos:



3/4 3/4 1/4
√ x x3/4
x +y ≥x x + 3y ⇔ √ ≥ 3/4
x + 3y x + y 3/4

y y 3/4
similarmente √ ≥ 3/4
y + 3x x + y 3/4

sumando obtenemos
√ √
x y
√ +√ ≥ 1.
x + 3y y + 3x
2
160 12. Soluciones

16. Sean a, b, c números reales positivos tales que ab + bc + ca ≤ 3abc, pruebe que

a + b + c ≤ a3 + b3 + c3 .

 Prueba. Como ab + bc + ca ≤ 3abc

ab + bc + ca
⇒ ≤ 3
abc
1 1 1
⇒ + + ≤ 3
a b c
 
1 1 1
Pero (a + b + c) + + ≥9
a b c

⇒ a+b+c≥3

además

3(a + b + c) ≤ (a + b + c)2
2
= a3/2 · a−1/2 + b3/2 · b−1/2 + c3/2 · c−1/2
≤ (a3 + b3 + c3 ) (a−1 + b−1 + c−1 )
≤ (a3 + b3 + c3 ) 3


⇒ 3(a + b + c) ≤ 3 a3 + b3 + c3

∴ a + b + c ≤ a3 + b3 + c3 .

17. Sean a, b, c números reales positivos tales que a2 + b2 + c2 = 3, pruebe que

1 1 1
+ + ≥ 1.
1 + 2ab 1 + 2bc 1 + 2ca
161

 Prueba. Recordar que

a2 + b2 ≥ 2ab
⇔ 1 + a2 + b2 ≥ 1 + 2ab
1 1
⇔ ≥
1 + 2ab 1 + a2 + b2

en efecto

1 1 1 12 12 12
+ + ≥ + +
1 + 2ab 1 + 2bc 1 + 2ca 1 + a2 + b2 1 + b2 + c2 1 + a2 + c2

aplicando el Lema de Titu

12 12 12 (1 + 1 + 1)2
+ + ≥
1 + a2 + b2 1 + b2 + c2 1 + a2 + c2 3 + 2(a2 + b2 + c2 )
32
= =1
3 + 2(3)

1 1 1
∴ + + ≥ 1.
1 + 2ab 1 + 2bc 1 + 2ca
2

18. Sean a, b, c números reales positivos tales que a2 + b2 + c2 = 3, pruebe que

1 1 1 3
+ + ≥ .
1 + ab 1 + bc 1 + ca 2

 Prueba.

2
b2 + c}2 ≥ ab + ac + bc
|a + {z
3

⇔ 3 ≥ ab + ac + bc
⇔ 6 ≥ (1 + ab) + (1 + ac) + (1 + bc)
9 9
⇔ ≤
6 3 + ab + ac + bc
162 12. Soluciones

9 3
⇔ ≥
3 + ab + ac + bc 2

además sabemos que

 
1 1 1
[(1 + ab) + (1 + ac) + (1 + bc)] + + ≥9
1 + ab 1 + ac 1 + bc
1 1 1 9
⇔ + + ≥
1 + ab 1 + ac 1 + bc 3 + ab + ac + bc

1 1 1 3
∴ + + ≥ .
1 + ab 1 + bc 1 + ca 2
2

19. Pruebe que



(a + b + c)4 ≥ 8 a2 + b2 + c2 (ab + ac + bc).

para todo a, b, c números reales.

 Prueba.

2
(a + b + c)4 = ((a + b + c)2 )
 2
2
= a b2 + c}2 + 2(ab + ac + bc)
| + {z | {z }
x y
2
= (x + y)
≥ 4xy
= 4 (a2 + b2 + c2 ) (2)(ab + ac + bc)
= 8 (a2 + b2 + c2 ) (ab + ac + bc)


∴ (a + b + c)4 ≥ 8 a2 + b2 + c2 (ab + ac + bc).

2
163

20. Dados a, b, c números reales positivos con abc = 8, pruebe que

1 1 1
√ +√ +√ ≥ 1.
1+a3 1+b3 1 + c3

 Prueba. La suma de cubos lo descomponemos para aplicar MA ≥ MG.


Veamos

1 + a3 = (1 + a)(1 − a + a2 ),

entonces

(1 + a) + (1 − a + a2 ) p
≥ (1 + a)(1 − a + a2 )
2
2 + a2 √
⇔ ≥ 1 + a3
2
2 1
⇒ 2
≤√ ;
2+a 1 + a3

luego

1 1 1 2 2 2
√ +√ +√ ≥ 2
+ 2
+
1+a3 1+b3 1+c 3 2+a 2+b 2 + c2

como vemos basta probar que

2 2 2
2
+ 2
+ ≥1
2+a 2+b 2 + c2
1 1 1 1
⇔ 2
+ 2
+ 2

2+a 2+b 2+c 2
⇔ (2 + b2 ) (2 + c2 ) + (2 + a2 ) (2 + c2 ) + (2 + a2 ) (2 + b2 )
1
≥ (2 + a2 ) (2 + b2 ) (2 + c2 )
2
164 12. Soluciones

⇔ 12 + 4 (a2 + b2 + c2 ) + |b2 c2 + a{z


2 2
c + a2 b}2
 
1 2 2 2 2 2 2 2 2 2
 2 2 2
≥ 8 + 4 (a + b + c ) + 2 a b + a c + b c +a b c
2 | {z }
1
⇔ 2 (a2 + b2 + c2 ) + 8 ≥ a2 b2 c2 ; abc = 8
2
⇔ 2 (a2 + b2 + c2 ) + 8 ≥ 32

⇔ a2 + b2 + c2 ≥ 12.

Es verdadera pues

3
a2 + b2 + c2 ≥ 3 a2 b2 c2 = 12.

21. Sean a, b números reales positivos, pruebe que

1 1 1
2
+ 2
≥ .
(1 + a) (1 + b) 1 + ab

 Prueba.

1 1 1
2
+ 2
≥ ;
(1 + a) (1 + b) 1 + ab

⇔ [(1 + a)2 + (1 + b)2 ] (1 + ab) ≥ (1 + a)2 (1 + b)2

⇔ (2 + 2a + 2b + a2 + b2 ) (1 + ab) ≥ (1 + 2a + a2 )(1 + 2b + b2 )

⇔ a3 b + ab3 + 1 ≥ 2ab + a2 b2

⇔ ab (a2 + b2 − 2ab) + (a2 b2 − 2ab + 1) ≥ 0

⇔ ab(a − b)2 + (ab − 1)2 ≥ 0.

La igualdad ocurre si y sólo si a = b = 1. 2


165

22. Sean a, b, c, d números reales positivos tales que abcd = 1, pruebe que

1 1 1 1
+ + + ≥ 1.
(1 + a)2 (1 + b)2 (1 + c)2 (1 + d)2

 Prueba. Aplicando el problema 21, tenemos

1 1 1 1 1 1
2
+ 2
+ 2
+ 2
≥ +
(1 + a) (1 + b) (1 + c) (1 + d) 1 + ab 1 + cd
| {z } | {z }
1 ab
= +
1 + ab ab + abcd
1 ab
= + = 1.
1 + ab ab + 1

1 1 1 1
∴ 2
+ 2
+ 2
+ ≥ 1.
(1 + a) (1 + b) (1 + c) (1 + d)2

La igualdad ocurre si y sólo si a = b = c = d = 1. 2

23. Sean a, b, c números reales positivos, pruebe que

2 (a3 + b3 + c3 ) + (3a + 1) (b2 + c2 ) + (3b + 1) (a2 + c2 ) + (3c + 1) (a2 + b2 )


≥ 2(ab + ac + bc) + 3(a + b)(a + c)(b + c)

 Prueba. (a − b)2 ≥ 0, para todo a, b reales, luego (a − b)2 + (a + b)3 ≥


(a + b)3 ; efectuando

a2 − 2ab + b2 + a3 + b3 + 3a2 b + 3ab2 ≥ (a + b)3


a3 + b3 + a2 (3b + 1) + b2 (3a + 1) ≥ (a + b)3 + 2ab; similarmente
a3 + c3 + a2 (3c + 1) + c2 (3a + 1) ≥ (a + c)3 + 2ac
b3 + c3 + b2 (3c + 1) + c2 (3b + 1) ≥ (b + c)3 + 2bc
166 12. Soluciones

sumando miembro a miembro:

2 (a3 + b3 + c3 ) + (3a + 1) (b2 + c2 ) + (3b + 1) (a2 + c2 ) + (3c + 1) (a2 + b2 )


≥ (a + b)3 + (a + c)3 + (b + c)3 +2ab + 2ac + 2bc
| {z }
≥ 3(a + b)(a + c)(b + c) + 2(ab + ac + bc).

24. (IMO 1995). Sean a, b, c números reales positivos tales que abc = 1, pruebe
que

1 1 1 3
+ 3 + 3 ≥ .
a3 (b+ c) b (c + a) c (a + b) 2

 Prueba. Como abc = 1, entonces la desigualdad es equivalente a:

abc abc abc 3


+ 3 + 3 ≥
a3 (b
+ c) b (c + a) c (a + b) 2
1 1 1
a2 2 2 3
⇔ + b + c ≥ .
1 1 1 1 1 1 2
+ + +
b c b c b c
1 1 1
Haciendo = x, = y, = z, ⇒ xyz = 1. Luego basta demostrar que
a b c
x2 y2 z2 3
+ + ≥ ,
y+z x+z x+y 2

aplicando el Lema de Titu:

x2 y2 z2 (x + y + z)2
+ + ≥
y+z x+z x+y (y + z) + (x + z) + (x + y)
(x + y + z)2 x+y+z 3 √ 3
= = ≥ · 3 xyz = .
2(x + y + z) 2 2 | {z } 2
1

2
167

25. Demuestre que

     
2 1 1 2 1 1 2 1 1
a + +b + +c + ≥ 2(a + b + c)
b c a c a b

para todo a, b, c números reales positivos.

 Prueba. Como

a2 + b2 ≥ 2ab
⇔ a2 − ab + b2 ≥ ab; multiplicando por(a + b)
⇔ (a + b) (a2 − ab + b2 ) ≥ ab(a + b)
⇔ a3 + b3 ≥ ab(a + b)
a3 b3
⇔ + ≥a+b
ab ab
a2 b2
⇔ + ≥ a + b; similarmente
b a
a2 c2
+ ≥a+c
c a
b2 c2
+ ≥b+c
c b

sumando miembro a miembro y agrupando

     
2 1 1 2 1 1 2 1 1
a + +b + +c + ≥ 2(a + b + c)
b c a c a b

26. Sean x, y, z números reales que pertenecen al intervalo [−1, 1] tales que x +
y + z = 0, pruebe que

p p √
1 + x + y2 + 1 + y + z2 + 1 + z + x2 ≥ 3.
168 12. Soluciones

√ √ √
 Prueba. Si ab ≥ 0, veamos que 1+a+ 1+b≥1+ 1 + a + b.
En efecto
√ √ √
1+a+ 1+b≥ 1+ 1+a+b
p √
⇔ 2 + a + b + 2 (1 + a)(1 + b) ≥ 1 + 2 1 + a + b + 1 + a + b
√ √
⇔ 1 + a + b + ab ≥ 1+a+b

⇔ 1 + a + b + ab ≥ 1 + a + b

⇔ ab ≥ 0.

Para aplicar la afirmación anterior, veamos el signo de x + y 2, y + z 2 , z + x2 .


Como x+y 2 +y +z 2 +z +x2 = x + y + z +x2 +y 2 +z 2 ≥ 0, entonces escogemos
| {z }
0
dos de ellos del mismo signo, sin pérdida de generalidad podemos escoger x+y 2
y y + z 2 . Luego tenemos que
p p √
1 + x + y2 + 1 + y + z2 + 1 + z + x2
p √
≥ 1+ 1 + x + y2 + y + z2 + 1 + z + x2
q √
= 1+ 1 + x + y +y 2 + z 2 + 1 + z + x2
| {z }
p √
= 1+ 1 − z + z2 + y2 + 1 + z + x2
q q
√ 2 √ 2
= 1+ 1−z+ z2 + y2 + 1 + z + x2
q √ √ 2
≥ 1+ 1 − z + z2 + 1+z + (x + y)2
q √ √ 2
= 1+ 1 − z + z2 + 1+z + (−z)2
q √ √ 2
= 1+ 1 − z + z2 + 1+z + z2
169

luego basta demostrar que


√ √ 2
1 − z + z2 + 1 + z + z2 ≥ 4
p
⇔ 1 − z + z 2 + 1 + z + 2 (1 − z + z 2 )(1 + z) + z 2 ≥ 4
p
⇔ 2z 2 + 2 (1 − z + z 2 )(1 + z) ≥ 2
p
⇔ z 2 + (1 − z + z 2 )(1 + z) ≥ 1
p 2
⇔ (1 − z + z 2 )(1 + z) ≥ 1| −
{zz};
(+)

elevando al cuadrado

⇔ 1 + z 3 ≥ 1 − 2z 2 + z 4
⇔ z 3 ≥ z 4 − 2z 2
⇔ z 4 − z 3 − 2z 2 ≤ 0
⇔ z 2 (z 2 − z − 2) ≤ 0
⇔ z 2 (z − 2)(z + 1) ≤ 0
⇔ z 2 (2 − z)(z + 1) ≥ 0

es verdadero pues |z| ≤ 1.


La igualdad ocurre si y sólo si x = y = z = 0. 2

27. Sean a, b, c números reales, pruebe que



a+b+c+ a2 + b2 + c2 − ab − bc − ca ≤ 3 máx {a, b, c}. (12.5)

 Prueba. Supongamos que máx {a, b, c} = a

⇒ a ≥ b; a ≥ c.

Luego (12.5) es equivalente a:



a + b + c + a2 + b2 + c2 − ab − bc − ca ≤ 3a

⇔ a2 + b2 + c2 − ab − bc − ca ≤ 2a
| −{zb − }c
(+)
170 12. Soluciones

elevando al cuadrado

⇔ a2 + b2 + c2 − ab − bc − ca ≤ 4a2 + b2 + c2 − 4ab − 4ac + 2bc


⇔ 0 ≤ 3a2 − 3ab − 3ac + 3bc
⇔ a2 − a(b + c) + bc ≥ 0
⇔ (a − b)(a − c) ≥ 0.
2

28. Dados x, y, z números reales positivos tales que x + y + z = 1, pruebe que


r
x y z 3
√ +√ +√ ≥ .
y+z z+x x+y 2

x
 Prueba. Definimos la función f (x) = √ ; x < 1, entonces
1−x
2−x
f ′ (x) = √
2(1 − x) 1 − x

′′ 1 − x(4 − x)
f (x) = >0
4(1 − x)3

luego f es convexa; entonces


 
x+y+z 1
f ≤ (f (x) + f (y) + f (z))
3 3
x+y+z  
3 1 x y z
⇒ r ≤ √ +√ +√
x+y+z 3 1−x 1−y 1−z
1−
3

reemplazando x + y + z = 1, tenemos

1  
1 x y z
r 3 ≤ √ +√ +√
1 3 1−x 1−y 1−z
1−
3
171

r
x y z 3
∴ √ +√ +√ ≥ .
y+z z+x x+y 2

1
La igualad ocurre si y sólo si x = y = z = . 2
3

29. (IMO 1964). Sean a, b, c los lados de un triángulo, pruebe que

a2 (b + c − a) + b2 (a + c − b) + c2 (a + b − c) ≤ 3abc.

 Prueba. Efectuando en el primer miembro

a2 (b + c) − a3 + b2 (a + c) − b3 + c2 (a + b) − c3 ≤ 3abc
⇔ a2 (b + c) + b2 (a + c) + c2 (a + b) ≤ a3 + b3 + c3 + 3abc
⇔ 0 ≤ (a3 − a2 (b + c) + abc) + (b3 − b2 (a + c) + abc) + (c3 − c2 (a + b) + abc)
⇔ 0 ≤ a (a2 − a(b + c) + bc) + b (b2 − b(a + c) + ac) + c (c2 − c(a + b) + ab)
⇔ 0 ≤ a(a − b)(a − c) + b(b − a)(b − c) + c(c − a)(c − b)
⇔ a(a − b)(a − c) + b(b − a)(b − c) + c(c − a)(c − b) ≥ 0

es verdadera, pues es la desigualdad de Schür. 2

30. Dados a, b números reales tales que a2 + b2 = 1, pruebe que



3 3
ab + máx {a; b} ≤ . (12.6)
4

 Prueba. Sin pérdida de generalidad, supongamos que

máx {a; b} = a

entonces la desigualdad (12.6) es equivalente a:



3 3
ab + a ≤ .
4
172 12. Soluciones

Definamos la función

f (a, b) = ab + a = a(b + 1)
⇒ f 2 (a, b) = a2 (b + 1)2 = (1 − b2 ) (1 + b)2
= (1 − b)(1 + b)3
1
= (3 − 3b)(1 + b)3
3

aplicando MG ≤ MG
 4
1 3 1 3 − 3b + 1 + b + 1 + b + 1 + b
(3 − 3b)(1 + b) ≤
3 3 4
 4
1 3 27
= =
3 2 16

27
⇒ f 2 (a, b) ≤ .
16

3 3
∴ f (a, b) ≤ .
4
2

31. Dados a, b, c, d números reales no negativos, pruebe que

a b c d
+ 2 + 2 + 2 ≥ 2.
b2 2
+c +d 2 2
c +d +a2 2
d +a +b2 a + b2 + c2

√ 1+x
 Prueba. Recordar que x≤ , entonces tenemos que
2

r b2 + c2 + d2
2
b +c +d 2 2 1+ a2 + b2 + c2 + d2
≤ a2 =
a2 2 2a2
173

a 2a2
⇒ ≥ ; similarmente
b + c2 + d 2
2 a2 + b2 + c2 + d2
b 2b2
≥ 2
c + d2 + a2
2 a + b2 + c2 + d2
c 2c2
≥ 2
d + a2 + b2
2 a + b2 + c2 + d2
d 2d2
≥ 2
a + b2 + c2
2 a + b2 + c2 + d2

sumando miembro a miembro se obtiene

a b c d
+ 2 + 2 + 2 ≥ 2.
b2 2
+c +d 2 2
c +d +a2 2
d +a +b2 a + b2 + c2
2

32. Sean a, b, c, d números reales no negativos tales que a2 +b2 +c2 +d2 = 4, pruebe
que

a3 + b3 + c3 + d3 ≤ 8.

 Prueba. Si utilizamos la desigualdad de la media potencial tendrı́amos


 1/2  3 1/3
a2 + b2 + c2 + d2 a + b3 + c3 + d3

4 4
 1/2  3 1/3
4 a + b3 + c3 + d3
⇔ ≤
4 4

de donde a3 + b3 + c3 + d3 ≥ 1
como vemos no hemos acotado superiormente, es decir esta propiedad no ayuda
resolver el problema; buscaremos otra opción.
De la condición a2 + b2 + c2 + d2 = 4, se deduce que

a2 ≤ 4;
⇒ a ≤ 2; a ≥ 0
174 12. Soluciones

⇒ a3 ≤ 2a2 ; similarmente

b3 ≤ 2b2

c3 ≤ 2c2

d3 ≤ 2d2

sumando miembro a miembro:

a3 + b3 + c3 + d3 ≤ 2 (a2 + b2 + c2 + d2 )

a3 + b3 + c3 + d3 ≤ 2 (4)

de donde a3 + b3 + c3 + d3 ≤ 8.
La igualdad ocurre si y sólo si (a, b, c, d) = (2, 0, 0, 0), y sus respectivas per-
mutaciones. [8, pág 15] 2

33. Sean a, b, c los lados de un triángulo, pruebe que

a b c
+ + ≥ 1.
3a − b + c 3b − c + a 3c − a + b

 Prueba. Multiplicando por 4 a los miembros de la desigualdad, tenemos


que

a b c
+ + ≥1
3a − b + c 3b − c + a 3c − a + b
4a 4b 4c
⇔ + + ≥4
3a − b + c 3b − c + a 3c − a + b
     
4a 4b 4c
⇔ −1 + −1 + −1 ≥1
3a − b + c 3b − c + a 3c − a + b
a+b−c b+c−a c+a−b
⇔ + + ≥1
3a − b + c 3b − c + a 3c − a + b
175

En el primero miembro de la desigualdad generamos las condiciones para


aplicar el Lema de Titu

a+b−c b+c−a c+a−b


+ +
3a − b + c 3b − c + a 3c − a + b
(a + b − c)2 (b + c − a)2 (c + a − b)2
= + +
(a + b − c)(3a − b + c) (b + c − a)(3b − c + a) (c + a − b)(3c − a + b)
(a + b − c + b + c − a + c + a − b)2

(a + b − c)(3a − b + c) + (b + c − a)(3b − c + a) + (c + a − b)(3c − a + b)
| {z }
(a+b+c)2

(a + b + c)2
= = 1.
(a + b + c)2

34. Pruebe que si a, b, c son números reales positivos tales que abc = 1,entonces

1 1 1 1 1 1
+ + ≤ + + . (12.7)
1+a+b 1+b+c 1+c+a 2+a 2+b 2+c

 Prueba. Sean x = a + b + c, y = ab + bc + ca, como abc = 1, entonces

x=a+b+c≥3
x ≥ 3; y ≥ 3
x2 ≥ 3y

pues (a + b + c)2 ≥ 3(ab + ac + bc).


La desigualdad (12.7) es equivalente a:

(1 + b + c)(1 + c + a) + (1 + a + b)(1 + c + a) + (1 + a + b)(1 + b + c)


(1 + a + b)(1 + b + c)(1 + c + a)
(2 + b)(2 + c) + (2 + a)(2 + c) + (2 + a)(2 + b)

(2 + a)(2 + b)(2 + c)
176 12. Soluciones

(a + b + c)2 + 4(a + b + c) + (ab + ac + bc) + 3



(a + b + c)2 + 2(a + b + c) + (ab + ac + bc) + (a + b + c)(ab + ac + bc)
12 + 4(a + b + c) + (ab + ac + bc)

9 + 4(a + b + c) + 2(ab + ac + bc)

Reemplazando tenemos

x2 + 4x + y + 3 12 + 4x + y
2

x + 2x + y + xy 9 + 4x + 2y
x2 + 4x + y + 3 12 + 4x + y
⇔ 2
−1≤ −1
x + 2x + y + xy 9 + 4x + 2y
2x − xy + 3 3−y
⇔ ≤ −1
x2 + 2x + y + xy 9 + 4x + 2y
⇔ 5x2 + 24x + y 2 + 3y + 27 ≤ 6xy + 3x2 y + xy 2
5 1
⇔ 5x2 + 24x + y 2 + 3y + 27 ≤ 5xy + xy + x2 y + x2 y + x2 y + xy 2
3 3

esta desigualdad es verdadera, pues

5 1
5x2 ≤ x2 y; y 2 ≤ x2 y; 9x ≤ xy 2
3 3
15x ≤ 5xy; 3y ≤ xy; 27 ≤ x2 y.

35. (IMO 2005). Dados a, b, c números reales positivos que satisfacen abc ≥ 1,
pruebe que

a5 − a2 b5 − b2 c5 − c2
+ + ≥ 0.
a5 + b2 + c2 b5 + c2 + a2 c5 + a2 + b2
177

 Prueba. La desigualdad es equivalente a:

     
a5 − a2 b5 − b2 c5 − c2
−1 + 5 −1 + 5 − 1 ≥ −3
a5 + b2 + c2 b + c2 + a2 c + a2 + b2
(a2 + b2 + c2 ) (a2 + b2 + c2 ) (a2 + b2 + c2 )
⇔ − − 5 − 5 ≥ −3
a5 + b2 + c2 b + c2 + a2 c + a2 + b2
(a2 + b2 + c2 ) (a2 + b2 + c2 ) (a2 + b2 + c2 )
⇔ + 5 + 5 ≤3
a5 + b2 + c2 b + c2 + a2 c + a2 + b2

La dificultad se presenta en los denominadores, por la potencia quinta, por ello


es conveniente deshacernos de dicha potencia, en efecto, aplicando Cauchy-
Schwarz.
2
(a5 + b2 + c2 ) (bc + b2 + c2 ) ≥ a5/2 b1/2 c1/2 + b2 + c2
2
= (abc)1/2 · a2 + b2 + c2
2
≥ (a2 + b2 + c2 ) ; pues abc ≥ 1

de donde

2
(a5 + b2 + c2 ) (bc + b2 + c2 ) ≥ (a2 + b2 + c2 )
bc + b2 + c2 a2 + b2 + c2
⇔ ≥
a2 + b2 + c2 a5 + b2 + c2
a2 + b2 + c2 bc + b2 + c2
⇔ 5 ≤ 2 ; similarmente
a + b2 + c2 a + b2 + c2
a2 + b2 + c2 ac + a2 + c2
≤ 2 ;
b5 + c2 + a2 a + b2 + c2
a2 + b2 + c2 ab + a2 + b2
≤ 2
c5 + a2 + b2 a + b2 + c2

sumando miembro a miembro tenemos:

a2 + b2 + c2 a2 + b2 + c2 a2 + b2 + c2
+ +
a5 + b2 + c2 b5 + c2 + a2 c5 + a2 + b2
bc + ac + ab + 2 (a2 + b2 + c2 ) 3 (a2 + b2 + c2 )
≤ ≤ = 3.
a2 + b2 + c2 a2 + b2 + c2
178 12. Soluciones

Recordar que

ab + ac + bc ≤ a2 + b2 + c2 .

La igualdad ocurre si y sólo si a = b = c. 2

36. Sean a, b, c números reales tales que a + b + c = 3, pruebe que

√ √ √
a+ b+ c ≥ ab + bc + ca.

 Prueba. Como 2(ab + bc + ca) = (a + b + c)2 − (a2 + b2 + c2 ) entonces la


desigualdad es equivalente a:

√ √ √
a2 + b2 + c2 + 2( a + b + c) ≥ (a b + }c)2 = 9
| +{z
3

agrupando convenientemente en el primer miembro

√   2 √  √ 
a + 2 a + b + 2 b + c2 + 2 c ≥ 9.
2

Pero
√ √ √ p √ √
a2 + 2 a = a2 + a + a ≥ 3 3 a2 a a = 3a

entonces

a2 + 2 a ≥ 3a; similarmente

b2 + 2 b ≥ 3b

c2 + 2 c ≥ 3c

sumando miembro a miembro, tenemos:


√ √ √
a2 + b2 + c2 + 2( a + b + c) ≥ 3(a + b + c)
√ √ √
a2 + b2 + c2 + 2( a + b + c) ≥ 9.
2
179

37. (APMO 1998). Sean x, y, z números reales positivos, pruebe que


 
x  y z 2(x + y + z)
1+ 1+ 1+ ≥2+ √ .
y z x 3 xyz

 Prueba. Efectuando en el primer miembro de la desigualdad

x y z x z y 2(x + y + z)
1+ + + + + + +1≥2+ √
y z x z y x 3 xyz

simplificando

x y z x z y 2(x + y + z)
+ + + + + ≥ √
y z x z y x 3 xyz

Aplicando MA ≥ MG convenientemente
s s
2
x y x x y x x3 3x
2 + = + + ≥33 =33 = √
y z y y z yz xyz 3 xyz
x y 3x
⇒2 + ≥ √ ; similarmente
y z 3 xyz
y z 3y
2 + ≥ √
z x 3 xyz

z x 3z
2 + ≥ √
x y 3 xyz

sumando las tres desigualdades:


   
x y z x y z
3 + + ≥ 3 √ +√ +√
y z x 3 xyz 3 xyz 3 xyz

x y z x+y+z
+ + ≥ √ , similarmente
y z x 3 xyz

x z y x+y+z
+ + ≥ √ .
z y x 3 xyz

Sumando miembro a miembro obtenemos la desigualdad que querı́amos de-


mostrar. 2
180 12. Soluciones

38. (USAMO 1998). Sean a, b, c números reales positivos, pruebe que

1 1 1 1
+ 3 + 3 ≤ .
a3 b3 3 3
+ + 3abc b + c + 3abc c + a + 3abc abc

 Prueba. Sabemos que

a2 + b2 ≥ 2ab, restando ab
⇔ a2 − ab + b2 ≥ ab, multiplicando por (a + b)
⇔ (a + b) (a2 − ab + b2 ) ≥ ab(a + b)
⇔ a3 + b3 ≥ ab(a + b), sumando abc
⇔ a3 + b3 + abc ≥ ab(a + b + c)
1 1
⇔ ≤
a3 3
+ b + abc ab(a + b + c)
abc abc
⇔ ≤
a3 3
+ b + abc ab(a + b + c)
abc c
⇔ ≤ , similarmente
a3 3
+ b + abc a+b+c
abc a

b3 + c3 + abc a+b+c
abc b
3 3

c + a + abc a+b+c

sumando miembro a miembro:

abc abc abc


+ 3 + 3 ≤1
a3 + b + 3abc b + c + 3abc c + a3 + 3abc
3 3

de donde
1 1 1 1
+ 3 + 3 ≤ .
a3 b3 3 3
+ + 3abc b + c + 3abc c + a + 3abc abc

2
181

39. Sean a, b, c reales positivos con suma 3, pruebe que

a b c 3
+ + ≥ .
1 + b2 1 + c2 1 + a2 2

 Prueba. Antes de dar la prueba, notemos que la MA ≥ MG en algunos


casos no ayuda a solucionar el problema. Veamos:

a b c a b c
2
+ 2
+ 2
≤ + +
1+b 1+c 1+a 2b 2c 2a

a b c 3
Pero aplicando MA ≥ MG, tenemos + + ≥ , y no podemos
2b 2c 2a 2
a b c 3
garantizar que + + ≥ .
1 + b2 1 + c2 1 + a2 2
Por ello utilizaremos otro procedimiento, veamos:

a ab2 ab2 ab
=a− ≥ a − =a− ,
1 + b2 1+b 2 2b 2
a ab
⇒ ≥ a − , similarmente
1 + b2 2
b bc
≥ b− ,
1 + c2 2
c ca
≥c− ,
1 + a2 2

sumando miembro a miembro, tenemos


 
a b c ab bc ac
2
+ 2
+ ≥ a+b+c− + +
1+b 1+c 1 + a2 2 2 2
1
= 3 − (ab + bc + ac)
2

luego es suficiente demostrar que

1 3
3 − (ab + bc + ac) ≥ .
2 2
182 12. Soluciones

Sabemos que

(a + b + c)2 ≥ 3(ab + bc + ac)


32 ≥ 3(ab + bc + ac)
3 ≥ ab + bc + ac
1 3
− (ab + bc + ac) ≥ −
2 2
1 3 3
3 − (ab + bc + ac) ≥ 3 − = . [9, pág 27]
2 2 2
2

40. Sean a, b, c, d números reales positivos, pruebe que

a3 b3 c3 d3 a+b+c+d
2 2
+ 2 2
+ 2 2
+ 2 2
≥ .
a +b b +c c +d d +a 2

 Prueba. La idea del problema anterior es interesante y la vamos a utilizar.


En efecto

a3 ab2
= a − .
a2 + b2 a2 + b2

Pero

1 1
a2 + b2 ≥ 2ab ⇔ ≤
a2
+b 2 2ab
2
−ab −ab2
⇔ 2 ≥
a + b2 2ab

entonces

a3 ab2 b
2 2
=a− 2 2
≥a−
a +b a +b 2
183

a3 b
⇒ 2 2
≥ a − , similarmente
a +b 2
3
b c
2 2
≥b−
b +c 2
3
c d
2 2
≥c−
c +d 2
3
d a
2 2
≥ d−
d +a 2

sumando miembro a miembro, tenemos

a3 b3 c3 d3 a+b+c+d
2 2
+ 2 2
+ 2 2
+ 2 2
≥ . [9, pág 28]
a +b b +c c +d d +a 2
2

41. Sean a, b, c, d números reales positivos tales que abcd = 1, pruebe que

1 1 1
S = + +
1 + ab + bc + ca 1 + bc + cd + ab 1 + cd + da + ac
1
+ ≤ 1.
1 + da + ab + bd

 Prueba. Como necesitamos formar ab + bc + ca y aprovechando abcd = 1


partimos de

1 1 1 1 1 1
+ + ≥ √ +√ +√
a b c ab ac bc
√ √ √
cd bd ad
= √ +√ +√
abcd abcd abcd
√ √ √
= cd + bd + ad
√ √ √ √
= d( c + b + a)
√ √ √ √
bc + ac + ab d( a + b + c)
⇒ ≥
abc · d d
√ √ √
a+ b+ c
⇒ bc + ac + ab ≥ √
d
184 12. Soluciones

√ √√ √
a+b+ c+ d
⇒ 1 + bc + ac + ab ≥ √
d

1 d
⇒ ≤ √ √ √ √ , similarmente
1 + bc + ac + ab a+ b+ c+ d

1 a
≤ √ √ √ √
1 + bc + cd + db a+ b+ c+ d

1 b
≤ √ √ √ √
1 + cd + da + ac a+ b+ c+ d

1 c
≤ √ √ √ √
1 + da + ab + bd a+ b+ c+ d

sumando miembro a miembro, tenemos

S ≤ 1.

42. Sean a, b, c números no negativos tales que a + b + c = 3, pruebe que

1 1 1
+ + ≥ 1.
2ab2 +1 2bc2 2
+ 1 2ca + 1

 Prueba. Efectuando tenemos la desigualdad equivalente:

(2bc2 + 1) (2ca2 + 1) + (2ab2 + 1) (2ca2 + 1) + (2ab2 + 1) (2bc2 + 1)


≥ (2ab2 + 1) (2bc2 + 1) (2ca2 + 1)
⇔ 1 + a2 c + b2 a + bc2 − 4(abc)3 ≥ 0

como

a2 c + b2 a + bc2 p
≥ 3 (abc)3
3
⇔ a c + b a + bc2 ≥ 3abc
2 2
185

luego

1 + a2 c + b2 a + bc2 − 4(abc)3 ≥ 1 + 3abc − 4(abc)3

basta demostrar

1 + 3abc − 4(abc)3 ≥ 0. (12.8)

Para ello observe que

a+b+c √
≥ 3 abc
3
⇒ 1 ≥ abc; pues a + b + c = 3
⇒ abc ≤ 1.

En (12.8) factorizando el primer miembro vemos que

(1 − abc)(2abc + 1)2 ≥ 0.

Esta desigualdad es verdadera debido a que abc ≤ 1. 2

43. Sean a, b, c, x, y, z números reales no negativos tales que a + b + c = x + y + z,


pruebe que

ax(a + x) + by(b + y) + cz(c + z) ≥ 3(abc + xyz).

 Prueba. La desigualdad que vamos a demostrar es equivalente a:

a2 x + ax2 + b2 y + by 2 + c2 z + cz 2 ≥ 3(abc + xyz)

aplicando la desigualdad de Cauchy-Schwarz a los números

√ √ √ √ √ √
a x, b y, c z, yz, zx, xy
186 12. Soluciones

en efecto se tiene:
√ √ √
(a2 x + b2 y + c2 z) (yz + zx + xy) ≥ (a xyz + b xyz + c xyz)2
(a2 x + b2 y + c2 z) (yz + zx + xy) ≥ xyz(a + b + c)2
(a2 x + b2 y + c2 z) (yz + zx + xy) ≥ xyz(x + y + z)2
≥ xyz(3(xy + xz + yz))
⇒ a2 x + b2 y + c2 z ≥ 3xyz; similarmente
ax2 + by 2 + cz 2 ≥ 3abc.

Sumando

ax(a + x) + by(b + y) + cz(c + z) ≥ 3(abc + xyz).

44. Sean a, b, c números reales positivos, pruebe que

a2 (b + c − a) b2 (c + a − b) c2 (a + b − c) ab + bc + ca
+ + ≤ .
b+c c+a a+b 2

 Prueba. Efectuando en el primer miembro obtenemos una desigualdad


equivalente
 
2 2 2 a3 b3 c3 ab + bc + ca
a +b +c − + + ≤
b+c c+a a+b 2
 4 4 4

a b c ab + bc + ca
⇔ a2 + b2 + c2 − + + ≤
ab + ac bc + ba ac + bc 2

Aplicando el Lema de Titu:

2
a4 b4 c4 (a2 + b2 + c2 )
+ + ≥
ab + ac bc + ba ac + bc 2(ab + ac + bc)
  2
a4 b4 c4 (a2 + b2 + c2 )
⇔ − + + ≤−
ab + ac bc + ba ac + bc 2(ab + ac + bc)
187

luego basta demostrar que


!
2
2 2 2 (a2 + b2 + c2 ) ab + bc + ca
a +b +c − ≤
2(ab + ac + bc) 2

En efecto equivale a:

2
2(ab + ac + bc) (a2 + b2 + c2 ) − (a2 + b2 + c2 ) ≤ (ab + ac + bc)2

haciendo a2 + b2 + c2 = x, ab + ac + bc = y, tenemos la desigualdad equivalente

2yx − x2 ≤ y 2

⇔ 0 ≤ x2 − 2xy + y 2

⇔ (x − y)2 ≥ 0.

45. Si a, b, c reales positivos, pruebe que


s s s
3 3
a b c3
+ + ≥ 1.
a + (b + c)3
3 b + (c + a)3
3 c3 + (a + b)3

 Prueba. Veamos para x > 0

√ p (1 + x) + (1 − x + x2 ) x2
1 + x3 = (1 + x) (1 − x + x2 ) ≤ =1+
2 2

√ x2
⇒ 1 + x3 ≤ 1 + .
2
188 12. Soluciones

Como
s v
a3 u 1 1
= u  3 = s
a3 + (b + c)3 ut b+c 
b+c
3
1+ 1+
a a

1 1 a2
≥  2 ≥ =
1 b+c b2 + c2 a2 + b2 + c2
1+ 1+
2 a a2

(hemos aplicado media cuadrática ≥ media aritmética)


luego
s
a3 a2
≥ ; en forma similar
a3 + (b + c)3 a2 + b2 + c2
s
b3 b2

b3 + (c + a)3 a2 + b2 + c2
s
c3 c2

c3 + (a + b)3 a2 + b2 + c2

sumando tenemos:
s s s
3 3
a b c3
+ + ≥ 1.
a + (b + c)3
3 b + (c + a)3
3 c3 + (a + b)3

La igualdad ocurre si y sólo si a = b = c. 2

46. Si a, b, c, d son números reales no negativos, pruebe que

a−b b−c c−d d−a


+ + + ≥ 0.
a + 2b + c b + 2c + d c + 2d + a d + 2a + b
189

1
 Prueba. Sumando a cada fracción tenemos la desigualdad equivalente
2
a la anterior
     
a−b 1 b−c 1 c−d 1
+ + + + +
a + 2b + c 2 b + 2c + d 2 c + 2d + a 2
 
d−a 1
+ + ≥2
d + 2a + b 2

efectuando

3a + c 3b + d 3c + a 3d + b
+ + + ≥4
a + 2b + c b + 2c + d c + 2d + a d + 2a + b

para aplicar el Lema de Titu, acondicionemos el numerador, en efecto

(3a + c)2 (3b + d)2 (3c + a)2


⇔ + +
(3a + c)(a + 2b + c) (3b + d)(b + 2c + d) (3c + a)(c + 2d + a)
(3d + b)2
+
(3d + b)(d + 2a + b)
(3a+c+3b+d+3c+a+3d+b)2
≥ (3a+c)(a+2b+c)+(3b+d)(b+2c+d)+(3c+a)(c+2d+a)+(3d+b)(d+2a+b)

16(a + b + c + d)2
= = 4.
4(a + b + c + d)2

47. Si a, b, c son números reales no negativos, entonces

√ √ √
a2 − bc b + c + b2 − ca c + a + c2 − ab a + b ≥ 0.

 Prueba. La desigualdad que vamos a demostrar es equivalente a

(b + c) (a2 − bc) (c + a) (b2 − ca) (a + b) (c2 − ab)


√ + √ + √ ≥0 (12.9)
b+c c+a a+b

Haciendo m = (b + c) (a2 − bc) ; n = (c + a) (b2 − ca) ; p = (a + b) (c2 − ab).


190 12. Soluciones

Efectuando y sumando obtenemos que m+n+p = 0, luego (12.9) es equivalente


a:

m n p
√ +√ +√ ≥ 0, con m + n + p = 0.
b+c c+a a+b

Sin pérdida de generalidad podemos considerar un orden a ≥ b ≥ c, de donde


se deduce que m ≥ 0 y p ≤ 0, y reemplazando n = −(m + p), tenemos

m (m + p) p
√ − √ +√ ≥0
b+c c+a a+b
   
1 1 1 1
⇔ m √ −√ +p √ −√ ≥0
b+c c+a a+b c+a
   
 1 1   1 1 
   
m √
⇔ |{z} −√  + (−p)  √ − √ ≥0
 b+c c + a | {z }  c+a b + a
(+) | {z } (+) | {z }
(+) (+)

48. Dados a, b, c números reales positivos, pruebe que


 2  2  2
a b c 3
+ + ≥ .
a+b b+c c+a 4

 Prueba. La desigualdad es equivalente a:


 2  2  2
 1  1 1  ≥3
 +  +
b 1+
c
1+
a 4
1+ b c
a

b c a
Haciendo = x, = y, = z, tenemos:
a b c
1 1 1 3
2
+ 2
+ 2
≥ , con xyz = 1.
(1 + x) (1 + y) (1 + z) 4
191

Veamos que

1 1 1
2
+ 2
≥ ; x, y > 0.
(1 + x) (1 + y) 1 + xy

Aplicando el resultado del problema 21 y aprovechando que xyz = 1, tenemos:

1 1 1 z z
+ ≥ = =
(1 + x)2 (1 + y)2 1 + xy z + xyz z+1
1 1 1 z 1
⇒ 2
+ 2
+ 2
≥ +
(1 + x) (1 + y) (1 + z) z + 1 (1 + z)2
z2 + z + 1
=
(1 + z)2
(z 2 + 2z + 1) − z
=
(1 + z)2
z
= 1− 2
z + 2z + 1
1
= 1−
1
z+ +2
z

acotando el segundo miembro:


1 1
z+ ≥2⇔z+ +2≥4
z z
1 1
⇔ 0< ≤
1 4
z+ +2
z
1 1
⇔ 0>− ≥−
1 4
z+ +2
z
1 3
⇔ 1>1− ≥
1 4
z+ +2
z

1 1 1 3
∴ 2
+ 2
+ 2
≥ .
(1 + x) (1 + y) (1 + z) 4
2
192 12. Soluciones

49. Sean a, b, c, d números reales positivos que satisfacen

1 1 1 1
+ + + = 1,
1+a 1+b 1+c 1+d

pruebe que abcd ≥ 81.

 Prueba. Haciendo

1 1−x
= x ⇒ a =
1+a x
1 1−y
= y ⇒ b =
1+b y
1 1−z
= z ⇒ c =
1+c z
1 1−w
= w ⇒ d =
1+d w
entonces tenemos que

x+y+z+w =1

y demostraremos que
    
1−x 1−y 1−z 1−w
≥ 81
x y z w
⇔ (y + z + w)(x + z + w)(x + y + w)(x + y + z) ≥ 81xyzw

como

y + z + w ≥ 3 3 yzw

x + z + w ≥ 3 3 xzw

x + y + w ≥ 3 3 xyw

x + y + z ≥ 3 3 xyz
p
⇒ (y + z + w)(x + z + w)(x + y + w)(x + y + z) ≥ 81 3 (xyzw)3
(y + z + w)(x + z + w)(x + y + w)(x + y + z) ≥ 81xyzw.
2
193

50. Sean a, b, c los lados de un triángulo, pruebe que

a b c
+ + ≥ 1.
3a − b + c 3b − c + a 3c − a + b

 Prueba.

a b c
S = + +
3a − b + c 3b − c + a 3c − a + b
4a 4b 4c
4S = + +
3a − b + c 3b − c + a 3c − a + b
a+b−c b+c−a c+a−b
4S = 1 + +1+ +1+
3a − b + c 3b − c + a 3c − a + b
(a + b − c)2 (b + c − a)2
4S = 3 + +
(a + b − c)(3a − b + c) (b + c − a)(3b − c + a)
(c + a − b)2
+
(c + a − b)(3c − a + b)

vemos que (a+b−c)(3a−b+c), (b+c−a)(3b−c+a), (c+a−b)(3c−a+b) ≥ 0,


pues a, b, c son los lados de un triángulo. Luego, aplicando el Lema de Titu,
tenemos

(a+b−c+b+c−a+c+a−b)2
4S ≥ 3 + (a+b−c)(3a−b+c)+(b+c−a)(3b−c+a)+(c+a−b)(3c−a+b)

(a + b + c)2 (a + b + c)2
= 3+ = 3 +
a2 + b2 + c2 + 2(ab + ac + bc) (a + b + c)2

= 3.

Luego

4S ≥ 4

∴ S ≥ 1.

2
194 12. Soluciones

51. Dados a, b, c, d números reales positivos, pruebe que


21
a a 16

3
≥ 21 21 21 21 .
3
a + 63bcd a 16 + b 16 + c 16 + d 16

 Prueba. Como

(x + y + z + t)3 = x3 + y 3 + z 3 + t3 + 3x2 (y + z + t)
+3y 2 (x + z + t) + 3z 2 (x + y + t)
+3t2 (x + y + z) + 6(xyz + xyt + xzt + yzt)

luego

(x + y + z + t)3 = x3 + y 3 + z 3 + t3 + 3x2 y + 3x2 z + 3x2 t


+3y 2 x + 3y 2 z + 3y 2 t + 3z 2 x + 3z 2 y + 3z 2 t
+3t2 x + 3t2 y + 3t2 z + 6xyz + 6xyt + 6xzt + 6yzt
p
≥ x3 + 63 63 x45 y 48 z 48 t48
15 16 16 16
= x3 + 63x 21 · y 21 · z 21 · t 21
q
3 15 16 16 16
(x + y + z + t) ≥ x3 + 63x 21 · y 21 · z 21 · t 21 luego
r  
5 16 16 16 16
= 3 x 7 x 7 + 63y 21 · z 21 · t 21
r 
5 16 3 16 16 16
3
= x 21 · x 21 + 63y 21 · z 21 · t 21

16 16 16 16
Haciendo x 21 = a, y 21 = b, z 21 = c, t 21 = d.
Tenemos
21 21 21 21 5 √
3
a 16 + b 16 + c 16 + d 16 ≥ a 16 · a3 + 63bcd

de donde
21
a a 16

3
≥ 21 21 21 21 .
3
a + 63bcd a 16 + b 16 + c 16 + d 16
2
195

52. Dados a, b, c, d números reales positivos, pruebe que

a b c d
A= √
3
+√
3
+√
3
+√
3
≥ 1.
a3 + 63bcd b3 + 63acd c3 + 63bda d3 + 63abc

 Prueba. Del problema 51 tenemos que

21
a a 16

3
≥ 21 21 21 21
a3 + 63bcd a 16 + b 16 + c 16 + d 16
21
b b 16

3 3
≥ 21 21 21 21
b + 63acd a 16 + b 16 + c 16 + d 16
21
c c 16

3 3
≥ 21 21 21 21
c + 63adb a 16 + b 16 + c 16 + d 16
21
d d 16

3
≥ 21 21 21 21
3
d + 63abc a 16 + b 16 + c 16 + d 16

sumando obtenemos

A ≥ 1.

53. Sean a, b, c los lados de un triángulo, pruebe que


r
3 a3 + b3 + c3 + 3abc
≥ máx {a, b, c}.
2

 Prueba. Sin pérdida de generalidad, sea a = máx {a, b, c}, entonces debe-
mos probar
r
3 a3 + b3 + c3 + 3abc
≥a
2
196 12. Soluciones

En efecto tenemos

a3 + b3 + c3 + 3abc ≥ 2a3
⇔ b3 + c3 − a3 + 3abc ≥ 0
⇔ b3 + c3 + (−a)3 + 3abc ≥ 0, (Identidad de Gauss)
⇔ (b + c − a)(b2 + c2 + a2 + bc − ab − ac) ≥ 0
1
⇔ (b + c − a)(2b2 + 2c2 + 2a2 + 2bc − 2ab − 2ac) ≥ 0
2
1
⇔ c − a})((a − b)2 + (a − c)2 + (b − c)2 ) ≥ 0
(b| + {z
2
(+)

pues a, b, c son los lados de un triángulo, por lo tanto está última desigualdad
es verdadera. 2

54. Sean a, b, c números reales positivos, pruebe que

(a + b)2 (b + c)2 (c + a)2


S= + + > 2.
a2 + b2 + c2 + ab a2 + b2 + c2 + bc a2 + b2 + c2 + ca

 Prueba. Vemos que

(a + b)2 (a + b)2
>
a2 + b2 + c2 + ab a2 + b2 + c2 + ab + ac + bc
2
(b + c) (b + c)2
>
a2 + b2 + c2 + bc a2 + b2 + c2 + ab + ac + bc
2
(c + a) (c + a)2
>
a2 + b2 + c2 + ca a2 + b2 + c2 + ab + ac + bc

sumando miembro a miembro se tiene:

2 (a2 + b2 + c2 + ab + ac + bc)
S> = 2.
a2 + b2 + c2 + ab + ac + bc
2
197

55. Dados a, b, c números reales tales que a, b, c ∈ [0; 1], pruebe que

a b c
+ + + (1 − a)(1 − b)(1 − c) ≤ 1.
b+c+1 c+a+1 a+b+1

 Prueba. Por la simetrı́a, sin pérdida de generalidad podemos tomar a ≥


b ≥ c. Primero veamos el caso cuando a 6= 1. Luego, como (1−a), (1−b), (1−c)
son números positivos, aplicamos MA ≥ MG, de la siguiente manera

(b + c + 1) + (1 − b) + (1 − c) p
≥ 3 (b + c + 1)(1 − b)(1 − c)
3
⇒ (b + c + 1)(1 − b)(1 − c) ≤ 1
1
⇒ (1 − b)(1 − c) ≤
b+c+1
1−a
⇒ (1 − a)(1 − b)(1 − c) ≤
b+c+1
a 1
⇒ + (1 − a)(1 − b)(1 − c) ≤ , y además
b+c+1 b+c+1
b b

c+a+1 c+b+1
c c

a+b+1 c+b+1

sumando miembro a miembro

a b c
+ + + (1 − a)(1 − b)(1 − c) ≤ 1.
b+c+1 c+a+1 a+b+1

En este caso la igualdad ocurre si y sólo si a = b = c = 0.


Ahora veamos el caso cuando a = 1. Se tiene

a 1
= ,
b+c+1 b+c+1
b b
≤ ,
c+2 c+b+1
c c
≤ ,
b+2 c+b+1
198 12. Soluciones

sumando miembro a miembro, obtenemos

a b c
+ + ≤ 1.
b+c+1 c+a+1 a+b+1

En este caso la igualdad ocurre si y sólo si (a, b, c) = (1, 1, 1), (a, b, c) = (1, 1, 0),
(a, b, c) = (1, 0, 0), o sus respectivas permutaciones. 2

56. Sean a, b, c números reales positivos, pruebe que

a b c 3
+ + ≤ .
2a + b + c 2b + c + a 2c + a + b 4

 Prueba. La desigualdad a demostrar es equivalente a


 
a+b+c a+b+c a+b+c 3
− + + ≤ −3
2a + b + c 2b + c + a 2c + a + b 4
a+b+c a+b+c a+b+c 9
⇔ + + ≥
2a + b + c 2b + c + a 2c + a + b 4
1 1 1
⇔ 4(a + b + c) + + ≥ 9.
2a + b + c 2b + c + a 2c + a + b

Haciendo 2a + b + c = x, 2b + c + a = y, 2c + a + b = z, entonces 4(a + b + c) =


x + y + z, luego tenemos que la desigualdad anterior es equivalente a:
 
1 1 1
(x + y + z) + + ≥ 9,
x y z

pero esta desigualdad es verdadera. 2

57. Sean a, b, c números reales tales que a2 + b2 + c2 = 1, pruebe que

√ √ √ √
1 − ab + 1 − bc + 1 − ca ≥ 6. (12.10)
199

 Prueba. Elevando al cuadrado tenemos el equivalente de la desigualdad


(12.10)

(1 − ab) + (1 − bc) + (1 − ca)


hp p p i
+2 (1 − ab)(1 − bc) + (1 − ab)(1 − ca) + (1 − bc)(1 − ca) ≥ 6

usando el dato a2 + b2 + c2 = 1;

1 2
1 − ab = [a + b2 + c2 + 1] − ab
2
1 2 1
= (c + 1) + (a2 + b2 − 2ab)
2 2
1 2 1 1
= (c + 1) + (a − b)2 ≥ (c2 + 1)
2 2 2
1 2
⇒ 1 − ab ≥ (c + 1) ; similarmente
2
1 2
1 − bc ≥ (a + 1)
2
1 2
1 − ca ≥ (b + 1)
2

multiplicando dos a dos tenemos:

1 2 1
(1 − ab)(1 − bc) ≥ (a + 1) (c2 + 1) ≥ (ac + 1)2
4 4
p 1
⇒ (1 − ab)(1 − bc) ≥ (ac + 1); similarmente
2
p 1
(1 − ab)(1 − ac) ≥ (bc + 1)
2
p 1
(1 − bc)(1 − ac) ≥ (ab + 1)
2
hp p p i
⇒ 2 (1 − ab)(1 − bc) + (1 − ab)(1 − ca) + (1 − bc)(1 − ca)
≥ ab + bc + ca + 3
200 12. Soluciones

∴ (1 − ab) + (1 − bc) + (1 − ca)


hp p p i
+2 (1 − ab)(1 − bc) + (1 − ab)(1 − ca) + (1 − bc)(1 − ca) ≥ 6.

58. Pruebe que

a4 + b4 + c4 + abc(a + b + c) ≥ ab(a2 + b2 ) + bc(b2 + c2 ) + ca(c2 + a2 ), (12.11)

se cumple para todo a, b, c números reales.

 Prueba. Sin pérdida de generalidad, podemos suponer a ≥ b ≥ c, por la


simetrı́a de las expresiones.
La desigualdad (12.11) es equivalente a:

a4 + b4 + c4 + abc(a + b + c) − ab(a2 + b2 ) − bc(b2 + c2 ) − ca(c2 + a2 ) ≥ 0

⇔ a4 − a3 (b + c) + a2 bc − b3 (c + a) + b2 ca − c3 (a + b) + c2 ab ≥ 0

⇔ a2 (a2 − a(b + c) + bc) + b2 (b2 − b(c + a) + bc) + c2 (c2 − c(a + b) + ab) ≥ 0

⇔ a2 (a − b)(a − c) + b2 (b − a)(b − c) + c2 (c − a)(c − b) ≥ 0,

esta desigualdad es verdadera, pues es la desigualdad de Schür. 2

59. Pruebe que

a3 + b3 + c3 + 4(a + b + c) + 9abc ≥ 8(ab + bc + ca),

se cumple para todo a, b, c números reales no negativos.


201

(ab + bc + ca)2
 Prueba. Aplicando MA ≥ MG a los números (a+b+c), ,
a+b+c
tenemos

(ab + bc + ca)2 s  
(a + b + c) + (ab + bc + ca)2
a+b+c ≥ (a + b + c)
2 a+b+c
4(ab + bc + ca)2
⇔ 4(a + b + c) + ≥ 8(ab + bc + ca).
a+b+c

Luego basta demostrar que

4(ab + bc + ca)2
a3 + b3 + c3 + 4(a + b + c) + 9abc ≥ 4(a + b + c) +
a+b+c
4(ab + bc + ca)2
⇔ a3 + b3 + c3 + 9abc ≥
a+b+c
⇔ a + ab + ac + ba + b + bc3 + ca3 + cb3 + c4 + 9abc(a + b + c)
4 3 3 3 4

≥ 4 (a2 b2 + b2 c2 + c2 a2 + 2abc(a + b + c))


⇔ a4 + b4 + c4 + abc(a + b + c) + ab(a2 + b2 ) + ac(a2 + c2 ) + bc(b2 + c2 )
≥ 4 (a2 b2 + b2 c2 + c2 a2 )

Por el problema anterior (problema 58), sabemos que

a4 + b4 + c4 + abc(a + b + c) ≥ ab(a2 + b2 ) + ac(a2 + c2 ) + bc(b2 + c2 )


⇒ a4 + b4 + c4 + abc(a + b + c) + ab(a2 + b2 ) + ac(a2 + c2 ) + bc(b2 + c2 )
≥ 2 [ab(a2 + b2 ) + ac(a2 + c2 ) + bc(b2 + c2 )] ≥ 4 [a2 b2 + b2 c2 + c2 a2 ]

La igualdad ocurre si y sólo si

(a, b, c) = (2, 2, 2) ∨ (a, b, c) = (2, 2, 0) o sus respectivas permutaciones.

2
202 12. Soluciones

60. Pruebe que la desigualdad



p p p 3 2
M= a2 + (1 − b)2 + b2 + (1 − c)2 + c2 + (1 − a)2 ≥ ,
2

se cumple para todo a, b, c números reales.

 Prueba. Aplicando la desigualdad de Minskowsky.


p p p
M = a2 + (1 − b)2 + b2 + (1 − c)2 + c2 + (1 − a)2
p
≥ (a + b + c)2 + (1 − b + 1 − c + 1 − a)2
p
= (a + b + c)2 + (3 − (a + b + c))2

Haciendo a + b + c = x.
Entonces
p √
M≥ x2 + (3 − x)2 = 2x2 − 6x + 9
s  
2
9 9
= 2 x − 3x + +
4 2
s  2
3 9
= 2 x− +
2 2
r √
9 3 2
≥ =
2 2

3 2
∴ M≥ .
2
2

61. Pruebe que


s s s s
3 3 3
a b c d3
3
+ 3
+ 3
+ ≥ 1,
a + 15bcd b + 15acd c + 15abd d3 + 15abc

se cumple para todo a, b, c números reales positivos.


203

 Prueba. Efectuando

(m + n + p + q)2
= m2 + n2 + p2 + q 2 + 2(mn + mp + mq + np + nq + pq)
= m2 + n2 + p2 + q 2 + mn + mn + mp + mp + · · · + pq + pq)
| {z }
p
≥ m2 + 15 15 m6 n8 p8 q 8
= m2 + 15m6/15 n8/15 p8/15 q 8/15
⇒ m+n+p+q
p
≥ m2 + 15m6/15 · n8/15 · p8/15 · q 8/15
r  24 
6
= m 15 m + 15n
15 8/15 ·p 8/15 ·q 8/15

r 
1 8 3
= m5 m 15 + 15n8/15 · p8/15 · q 8/15

⇒ m+n+p+q
r 3
1 8
≥ m5 m 15 + 15n8/15 · p8/15 · q 8/15

Haciendo

m8/15 = a; p8/15 = c
n8/15 = b; q 8/15 = d.

Tenemos:


a15/8 + b15/8 + c15/8 + d15/8 ≥ a3/8 a3 + 15bcd
a3/2 a15/8
⇒ √ ≥ 15/8
a3 + 15bcd a + b15/8 + c15/8 + d15/8
204 12. Soluciones

r
a3 a15/8
⇒ ≥ ; similarmente
a3 + 15bcd a15/8 + b15/8 + c15/8 + d15/8
r
b3 b15/8

b3 + 15acd a15/8 + b15/8 + c15/8 + d15/8
r
c3 c15/8

c3 + 15abd a15/8 + b15/8 + c15/8 + d15/8
r
d3 d15/8

d3 + 15abc a15/8 + b15/8 + c15/8 + d15/8

sumando miembro a miembro:


s s s s
a3 b3 c3 d3
+ + + ≥ 1,
a3 + 15bcd b3 + 15acd c3 + 15abd d3 + 15abc
2

62. Sean a, b, c números reales no negativos tales que ab + bc + ca = 3, pruebe que

1 1 1
+ 2 + 2 ≤ 1.
a2 +2 b +2 c +2

 Prueba. La desigualdad que demostraremos es equivalente a:

(b2 + 2) (c2 + 2) + (a2 + 2) (c2 + 2) + (a2 + 2) (b2 + 2)


≤ (a2 + 2) (b2 + 2) (c2 + 2)
⇔ a2 b2 c2 + a2 b2 + a2 c2 + b2 c2 − 4 ≥ 0

Haciendo ab = x, ac = y, bc = z, tenemos que demostrar

xyz + x2 + y 2 + z 2 − 4 ≥ 0, sabiendo que x + y + z = 3


⇔ x2 + y 2 + 2yz + z 2 + xyz − 2yz − 4 ≥ 0
⇔ x2 + (y + z)2 + yz(x − 2) − 4 ≥ 0
 
2 2 2−x
⇔ x + (y + z) − 4yz −4≥0
4
205

Como:

   
2 2 2−x 2 2 2 2−x
x + (3 − x) − 4yz − 4 ≥ x + (3 − x) − (y + z) −4
4 4
 
2 2 2 2−x
= x + (3 − x) − (3 − x) −4
4

basta demostrar que


 
2−x
x2 + (3 − x)2 − (3 − x)2 −4 ≥0
4
 
2 2 x−2
⇔ x + (x − 3) + (x − 3)2 − 4 ≥ 0
4
⇔ x3 − 3x + 2 ≥ 0
⇔ (x − 1)2 (x + 2) ≥ 0

La igualdad ocurre si y sólo si a = b = c = 1. 2

63. Sean a, b, c números reales positivos, pruebe que

a3 b3 c3 3 2  1
+ + ≥ a + b2 + c2 − (ab + bc + ca).
a+b b+c c+a 4 4

 Prueba. Aplicando MA ≥ MG:

a3 a(a + b) r
+ a3 a(a + b)
a+b 4 ≥ ·
2 a+b 4
a3 a2 + ab
+ ≥ a2
a+b 4
a3 3 2 1
≥ a − ab; similarmente
a+b 4 4
b3 3 2 1
≥ b − bc
b+c 4 4
c3 3 2 1
≥ c − ca
c+a 4 4
206 12. Soluciones

sumando tenemos:

a3 b3 c3 3 2  1
+ + ≥ a + b2 + c2 − (ab + bc + ca).
a+b b+c c+a 4 4
2

64. Sean a, b, c números reales positivos, pruebe que

ab bc ca 1
S= + + ≤ (a + b + c).
2c + a + b 2a + b + c 2b + c + a 4

 Prueba. Sean m, n ∈ R+ , entonces

4mn ≤ (m + n)2
4 m+n
⇔ ≤
m+n mn
4 1 1
⇔ ≤ +
m+n m n

Aplicando dicho resultado en el ejercicio tenemos:


4 1 1
≤ +
(c + a) + (c + b) c+a c+b
4ab ab ab
⇔ ≤ + ; similarmente
(c + a) + (c + b) c+a c+b
4bc bc bc
≤ +
(a + b) + (a + c) a+b a+c
4ca ca ca
≤ +
(b + c) + (b + a) b+c b+a

sumando miembro a miembro:


b(a + c) a(c + b) c(a + b)
4S ≤ + + =a+b+c
a+c c+b a+b
⇒ 4S ≤ a + b + c

1
∴ S ≤ (a + b + c).
4
2
207

65. Si a ≥ b ≥ c ≥ d ≥ e, entonces

(a + b + c + d + e)2 ≥ 8(ac + bd + ce).

¿Cuándo ocurre la igualdad?

 Prueba. Veamos

(a + b + c + d + e)2 − 8(ac + bd + ce)


= (a + b + c + d + e)2 − 8c(a + b + c + d + e) + 16c2 +8c(a + b + c + d + e)
| {z }
−8(ac + bd + ce) − 16c2
= (a + b + c + d + e − 4c)2 + 8c(a + b + c + d + e) − 8(ac + bd + ce) − 16c2
= (a + b + d + e − 3c)2 + 8(bc − c2 + cd − bd)
= (a + b + d + e − 3c)2 + 8[b(c − d) + c(d − c)]
= (a + b + d + e − 3c)2 + 8(c − d)(b − c) ≥ 0

pues c ≥ d ∧ b ≥ c.
La igualdad ocurre si

a+d+e a+b+e
b=c= ∨ c=d= .
2 2
2

66. Sean x, y, z números reales positivos tales que xyz ≥ xy + yz + zx.


Pruebe que:

xyz ≥ 3(x + y + z).

 Prueba. De xyz ≥ xy + yz + zx tenemos:

1 1 1
1≥ + +
z x y
208 12. Soluciones

1 1 1
Haciendo = a, = b, = c
x y z

⇒ a + b + c ≤ 1,

en seguida basta probar que


 
1 1 1 1
≥3 + +
abc a b c
⇔ 1 ≥ 3(bc + ac + ab)

como

1 ≥ a + b + c ⇒ 1 ≥ (a + b + c)2
⇔ 1 ≥ a2 + b2 + c2 + 2(ab + bc + ca) ≥ 3(ab + bc + ca)
⇔ 1 ≥ 3(ab + bc + ca).
2

67. Sean x, y números reales positivos tales que x + y = 1, pruebe que

x y 2
√ +p ≥√ .
1 − x2 1 − y2 3

 Prueba. Del Lema de Titu, se tiene que

x y x2 y2 (x + y)2
√ +p = √ + p ≥ √ p
1 − x2 1 − y2 x 1 − x2 y 1 − y 2 x 1 − x2 + y 1 − y 2
1
= √ p
x 1 − x2 + y 1 − y 2

luego es suficiente probar que


1 2 √ p 3
√ p ≥ √ ⇔ x 1−x +y 1−y ≤
2 2
x 1 − x2 + y 1 − y 2 3 2

√ √ √ p 3
⇔ x x − x3 + y y − y 3 ≤
2
209

aplicando Cauchy-Schwarz

√ √ √ p 2 √ √ 2  √ 2 p 2

3 3 2 3 3
x x−x + y y−y ≤ x + y x−x + y−y
= (x + y) (x + y − (x3 + y 3 ))
= 1 − (x3 + y 3 )

Como x + y = 1, aplicando la Media Potencial, tenemos:


1
x3 + y 3 ≥
4
1
⇔ − (x3 + y 3 ) ≤ −
4
3
⇔ 1 − (x3 + y 3 ) ≤
4
entonces
√ √ √ p 2 3
x x − x3 + y y − y 3 ≤
4
de donde

√ p 3
x 1 − x2 + y 1 − y 2 ≤ .
2
2

68. Sean a1 , a2 , . . . , an , b1 , b2 , . . . , bn números reales positivos, tales que a1 + a2 +


· · · + an = b1 + b2 + · · · + bn . Pruebe que

a21 a22 a2n 1


+ +···+ ≥ (a1 + a2 + · · · + an ).
a1 + b1 a2 + b2 an + bn 2

 Prueba. Aplicando el Lema de Titu, tenemos

a21 a22 a2n


+ +···+
a1 + b1 a2 + b2 an + bn
(a1 + a2 + · · · + an )2

(a1 + a2 + · · · + an ) + (b1 + b2 + · · · + bn )
210 12. Soluciones

a21 a22 a2n (a1 + a2 + · · · + an )2


⇔ + +···+ ≥
a1 + b1 a2 + b2 an + bn 2(a1 + a2 + · · · + an )
1
= (a1 + a2 + · · · + an )
2

de donde

a21 a22 a2n 1


+ + ···+ ≥ (a1 + a2 + · · · + an ).
a1 + b1 a2 + b2 an + bn 2
2

69. Sean a, b, c números reales positivos, pruebe que

2ab 2bc 2ca


+ + ≤ a + b + c.
a+b b+c c+a

 Prueba. Aplicando MH ≤ MA, tenemos:

2ab a+b

a+b 2
2bc b+c

b+c 2
2ca c+a
≤ ,
c+a 2

sumando miembro a miembro, tenemos

2ab 2bc 2ca


+ + ≤ a + b + c.
a+b b+c c+a
2

70. (Rusia 1992). Sean x, y, z números reales positivos, pruebe que


x4 + y 4 + z 2 ≥ 8xyz.
211

z2 z2
 Prueba. Aplicando MA ≥ MG a los números x4 , y 4, ,
2 2
r
4 z2 z2
4 4
4 4
z2 z2
x +y + + ≥ 4 x ·y · ·
2 2 2 2
xyz 4
⇔ x4 + y 4 + z 2 ≥ 4 · √4
= √ xyz
4 2

∴ x4 + y 4 + z 2 ≥ 8xyz.

71. Sean a, b, c números reales positivos con a + b + c = 1, pruebe que


   
1 1 1
+1 +1 +1 ≥ 64.
a b c

 Prueba. Efectuando el primer miembro


   
1 1 1 1 1 1 1 1 1 1
1+ 1+ 1+ =1+ + + + + + +
a b c a b c ab ac bc abc

como
r
1 1 1 1 3
+ + ≥ 33 = √
a b c abc 3
abc
r
1 1 1 1 3
+ + ≥ 33 2 2 2 = √ 2
ab ac bc abc 3
abc
 3
1 1
= √
abc 3
abc

luego

     3  3
1 1 1 3 3 1 1
1+ 1+ 1+ ≥ 1+ √ +√ 2+ √ = 1+ √
a b c 3
abc 3 abc 3
abc 3
abc
212 12. Soluciones

De la hipótesis a + b + c = 1, tenemos que

1 √3 1
≥ abc ⇔ √ ≥3
3 3
abc
1
⇔ 1+ √ 3
≥4
abc
 3
1
⇔ 1+ √3
≥ 64
abc

de donde
   
1 1 1
+1 +1 +1 ≥ 64.
a b c
2

72. Sean a, b, c números reales, y


√ √ √
x= b2 − bc + c2 ; y = c2 − ca + a2 ; z = a2 − ab + b2

pruebe que

xy + yz + zx ≥ a2 + b2 + c2 .

 Prueba. Escribiendo convenientemente:


r
√ c 2 3c2
x= b2 − bc + c2 = + b−
2 4
r 
√ c 2 3c2
y = c2 − ca + a2 = a− +
2 4

entonces, aplicando Cauchy-Schwarz


   
2 2 c 2 3c2 c 2 3c2
xy = b− + a− + ;
2 4 2 4
 2
c c  3c2
≥ b− a− +
2 2 4
213

ac bc
⇒ xy ≥ ab + c2 − − ; similarmente
2 2
ab bc
⇒ xz ≥ ac + b2 − −
2 2
ab ac
⇒ yz ≥ bc + a2 − −
2 2
sumando miembro a miembro, se tiene

xy + yz + zx ≥ a2 + b2 + c2 .

La igualdad ocurre si y sólo si a = b = c. 2

73. Sean x, y, z números reales positivos tales que x + y + z = 3, pruebe que

(1 − x)2 (1 − y)2 (1 − z)2


+ + ≥ 0.
1 − x4 1 − y4 1 − z4

 Prueba. La desigualdad es equivalente a:

1−x 1−y 1−z


2 3
+ 2 3
+ ≥0
1+x+x +x 1+y+y +y 1 + z + z2 + z3

Podemos tomar un orden x ≤ y ≤ z, entonces 1 − x ≥ 1 − y ≥ 1 − z y

1 1 1
≥ ≥
1 + x + x2 + x3 1 + y + y2 + y3 1 + z + z2 + z3

esto significa que podemos aplicar la desigualdad de Chebyshev

1−x 1−y 1−z


2 3
+ 2 3
+
1+x+x +x 1+y+y +y 1 + z + z2 + z3
 
1 1 1
≥ (1 − x + 1 − y + 1 − z) + +
1 + x + x2 + x3 1 + y + y 2 + y 3 1 + z + z 2 + z 3
 
1 1 1
= (3 − (x + y + z)) + +
| {z } 1 + x + x2 + x3 1 + y + y 2 + y 3 1 + z + z 2 + z 3
3

= 0.
214 12. Soluciones

1−x 1−y 1−z


∴ 2 3
+ 2 3
+ ≥ 0.
1+x+x +x 1+y+y +y 1 + z + z2 + z3
2

74. Si a1 , a2 , . . . , an son números reales, pruebe que

n X
X n
i · j · cos (ai − aj ) ≥ 0.
i=1 j=1

 Prueba.
n X
X n
i · j (cos ai cos aj + sen ai sen aj )
i=1 j=1
n X
X n
= (i · j cos ai cos aj + i · j sen ai sen aj )
i=1 j=1

X n
n X n X
X n
= ij cos ai cos aj + ij sen ai sen aj
i=1 j=1 i=1 j=1
n
X n
X n
X n
X
= i cos ai j cos aj + i sen ai j sen aj
i=1 j=1 i=1 j=1
| {z } | {z }
n
X Xn n
X Xn
= i cos ai i cos ai + i sen ai i sen ai
i=1 i=1 i=1 i=1
n
!2 n
!2
X X
= i cos ai + i sen ai ≥ 0.
i=1 i=1
2

75. Sean a, b, c números reales, tales que


√ √ √
x= b2 + bc + c2 ; y = c2 + ca + a2 ; z = a2 + ab + b2

pruebe que

xy + yz + zx ≥ (a + b + c)2 .
215

 Prueba. Utilizando la idea del problema 72 tenemos que

ac bc
xy ≥ ab + c2 + +
2 2
ab bc
xz ≥ ac + b2 + +
2 2
ab ac
yz ≥ bc + a2 + +
2 2

sumando miembro a miembro, tenemos

xy + yz + zx ≥ a2 + b2 + c2 + 2(ab + ac + bc)

∴ xy + yz + zx ≥ (a + b + c)2 .

76. Sean a, b, c, d, e, f números reales positivos, pruebe que

a b c d e f
+ + + + + ≥ 3.
b+c c+d d+e e+f f +a a+b

 Prueba. Para aplicar el Lema de Titu, el primer miembro de la desigual-


dad se expresa de la siguiente manera

a b c d e f
S = + + + + +
b+c c+d d+e e+f f +a a+b
a2 b2 c2 d2 e2 f2
= + + + + +
ab + ac bc + bd cd + ce ed + f d ef + ea af + bf
(a + b + c + d + e + f )2

ab + ac + bc + bd + cd + ce + ed + f d + ef + ea + af + bf
(a + b + c + d + e + f )2
=
M
216 12. Soluciones

donde

M = ab + ac + bc + bd + cd + ce + ed + f d + ef + ea + af + bf
2M = (a + b + c + d + e + f )2 − (a + d)2 − (b + e)2 − (c + f )2 (12.12)

como aparece (a + d)2 + (b + e)2 + (c + f )2, entonces aplicando Cauchy-Schwarz

 
(a + d)2 + (b + e)2 + (c + f )2 12 + 12 + 12 ≥ (a + b + c + d + e + f )2


3 (a + d)2 + (b + e)2 + (c + f )2 ≥ (a + b + c + d + e + f )2 (12.13)

De (12.12) tenemos

(a + d)2 + (b + e)2 + (c + f )2 = (a + b + c + d + e + f )2 − 2M
3 [(a + d)2 + (b + e)2 + (c + f )2 ] = 3(a + b + c + d + e + f )2 − 6M

luego de (12.13) se tiene que:

3(a + b + c + d + e + f )2 − 6M ≥ (a + b + c + d + e + f )2
2(a + b + c + d + e + f )2 ≥ 6M
(a + b + c + d + e + f )2
≥ 3.
M

a b c d e f
∴ + + + + + ≥ 3.
b+c c+d d+e e+f f +a a+b
2

77. Sean a, b, c, x, y, z, t, v, u números reales positivos, entonces

  
a3 + b3 + c3 x3 + y 3 + z 3 t3 + u3 + v 3 ≥ (axt + byu + czv)3 .
217

 Prueba. Observemos que

a3 b3 c3
+ + +
a3 + b3 + c3 a3 + b3 + c3 a3 + b3 + c3
x3 y3 z3
+ + +
x3 + y 3 + z 3 x3 + y 3 + z 3 x3 + y 3 + z 3
t3 u3 v3
+ + =3
t3 + u3 + v 3 t3 + u3 + v 3 t3 + u3 + v 3

luego, aplicando MA ≥ MG, tenemos que

a3 x3 t3
+ +
a3 + b3 + c3 x3 + y 3 + z 3 t3 + u3 + v 3
3axt
≥ p ,
3
(a3 + b3 + c3 ) (x3 + y 3 + z 3 ) (t3 + u3 + v 3 )
b3 y3 u3
+ +
a3 + b3 + c3 x3 + y 3 + z 3 t3 + u3 + v 3
3byu
≥ p ,
3
(a3 + b3 + c3 ) (x3 + y 3 + z 3 ) (t3 + u3 + v 3 )
c3 z3 v3
+ +
a3 + b3 + c3 x3 + y 3 + z 3 t3 + u3 + v 3
3czv
≥ p ,
3
(a + b + c ) (x + y 3 + z 3 ) (t3 + u3 + v 3 )
3 3 3 3

sumando miembro a miembro, tenemos

3axt + 3byu + 3czv


3≥ p
3
(a3 + b3 + c3 ) (x3 + y 3 + z 3 ) (t3 + u3 + v 3 )
p
⇔ 3
(a3 + b3 + c3 ) (x3 + y 3 + z 3 ) (t3 + u3 + v 3 ) ≥ axt + byu + czv
⇔ (a3 + b3 + c3 ) (x3 + y 3 + z 3 ) (t3 + u3 + v 3 ) ≥ (axt + byu + czv)3 .

A continuación veremos como una aplicación del problema 77 un problema


planteado en la IMO 2001.
218 12. Soluciones

78. (IMO 2001). Pruebe para a, b, c números reales positivos

a b c
√ +√ +√ ≥ 1.
a2 + 8bc b2 + 8ca c2 + 8ab

 Prueba. Aplicando el problema anterior:


 
a b c
√ +√ +√
a2 + 8bc b2 + 8ca c2 + 8ab
 
a b c
· √ +√ +√
a2 + 8bc b2 + 8ca c2 + 8ab
· [a (a2 + 8bc) + b (b2 + 8ca) + c (c2 + 8ca)] ≥ (a + b + c)3
 2
a b c
⇒ √ +√ +√
a2 + 8bc b2 + 8ca c2 + 8ab
· (a (a2 + 8bc) + b (b2 + 8ca) + c (c2 + 8ca)) ≥ (a + b + c)3

basta demostrar que

(a + b + c)3 ≥ a (a2 + 8bc) + b (b2 + 8ca) + c (c2 + 8ca)


⇔ (a + b + c)3 ≥ a3 + b3 + c3 + 24abc
⇔ a3 + b3 + c3 + 3(a + b)(a + c)(b + c) ≥ a3 + b3 + c3 + 24abc
⇔ (a + b)(a + c)(b + c) ≥ 8abc

pero esta última desigualdad es verdadera.

a b c
∴ √ +√ +√ ≥ 1.
a2 + 8bc b2 + 8ca c2 + 8ab
2

1 1 1
79. (APMO 2002). Sean a, b y c números reales positivos que satisfacen + + =
a b c
1, pruebe que

√ √ √ √ √ √ √
a + bc + b + ca + c + ab ≥ abc + a+ b+ c.
219

 Prueba. Como

1 1 1
+ + =1
a b c
 
√ 1 1 1 √
⇔ abc + + = abc
a b c
r r r
bc ac ab √
⇔ + + = abc.
a b c

Luego la desigualdad es equivalente a:

rr r
√ √ √ bc ac ab √ √ √
a + bc + b + ca + c + ab ≥ + + + a+ b+ c
a b c
r ! r 
√ √ √ bc √ ac √
a + bc + b + ca + c + ab ≥ + a + + b
a b
r !
ab √
+ + c
c

Veamos que es suficiente probar que


r
√ bc √
a + bc ≥ + a; elevando al cuadrado
a
bc √
⇔ a + bc ≥ + 2 bc + a
a
 
1 √ 1 1 1
⇔ bc ≥ bc + 2 bc; = 1− −
a a b c
 
1 1 √
⇔ bc ≥ bc 1 − − + 2 bc
b c

⇔ bc ≥ bc − (c + b) + 2 bc

⇔ b+c ≥ 2 bc
√ √ 2
⇔ b− c ≥ 0

2
220 12. Soluciones

80. Sean a, b, c números reales positivos que satisfacen abc = 1, pruebe que

3 6
1+ ≥ .
a+b+c ab + ac + bc

1 1 1
 Prueba. Haciendo = x, = y, = z ⇒ xyz = 1.
a b c
La desigualdad es equivalente a:

3 6
1+ ≥
1 1 1 1 1 1
+ + + +
x y z xy xz yz
3 6
⇔ 1+ ≥ (12.14)
xy + xz + yz x+y+z

Pero

(x + y + z)2 ≥ 3(xy + xz + yz)


3 9
⇔ ≥
xy + xz + yz (x + y + z)2
3 9
⇔ 1+ ≥ 1+ (12.15)
xy + xz + yz (x + y + z)2

De (12.14) y (12.15) basta demostrar

9 6
1+ 2

(x + y + z) x+y+z
 2
3
⇔ 1− ≥ 0.
x+y+z
2

81. Sean a, b, c, x, y, z números reales positivos tales que x + a = y + b = z + c = 1,


pruebe que
 
1 1 1
(abc + xyz) + + ≥ 3.
ay bz cx
221

 Prueba. De x+a = y +b = z +c = 1 se tiene que x = 1−a, y = 1−b, z =


1 − c entonces

abc + xyz = abc + (1 − a)(1 − b)(1 − c)


= abc + 1 − (a + b + c) + (ab + ac + bc) − abc
= 1 − (b + c) + bc + ab + ac − a
= (1 − b)(1 − c) + ab + ac − a
= (1 − b)(1 − c) + a(b − 1) + ac
abc + xyz (1 − b)(1 − c) ac
⇔ = −1+
a(1 − b) a(1 − b) a(1 − b)
abc + xyz 1−c c
⇔ = + − 1; similarmente
ay a 1−b
abc + xyz 1−a a
= + −1
bz b 1−c
abc + xyz 1−b b
= + −1
cx c 1−a

sumando miembro a miembro:

 
1 1 1 1−c c 1−a a 1−b
(abc + xyz) + + = + + + +
ay bz cx a 1−b b 1−c c
b
+ −3
1−a
   
1−c a c 1−b
= + + +
a 1−c 1−b c
 
1−a b
+ + −3
b 1−a
≥ 2 + 2 + 2 − 3 = 3.
2

82. Sean x, y, z números reales positivos, pruebe que

x+y+z xy + yz + zx
√ ≥p p √ .
3 3 x2 + xy + y 2 + y 2 + yz + z 2 + z 2 + zx + x2
222 12. Soluciones

 Prueba. Como

x2 + y 2 ≥ 2xy ⇔ 4x2 + 4y 2 + 4xy ≥ 3x2 + 3y 2 + 6xy


⇔ 4 (x2 + xy + y 2) ≥ 3(x + y)2
3
⇔ x2 + xy + y 2 ≥ (x + y)2
4

p 3
⇔ x2 + xy + y 2 ≥ (x + y); similarmente
2

p 3
⇔ y 2 + yz + z 2 ≥ (y + z)
2

√ 3
⇔ z 2 + zx + x2 ≥ (z + x)
2

sumando miembro a miembro, tenemos

p p √
x2 + xy + y 2 + y 2 + yz + z 2 + z 2 + zx + x2

3
≥ (x + y + y + z + z + x)
√2
= 3(x + y + z)
1
⇔ p p √
x + xy + y + y + yz + z 2 + z 2 + zx + x2
2 2 2

1
≤ √
3(x + y + z)
xy + yz + zx
⇔ p p √
x + xy + y + y 2 + yz + z 2 + z 2 + zx + x2
2 2

xy + yz + zx
≤ √ (12.16)
3(x + y + z)

Pero

x2 + y 2 + z 2 ≥ xy + xz + yz
⇔ (x + y + z)2 ≥ 3(xy + xz + yz)
x+y+z xy + xz + yz
√ ≥ √ (12.17)
3 3 3(x + y + z)
223

Por transitividad de (12.16) y (12.17)

xy + yz + zx x+y+z
p p √ ≤ √ .
x2 + xy + y2 2 2 2
+ y + yz + z + z + zx + x2 3 3
2

83. Sean x, y, z, t números reales tales que x2 + y 2 ≤ 1 y z 2 + t2 ≤ 1, pruebe que


p p √
(x + z)2 + (y + t)2 + (x − z)2 + (y − t)2 ≤ 2 2.

 Prueba. Como sabemos (a + b)2 ≤ 2(a2 + b2 )


p p 2
⇒ 2 2 2
(x + z) + (y + t) + (x − z) + (y − t) 2

≤ 2 ((x + z)2 + (y + t)2 + (x − z)2 + (y − t)2 )


p p
⇒ (x + z)2 + (y + t)2 + (x − z)2 + (y − t)2
√ hp i
≤ 2 2 2 2
2 (x + z + y + t ) 2

√ p  
≤ 2 2(1 + 1)

= 2 2
p p √
∴ (x + z)2 + (y + t)2 + (x − z)2 + (y − t)2 ≤ 2 2.
2

84. Dados a, b, c números reales tales que a2 +b2 +c2 ≤ 8, pruebe que ab+bc+2ca ≥
−8.

 Prueba. Como

8 ≥ a2 + b2 + c2
⇔ 8 + ab + bc + 2ca ≥ a2 + b2 + c2 + ab + bc + 2ca
b2 3b2
= a2 + + c2 + ab + bc + 2ca +
| 4 {z } 4
 2
b 3b2
= a+ +c + ≥0
2 4
224 12. Soluciones

La igualdad ocurre cuando

b
a+ + c = 0 ∧ b = 0 ∧ a2 + b2 + c2 = 8,
2

de donde

(a, b, c) = (2, 0, −2) ∨ (a, b, c) = (−2, 0, 2).

85. Dados a, b, c números reales no negativos tales que a + b + c = 1, pruebe que


256
a4 b + b4 c + c4 a ≤ .
3125

 Prueba. Sin pérdida de generalidad, supongamos que a = máx {a, b, c},


entonces

b4 c ≤ a3 bc y c4 a ≤ c2 a3 ≤ ca4

3c c
y desde que ≥ , tenemos:
4 2

a4 b + b4 c + c4 a ≤ a4 b + a3 bc + c4 a
4 3 c4 a c4 a
= a b + a bc + +
2 2
4 2 3
ca c a
≤ a4 b + a3 bc + +
2 2
3
ac
= a3 b(a + c) + (a + c)
2
  
3 c 3 3c
= a (a + c) b + ≤ a (a + c) b +
2 4
225

 
3c
⇒ a4 b + b4 c + c4 a ≤ a3 (a + c) b +
4
  
4 a a a a+c 3c
= 4 · · · · b+
4 4 4 4 4
 
a a a a+c 3c 5
+ + + +b+
 4 
≤ 44  4 4 4 4 
5
 5  5
4 a+b+c 41 256
= 4 =4 = .
5 5 3125
2

86. Si a, b, y c son los lados de un triángulo, pruebe que

a b c
+ + ≥ 3.
b+c−a c+a−b a+b−c

 Prueba. Haciendo b + c − a = x, c + a − b = y, a + b − c = z, tenemos


y+z x+z x+y
que a = ,b= ,c= .
2 2 2
Reemplazando, la desigualdad es equivalente a:

y+z x+z x+y


+ + ≥3
2x 2y 2z
y+z x+z x+y
⇔ + + ≥6
x y z
    
y x z x z y
⇔ + + + + + ≥6
x y x z y z

m n
es verdadera pues + ≥ 2, ∀ m, n ∈ R+ . 2
n m
87. Sea n un número entero ≥ 2, pruebe que
!4
X  X
8 xi xj x2i + x2j ≤ xi .
1≤i<j≤n 1≤i≤n
226 12. Soluciones

 Prueba. Recordar

(x1 + x2 + · · · + xn )2 = (x21 + x22 + · · · + x2n ) + 2 (x1 x2 + · · · + xn−1 xn )


X X
= x2i + 2 xi xj ,
1≤i≤n 1≤i<j≤n

luego, tenemos que

!4  !2  2 !2
X X X X
xi = xi  = x2i + 2 xi xj
1≤i≤n 1≤i≤n 1≤i≤n 1≤i<j≤n

! !
X X
≥ 4 x2i 2 xi xj
1≤i≤n 1≤i<j≤n
!
X X
= 8 x2i xi xj
1≤i≤n 1≤i<j≤n

 X
= 8 x21 + x22 + · · · + x2n xi xj
1≤i<j≤n
X 
= 8 xi xj x21 + x22 + · · · + x2n
1≤i<j≤n
X 
≥ 8 xi xj x2i + x2j .
1≤i<j≤n

88. Sean a1 , a2 , . . . , an números reales positivos y S = a1 + a2 + · · · + an , pruebe


que

S S S n2
+ +···+ ≥ .
S − a1 S − a2 S − an n−1
227

 Prueba. Como MA ≥ MH, tenemos que

(S − a1 ) + (S − a2 ) + · · · + (S − an ) n

n 1 1 1
+ +···+
S − a1 S − a2 S − an
  
 
1 1 1
⇔ nS − a1 + a2 + · · · + an  + +···+ ≥ n2
| {z } S − a1 S − a2 S − an
S
 
1 1 1
⇔ (n − 1)S + +···+ ≥ n2
S − a1 S − a2 S − an
S S S n2
⇔ + +···+ ≥ .
S − a1 S − a2 S − an n−1

89. Sean a, b, c números reales no negativos tales que a + b + c = 3, pruebe que

√ √ √
a 1 + b3 + b 1 + c3 + c 1 + a3 ≤ 5.

 Prueba. Teniendo presente que

√ p
a 1 + b3 = a (1 + b) (1 − b + b2 )

La desigualdad es equivalente a:

p p p
a (1 + b) (1 − b + b2 ) + b (1 + c) (1 − c + c2 ) + c (1 + a) (1 − a + a2 ) ≤ 5

pero
 
p (1 + b) + (1 − b + b2 )
a (1 + b) (1 − b + b2 ) ≤ a
2
p 1
⇔ a (1 + b) (1 − b + b2 ) ≤ (a) (2 + b2 )
2
228 12. Soluciones

luego basta demostrar que

1 1 1
a (2 + b2 ) + b (2 + c2 ) + c (2 + a2 ) ≤ 5
2 2 2
⇔ 2(a b + }c) + ab + bc2 + ca2 ≤ 10
| +{z
2

⇔ 6 + ab2 + bc2 + ca2 ≤ 10


⇔ ab2 + bc2 + ca2 ≤ 4.

Supongamos sin pérdida de generalidad que

c≤b≤a
⇒ a(b − a)(b − c) ≤ 0
⇔ ab2 + a2 c ≤ abc + a2 b

entonces es suficiente probar

abc + a2 b + bc2 ≤ 4
⇔ b (ac + a2 + c2 ) ≤ 4

Pero

b (ac + a2 + c2 ) ≤ b(a + c)2


  
a+c a+c
= 4b
2 2
 3
a+b+c
≤ 4 =4
3

la igualdad ocurre cuando c = 0, b = 1, a = 2 y sus correspondientes permuta-


ciones. 2

90. Sean a, b, c, d números reales no negativos, pruebe que


    
2a 2b 2c 2d
1+ 1+ 1+ 1+ ≥ 9.
b+c c+d d+a a+b
229

 Prueba. Efectuando convenientemente:


    
2a + b + c 2b + c + d 2c + d + a 2d + a + b
≥9
b+c c+d d+a a+b
    
2a + b + c 2b + c + d 2c + d + a 2d + a + b
⇔ ≥9
a+b b+c c+d d+a
    
a+c b+d c+a d+b
⇔ 1+ 1+ 1+ 1+ ≥9
a+b b+c c+d d+a

utilizando el problema 2 tenemos que

    2
a+c a+c 2(a + c)
1+ 1+ ≥ 1+
a+b c+d a+b+c+d

    2
b+d d+b 2(b + d)
1+ 1+ ≥ 1+
b+c d+a a+b+c+d
    
a+c b+d c+a d+b
⇒ 1+ 1+ 1+ 1+
a+b b+c c+d d+a

 2  2
2(a + c) 2(b + d)
≥ 1+ 1+
a+b+c+d a+b+c+d

luego basta demostrar que

 2  2
2(a + c) 2(b + d)
1+ 1+ ≥9
a+b+c+d a+b+c+d
  
2(a + c) 2(b + d)
⇔ 1+ 1+ ≥ 3.
a+b+c+d a+b+c+d
230 12. Soluciones

Haciendo a + c = m y b + d = n, tenemos equivalente a


  
2m 2n
1+ 1+ ≥3
m+n m+n

2(m + n) 4mn
⇔ 1+ + ≥3
m+n (m + n)2

4mn
⇔ 3+ ≥3
(m + n)2

4mn
⇔ ≥ 0.
(m + n)2

91. Si a, b, c son números reales positivos, entonces


r r r
2a 2b 2c
+ + ≤ 3.
a+b b+c c+a

 Prueba. La desigualdad a probar es equivalente a:


v v v
u 2 u 2 u 2
u u u
t b
+t c +t a ≤3
1+ 1 + 1 +
a b c

b c a
Haciendo = x2 , = y 2 , = z 2 , con x, y, z positivos ⇒ (xyz)2 = 1
a b c
Reemplazando tenemos:
r r r
2 2 2
+ + ≤3
1 + x2 1 + y2 1 + z2
231

Pero

r r 2  
2 2 2 2 2 2
·1+ ·1 ≤ (1 + 1 ) +
1 + x2 1 + y2 1 + x2 1 + y 2
 
1 1
= 4 +
1 + x2 1 + y 2
 
1 − (xy)2
= 4 1+
(1 + x2 ) (1 + y 2 )
 
1 − (xy)2
≤ 4 1+
(1 + xy)2
8 8z
= =
1 + xy z+1
r r √
2 2 2 2z
⇒ + ≤ √
1 + x2 1 + y2 z+1

luego basta demostrar que

√ r
2 2z 2
√ + ≤3
z+1 1 + z2

Pero

(z + 1)2 ≤ 2 (1 + z 2 )
2 4
⇔ 2

1+z (z + 1)2
r
2 2
⇔ 2
≤ ,
1+z (z + 1)

luego será suficiente probar que


2 2z 2
√ + ≤3
z+1 z+1
232 12. Soluciones

p
⇔ 2 2z(z + 1) + 2 ≤ 3 + 3z
p p
⇔ 0 ≤ (3z + 1) − 2 2z(z + 1) = 2z − 2 2z(z + 1) + (z + 1)
√ √
⇔ 0 ≤ ( 2z − z + 1)2 . [8, pág 34]
2

92. Si x1 , x2 , . . . , xn números reales positivos tales que

1 1 1
+ +···+ = 1,
1 + x1 1 + x2 1 + xn

pruebe que

x1 x2 · · · xn ≥ (n − 1)n .

 Prueba. De la condición

1 1 1 1
+ +···+ + =1
1 + x1 1 + x2 1 + xn−1 1 + xn

tenemos

1 1 1 1 xn
+ +···+ =1− =
1 + x1 1 + x2 1 + xn−1 1 + xn 1 + xn

Luego aplicando MA ≥ MG:

1 1 1
+ +···+ r
1 + x1 1 + x2 1 + xn−1 1 1 1
≥ n−1
· ···
n−1 1 + x1 1 + x2 1 + xn−1
xn n−1
⇒ ≥ p
1 + xn (1 + x1 ) (1 + x2 ) · · · (1 + xn−1 )
n−1

xi n−1
⇒ ≥ v ; i = 1, 2, . . . , n
1 + xi u Y n
u
u (1 + xj )
u
u
n−1
t j=1
j 6= i
233

Multiplicando sucesivamente para todo i = 1, 2, . . . , n, tenemos

x1 x2 · · · xn ≥ (n − 1)n .

93. Sean a, b, c números reales positivos tales que a2 + b2 + c2 = 3, pruebe que

ab bc ca
+ + ≥ 3.
c a b

 Prueba. Como a, b, c > 0, la desigualdad es equivalente a


!
a2 b2 b2 c2 c2 a2 2
+ 2 + 2 +2 a b2 + c}2 ≥ 9
| + {z
c2 a b
3
2 2 2 2 2 2
ab bc c a
⇔ 2
+ 2 + 2 ≥3
c a b
2 2 2 2 2 2
ab bc c a
⇔ 2
+ 2 + 2 ≥ a2 + b2 + c2
c a b

Aplicando MA ≥ MG:
r
a2 b2 b2 c2 a2 b2 b2 c2
+ 2 ≥ 2 · 2 = 2b2
c2 a c2 a
r
a2 b2 c2 a2 a2 b2 c2 a2
+ 2 ≥ 2 · 2 = 2a2
c2 b c 2 b
r
b2 c2 c2 a2 b2 c2 c2 a2
+ 2 ≥ 2 · 2 = 2c2
a2 b a2 b

Sumando:

a2 b2 b2 c2 c2 a2
+ 2 + 2 ≥ a2 + b2 + c2 .
c2 a b
2
234 12. Soluciones

94. Sean a, b, c números reales positivos, pruebe que

a+b+c
aa · bb · cc ≥ (abc) 3 .

 Prueba. Sean a ≥ b ≥ c y ln a ≥ ln b ≥ ln c, aplicando la Desigualdad de


Chebyshev
  
a ln a + b ln b + c ln c a+b+c ln a + ln b + ln c

3 3 3
 
a b c a+b+c
ln a + ln b + ln c ≥ ln abc
3
 
a+b+c
 
ln aa bb cc ≥ ln abc 3

de donde

a+b+c
a b c 3
a · b · c ≥ (abc) .

95. Sean a, b, c números reales positivos, pruebe que

a b c c+a a+b b+c


+ + ≥ + + .
b c a c+b a+c b+a

a b c
 Prueba. Haciendo = x, = y, = z, tenemos xyz = 1 y además
b c a
a+c 1 + xy 1−x
= =x+
b+c 1+y 1+y

y ası́ similarmente para los otros.


235

Reemplazando tenemos una desigualdad equivalente a la primera

1−x 1−y 1−z


x+y+z ≥ x+ +y+ +z+
1+y 1+z 1+x
x−1 y−1 z−1
⇔ + + ≥0
y+1 z+1 x+1
⇔ (x2 − 1) (z + 1) + (y 2 − 1) (x + 1) + (z 2 − 1) (y + 1) ≥ 0
⇔ x2 z + y 2x + z 2 y + x2 + y 2 + z 2 − (x + y + z) − 3 ≥ 0
⇔ x2 z + y 2x + z 2 y + x2 + y 2 + z 2 ≥ x + y + z + 3
p
como x2 z + y 2x + z 2 y ≥ 3 · 3
x3 y 3 z 3 = 3 y x2 + y 2 + z 2 ≥ x + y + z, pues
(x + y + z)2
x + y + z ≥ 3 y x2 + y 2 + z 2 ≥ .
3
∴ x2 z + y 2 x + z 2 y + x2 + y 2 + z 2 ≥ x + y + z + 3 es verdadero. 2

96. Si a, b, c, d son números reales positivos, entonces

a−b b−c c−d d−a


+ + + ≥ 0.
b+c c+d d+a a+b

 Prueba. Tenemos que

a−b b−c c−d d−a


+ + +
b+c c+d d+a a+b
       
a−b b−c c−d d−a
= +1 + +1 + +1 + +1 −4
b+c c+d d+a a+b
a+c b+d c+a d+b
= + + + −4
b+c c+d d+a a+b
   
1 1 1 1
= (a + c) + + (b + d) + −4
b+c a+d c+d a+b

pero

1 1 4 1 1 4
+ ≥ ; + ≥
b+c a+d (b + c) + (d + a) c + d a + b (c + d) + (a + b)
236 12. Soluciones

entonces
 
1 1 4(a + c)
(a + c) + ≥
b+c a+d a+b+c+d
 
1 1 4(b + d)
(b + d) + ≥
c+d a+b a+b+c+d

luego sumando miembro a miembro, tenemos que


   
1 1 1 1
(a + c) + + (b + d) + ≥4
b+c a+d c+d a+b

a−b b−c c−d d−a


∴ + + + ≥ 0.
b+c c+d d+a a+b
2

97. Sean a, b, c números reales positivos, pruebe que

a3 b3 c3 3(ab + ac + bc)
2 2
+ 2 2
+ 2 2
≥ .
b − bc + b c − ac + a a − ab + b a+b+c

3(ab + ac + bc)
 Prueba. Como (a+b+c)2 ≥ 3(ab+ac+bc) ⇒ a+b+c ≥ .
a+b+c
Demostraremos la siguiente desigualdad que es más fuerte

a3 b3 c3
+ + ≥ a + b + c.
b2 − bc + b2 c2 − ac + a2 a2 − ab + b2

Luego para aplicar el Lema de Titu en el primer miembro de la desigualdad,


buscamos un equivalente adecuado, en efecto, tenemos que

a3 b3 c3
+ +
b2 − bc + b2 c2 − ac + a2 a2 − ab + b2
a4 b4 c4
= + +
ab2 − abc + ab2 bc2 − abc + a2 b a2 c − abc + b2 c
2
(a2 + b2 + c2 )

ab2 + ac2 + bc2 + ba2 + a2 c + b2 c − 3abc
237

entonces basta demostrar que

2
(a2 + b2 + c2 )

≥ (a + b + c) (ab2 + ac2 + bc2 + ba2 + a2 c + b2 c − 3abc)

a4 + b4 + c4 + 2 (a2 b2 + a2 c2 + b2 c2 )

≥ a2 b2 + a2 c2 + abc2 + a3 b + a3 c + acb2 − 3a2 bc

+ab3 + abc2 + b2 c2 + b2 a2 + bca2 + cb3 − 3ab2 c

+acb2 + ac3 + bc3 + a2 bc + a2 c2 + b2 c2 − 3abc2

a4 + b4 + c4 + a2 bc + ab2 c + abc2

≥ ab (a2 + b2 ) + bc (b2 + c2 ) + ca (c2 + a2 )

a2 (a2 − (b + c)a + bc) + b2 (b2 − (c + a)b + ac) + c2 (c2 − (b + a)c + ab) ≥ 0

a2 (a − b)(a − c) + b2 (b − a)(b − c) + c2 (c − a)(c − b) ≥ 0.

98. Si a, b, c, d son números reales positivos, entonces


 2  2  2  2
a b c d
+ + + ≥ 1.
a+b b+c c+d d+a

 Prueba. La desigualdad es equivalente a:


 2  2  2  2
 1  1  + 1  1
 + +  ≥1
b 1+
c  d 1+
a
1+ b 1+ d
a c
238 12. Soluciones

Haciendo

b c
= x, = y,
a b
d a
= z, = w.
c d

La desigualdad se convierte en:

 2  2  2  2
1 1 1 1
+ + + ≥1
1+x 1+y 1+z 1+w

con xyzw = 1.
Aplicando el problema 21, tenemos que

1 1 1
2 + 2 ≥ ; similarmente
(1 + x) (1 + y) 1 + xy
1 1 1 xy xy
2 + 2 ≥ = =
(1 + z) (1 + w) 1 + zw xy + xyzw xy + 1

sumando miembro a miembro, tenemos que

1 1 1 1 1 + xy
2 + 2 + 2 + 2 ≥ =1
(1 + x) (1 + y) (1 + z) (1 + w) xy + 1

1 1 1 1
∴ 2 + 2 + 2 + ≥ 1.
(1 + x) (1 + y) (1 + z) (1 + w)2

99. Si a, b, c son números reales no negativos, entonces

  
3 1 − a + a2 1 − b + b2 1 − c + c2 ≥ 1 + abc + a2 b2 c2 .
239

 Prueba. Desarrollar todo el primer miembro es muy tedioso y aprovechan-


do la forma de la expresión, efectuaremos los dos primeros factores, veamos

2 (1 − a + a2 ) (1 − b + b2 )

= 2 − 2b + 2b2 − 2a + 2ab − 2ab2 + 2a2 − 2a2 b + 2a2 b2

= 1 + a2 b2 + (a2 − 2ab + b2 ) + a2 + b2 + a2 b2 + 1 − 2a2 b − 2ab2 − 2a − 2b

+4ab

= 1 + a2 b2 + (a − b)2 + (a − 1)2 + b2 (1 − 2a + a2 ) − 2b (a2 − 2a + 1)

= 1 + a2 b2 + (a − b)2 + (a − 1)2 (1 + b2 − 2b)

= 1 + a2 b2 + (a − b)2 + (a − 1)2 (b − 1)2

⇒ 2 (1 − a + a2 ) (1 − b + b2 ) ≥ 1 + a2 b2
1
⇔ (1 − a + a2 ) (1 − b + b2 ) ≥ (1 + a2 b2 )
2
3
⇔ 3 (1 − a + a2 ) (1 − b + b2 ) (1 − c + c2 ) ≥ (1 + a2 b2 ) (1 − c + c2 )
2

Luego basta demostrar que

3  
1 + a2 b2 1 − c + c2 ≥ 1 + abc + a2 b2 c2
2

en efecto, transformando a una desigualdad cuadrática en c, tenemos:

3 (1 + a2 b2 ) c2 + 3 (1 + a2 b2 ) (1 − c) ≥ 2 + 2abc + 2a2 b2 c2

(3 + 3a2 b2 − 2a2 b2 ) c2 − (3a2 b2 + 2ab + 3) c + (3 + 3a2 b2 − 2) ≥ 0


 
3 + a2 b2  c2 − (3a2 b2 + 2ab + 3) c + (1 + 3a2 b2 ) ≥ 0
| {z }
(+)
240 12. Soluciones

calculando su discriminante

2
∆ = (3a2 b2 + 2ab + 3) − 4 (3 + a2 b2 ) (1 + 3a2 b2 )

= 9a4 b4 + 4a2 b2 + 9 + 12a3 b3 + 18a2 b2 + 12ab − 4 (3 + 10a2 b2 + 3a4 b4 )

= −3a4 b4 + 12a3 b3 − 18a2 b2 + 12ab − 3

= −3 [a4 b4 + 4a3 b3 − 6a2 b2 + 4ab − 1]

= −3[ab − 1]4 ≤ 0

Como ∆ ≤ 0, entonces la desigualdad se cumple para todo c, a, b ∈ R, en


particular para a, b, c ∈ R+
0.

La igualdad ocurre si y sólo si a = b = c = 1. 2

100. Sean a, b, c son números reales no negativos, pruebe que

a2 + b2 + c2 + 2abc + 1 ≥ 2(ab + ac + bc).

 Prueba. La desigualdad es equivalente a:

a2 + b2 + c2 + 2abc + 1 − 2(ab + ac + bc) ≥ 0


⇔ a2 + b2 + c2 + 2abc + 1 − 2ab − 2ac − 2bc ≥ 0
⇔ a2 − 2ab + b2 + (c2 − 2c + 1) + 2c − 2ac + 2abc ≥ 0
⇔ (a − b)2 + (c − 1)2 + 2c(1 − (a + b) + ab) ≥ 0
⇔ (a − b)2 + (c − 1)2 + 2c(1 − a)(1 − b) ≥ 0 (12.18)

es importante observar que también podrı́amos haber agrupado de de las si-


guientes maneras

(a − 1)2 + (b − c)2 + 2a(1 − b)(1 − c) ≥ 0 (12.19)


(b − 1)2 + (a − c)2 + 2b(1 − a)(1 − c) ≥ 0 (12.20)
241

Aplicando el problema 3 a los números (1 − a), (1 − b), (1 − c), tenemos que al


menos una de las siguientes desigualdades es verdadera

(1 − a)(1 − b) ≥ 0
(1 − b)(1 − c) ≥ 0
(1 − c)(1 − a) ≥ 0.

Sin pérdida de generalidad supongamos que (1 − a)(1 − b) ≥ 0, luego

(a − b)2 + (c − 1)2 + 2c(1 − a)(1 − b) ≥ 0.

101. Sean a, b, c números reales positivos tales que abc ≥ 1, pruebe que

a3 + b3 + c3 ≥ ab + ac + bc.

 Prueba. Podemos asumir a ≥ b ≥ c, a2 ≥ b2 ≥ c2


Aplicando Chebyshev
  2 
a · a2 + b · b2 + c · c2 a+b+c a + b2 + c2

3 3 3

3
a3 + b3 + c3 ≥ abc · (ab + ac + bc)
≥ 1(ab + ac + bc)

∴ a3 + b3 + c3 ≥ ab + ac + bc.

102. Sean a, b, c números reales positivos tales que a + b + c = 1, pruebe que

√ √ √ √ √ √
ab + c + bc + a + ca + b ≥ 1 + ab + bc + ca.
242 12. Soluciones

 Prueba. Como

a + b ≥ 2 ab

⇔ a + b + c ≥ 2 ab + c

⇔ 1 ≥ 2 ab + c

⇔ c ≥ 2 abc + c2

⇔ ab + c ≥ ab + 2 abc + c2

⇔ ab + c ≥ ( ab + c)2
√ √
⇔ ab + c ≥ ab + c; similarmente
√ √
bc + a ≥ bc + a
√ √
ca + b ≥ ca + b

sumando miembro a miembro tenemos:


√ √ √ √ √ √
ab + c + bc + a + ca + b ≥ a + b
| {z }+ c + ab + bc + ca
1

√ √ √ √ √ √
∴ ab + c + bc + a + ca + b ≥ 1 + ab + bc + ca.

103. Sean a, b números reales positivos tales que a + b ≤ 1, halle el mı́nimo valor
de f (a, b) = ab + (ab)−1 .

1
Solución. Como a, b > 0 entonces ab + ≥ 2 y decimos que el mı́nimo de
ab
f es 2, pero veamos cuándo ocurre este mı́nimo.
1
El mı́nimo ocurre si ab = =1
ab
entonces

√ a+b 1
1= ab ≤ ≤ , ¡contradicción!
2 2

esto significa que el mı́nimo de f no es 2.


243

Haciendo

1 1 1 1
h= ⇒h= ≥ 2 ≥  2 = 4
ab ab a+b 1
2 2

entonces el problema se transforma en:


1
Si h ≥ 4, halle el mı́nimo de h + .
h
En estos casos se recomienda el siguiente procedimiento:
Tomamos h = 4, luego buscamos un parámetro α adecuado

h 4
= 4 1
α α ⇒ = ⇒ α = 16
1 1 α 4
=
h 4

Luego
  r
1 h 1 15h h 1 15h
h+ = + + ≥ 2 · +
h 16 h 16 16 h 16
2 15h 2 15 × 4 17
= + ≥ + =
4 16 4 16 4

de donde

1 17
h+ ≥
h 4

entonces

1 17
ab + ≥
ab 4

17 1
∴ el mı́nimo es y ocurre cuando a = b = c = .
4 2
2
244 12. Soluciones

3
104. Sean a, b, c números reales positivos tales que a + b + c ≤ , halle el mı́nimo
r r r 2
1 1 1
de f (a, b, c) = a2 + 2 + b2 + 2 + c2 + 2 .
b c a
Solución. Aplicando MA ≥ MG:
sr r r
3 1 1 1
f (a, b, c) ≥ 3 a2 + 2 · b2 + 2 · c2 +
b c a2
s   
1 1 1
= 3 6
a2 + 2 b2 + 2 c2 + 2
b c a
s r r r
6 1 1 1
≥ 3 2 a2 · 2 · 2 b2 · 2 · 2 c2 · 2 ·
b c a

= 3 2

√ √
entonces f (a, b, c) ≥ 3 2, el mı́nimo de f es 3 2 y ocurre cuando

1 1 1
a=b=c= = = = 1,
a b c

3
entonces a + b + c = 3, pero a + b + c ≤ ¡contradicción!
2

Esto significa que el mı́nimo no es 3 2.
Analizamos la expresión y aprovechando la forma simétrica estimamos que el
1
mı́nimo de f (a, b, c) ocurre cuando a = b = c = , luego como en el problema
2
anterior hacemos

 1

 a2 = b2 = c2 =
1 4
a=b=c= ⇒
2 
 1 = 1 = 1 = 4

αa2 αb2 αc2 α
1 4
⇒ = ⇒ α = 16
4 α
245

luego tenemos que

v v
u 1 1 u
u
f (a, b, c) = ua2 + + · · · + + ub2 + 1 + · · · + 1
2 2 u 2 2
t |16b {z 16b } t |16c {z 16c }
16 sumandos 16 sumandos

v
u 1 1
+u
uc2 + 2
+···+ 2
t |16a {z 16a }
16 sumandos

s r s r s r
2 2
17 a 17 b 17 c2
≥ 17 + 17 + 17
1616 b32 1616 c32 1616 a32
"r r r #
√ a b c
= 17 17 8 16
+ 17 8 16
+ 17 +
16 b 16 c 168 a16
sr
r
√ 3 17 abc √ 1
≥ 3 17 24 16 16 16
= 3 17 · 17
16 a b c 16 (abc)5
8

√ √ √
3 17 3 17 3 17
= 17
p = 17p = 17p
168 (abc)5 217 · 215 · (abc)5 2 ((2a)(2b)(2c))5
√ √
3 17 3 17
≥ r ≥ ;
17 2a + 2b + 2c 5 2
2 ( )
3

1
como vemos para a = b = c = ocurre el mı́nimo de f (a, b, c) y dicho mı́nimo
√ 2
3 17
es . 2
2
105. Sean a, b, c números reales tales que a ≥ 2, b ≥ 6 y c ≥ 12, halle el máximo
valor de
√ √ √
bc a − 2 + ca 3 b − 6 + ab 4 c − 12
f (a, b, c) = .
abc
246 12. Soluciones

Solución.

√ bc p bc (a − 2) + 2 abc
bc a − 2 = √ (a − 2) · 2 ≤ √ · = √
2 2 2 2 2
√ ca 3p ca (b − 6) + 3 + 3 abc
ca 3 b − 6 = √ (b − 6) · 9 ≤ √ · = √
3
9 3
9 3 339
√ ab p ab (c − 12) + 4 + 4 + 4 abc
ab 4 c − 12 = √ 4
(c − 12) · 64 ≤ √ · = √
4
64 4
64 4 4 4 64
abc
= √
8 2

entonces
 
1 abc abc abc 5 1
f (a, b, c) ≤ √ + √ + √ = √ + √
abc 2 2 339 8 2 8 2 339

el mı́nimo ocurre para

a − 2 = 2, b − 6 = 3, c − 12 = 4
a = 4, b = 9, c = 16

5 1
∴ máx f (a, b, c) = √ + √ .
8 2 339
2

a3 b3 c3
106. Sean a, b, c números reales positivos, pruebe que + + ≥ ab + bc + ca.
b c a
 Prueba. Aplicando MA ≥ MG:
r
a3 b3 3 a
3
b3
+ + bc ≥ 3 · · bc = 3ab
b c b c
r
b3 c3 3 b
3
c3
+ + ca ≥ 3 · · ca = 3bc
c a c a
r
3 3 3
c a 3 c a3
+ + ab ≥ 3 · · ab = 3ca
a b a b
247

sumando miembro a miembro:


 
a3 b3 c3
2 + + + bc + ca + ab ≥ 3ab + 3bc + 3ca
b c a

a3 b3 c3
de donde + + ≥ ab + bc + ca.
b c a
La igualdad ocurre si y sólo si a = b = c. 2

107. Sean a, b, c, d números reales positivos, halle el mı́nimo valor de la función

a b c d
f (a, b, c, d) = + + +
b+c+d c+d+a d+a+b a+b+c
b+c+d c+d+a d+a+b a+b+c
+ + + + .
a b c d

Solución. Si aplicamos MA ≥ MG, tenemos que

q
a b c d b+c+d c+d+a d+a+b a+b+c
f (a, b, c, d) ≥ 8 8 b+c+d · c+d+a
· d+a+b
· a+b+c
· a
· b
· c
· d

f (a, b, c, d) ≥ 8

⇒ mı́n f = 8.
¿Cuándo ocurre tal mı́nimo?


a= b+c+d




b = c + d + a
mı́n f = 8 ⇔

c = d+a+b




d = a + b + c

sumando tenemos

|a + b {z
+ c + d} = 3(a
| + b {z
+ c + d})
(+) (+)

⇒ 1 = 3 imposible.
248 12. Soluciones

Luego el mı́nimo de f no es 8.
Sean a = b = c = d > 0

a b c d 1
⇒ = = = =
b+c+d c+d+a d+a+b a+b+c 3
b+c+d c+d+a d+a+b a+b+c 3
= = = =
αa αb αc αd α

1 3
igualando = ⇒ α = 9.
3 α
Esto significa que podemos agrupar de esta manera:

a b c d b+c+d c+d+a
+ + + + +
b+c+d c+d+a d+a+b a+b+c 9a 9b
d+a+b a+b+c
+ +
9c 9d
 
8 b+c+d c+d+a d+a+b a+b+c
+ + + +
9 a b c d
q
a b c d
≥ 8 8 b+c+d · c+d+a · d+a+b · a+b+c · b+c+d
9a
· c+d+a
9b
· d+a+b
9c
· a+b+c
9d
 
8 b c d c d a d a b a b c
+ + + + + + + + + + + +
9 a a a b b b c c c d d d
r
8 8 b c d c d a d a b a b c
≥ + · 12 12 · · · · · · · · · · ·
3 9 a a a b b b c c c d d d
8 32 40
= + =
3 3 3

40
∴ fmı́n = y ocurre cuando a = b = c = d > 0.
3
2

108. Halle el máximo valor de

a3 (b + c + d) + b3 (c + d + a) + c3 (d + a + b) + d3 (a + b + c)

donde a, b, c, d son números reales tales que la suma de cuadrados es 1.


249

Solución.

f (a, b, c, d) = a3 (b + c + d) + b3 (c + d + a) + c3 (d + a + b)
+d3 (a + b + c)
= (a3 b + ab3 ) + (a3 c + ac3 ) + (a3 d + ad3 )
+ (b3 c + bc3 ) + (b3 d + bd3 ) + (c3 d + cd3 )
= ab (a2 + b2 ) + ac (a2 + c2 ) + ad (a2 + d2 )
+bc (b2 + c2 ) + bd (b2 + d2 ) + cd (c2 + d2 )

Pero

(a − b)4 = a4 − 4ab (a2 + b2 ) + 6a2 b2 + b4


a4 + b4 + 6a2 b2 − (a − b)4
⇒ ab (a2 + b2 ) =
4

luego

f (a, b, c, d)
3 (a2 + b2 + c2 + d2 ) + 6 (a2 b2 + a2 c2 + a2 d2 + b2 c2 + b2 d2 + c2 d2 )
=
4
(a − b)4 + (a − c)4 + (a − d)4 + (b − c)4 + (b − d)4 + (c − d)4

4
 2
1
z }| {
 2 2 2 2
3 a + b + c + d  −[(a−b)4 +(a−c)4 +(a−d)4 +(b−c)4 +(b−d)4 +(c−d)4 ]
 

= 4
3 1
= − [(a − b)4 + (a − c)4 + (a − d)4 + (b − c)4 + (b − d)4 + (c − d)4 ]
4 4
3
≤ .
4

1
La igualdad ocurre cuando a = b = c = d = . 2
2
250 12. Soluciones

109. Sean a, b, c números reales positivos, pruebe que

 
a+b+c √
3 √ √ 2 √ √ 2 √ √ 2
− abc ≤ máx a− b , b− c , c− a .
3

Solución. Sin pérdida de generalidad podemos tomar el orden

a≤b≤c

entonces debemos demostrar que

a+b+c √
3 √ √ 2
− abc ≤ c− a .
3

Definamos la función

a+x+c √ x+a+c √
f (x) = − 3 axc = − 3 acx
3 3

donde a ≤ x ≤ c, luego

1 1 −2/3
f ′ (x) = − x
3 3
2 −5/3 √
f ′′ (x) = x ac ≥ 0, x ≥ 0
9

entonces f es cóncava, luego

f (x) ≤ máx {f (a), f (c)}

supongamos que f (c) ≤ f (a), entonces f (a) es el máximo de f , en efecto basta


demostrar que

√ √ 2
f (a) ≤ ( c − a)
1 √3 √
⇔ (2a + c) − a2 c ≤ c + a − 2 ac
3
251


3 1 2 √
⇔ a2 c + a + c − 2 ac ≥ 0
3 3

3 √
3 a2 c + a + 2c ≥ 6 ac

Pero vemos que aplicando MA ≥ MG se tiene


√ √ √ q
3
a2 c +
3
a2 c +
3
a2 c + a + c + c 6 √
3 3
≥ a2 c · a · c2
6

6
= a3 c3

ac
=

3 √
⇒ 3 a2 c + a + 2c ≥ 6 ac

 
a+b+c √
3 √ √ 2 √ √ 2 √ √ 2
∴ − abc ≤ máx a− b , b− c , c− a .
3

La igualdad ocurre cuando a = b = c. 2

110. Si a, b, c son números reales no negativos, pruebe que


 3
3 3 b+c 3
a +b +c +2 a− ≥ 3abc.
2

b+c
 Prueba. Si a − ≥ 0, entonces la desigualdad se cumple trivialmente
2
3 3 3
pues a + b + c ≥ 3abc.
b+c
Si a − < 0 definimos
2
 3
3 3 3 b+c
f (a, b, c) = a + b + c − 3abc + 2 a −
2

Haciendo b = a + 2p ∧ c = a + 2q, tenemos

 3
3 3 3 2a + 2p + 2q
f = a + (a + p) + (a + q) − 3a(a + p)(a + q) + 2 a −
2
252 12. Soluciones

efectuando

f = 12a (p2 − pa + q 2 ) + 6(p + q)(p − q)2 ≥ 6(p + q)(p − q)2


⇒ f ≥ 6(p + q)(p − q)2
  2
b+c b−c
⇒ f (a, b, c) ≥ 6 −a
2 2
 
3 
 b + c − a (b − c)2 ≥ 0
⇒ f (a, b, c) ≥
2 | 2 {z }
(+)

La igualdad ocurre si y sólo si (a, b, c) = (1, 1, 1) ∨ [(a, b, c) = (0, 1, 1)] y sus


correspondientes permutaciones. 2

111. Sean x1 , x2 , x3 , x4 , x5 números reales tales que x1 + x2 + x3 + x4 + x5 = 0,


pruebe que

|cos x1 | + |cos x2 | + |cos x3 | + |cos x4 | + |cos x5 | ≥ 1.

Solución. Sabemos que

| sen (x + y)| ≤ mı́n {| cos x| + | cos y|, | sen x| + | sen y|}


| cos (x + y)| ≤ mı́n {| sen x| + | cos y|, | sen x| + | cos y|}

entonces
! !
5
X 5
X

cos xi ≤ | cos x1 | + sen xi

i=1 i=2

≤ | cos x1 | + | cos x2 | + | cos (x3 + x4 + x5 )|

entonces

!
5
X

cos xi ≤ | cos x1 | + | cos x2 | + | cos x3 | + | cos x4 | + | cos x5 |

i=1

| cos (0)| = 1 ≤ | cos x1 | + | cos x2 | + | cos x3 | + | cos x4 | + | cos x5 |


253

∴ |cos x1 | + |cos x2 | + |cos x3 | + |cos x4 | + |cos x5 | ≥ 1.

112. Sean a, b, c números reales diferentes, pruebe que

a2 b2 c2
+ + ≥ 2.
(b − c)2 (c − a)2 (a − b)2

 Prueba. Aplicando el desarrollo de un trinomio al cuadrado.

 2
a b c
+ +
b−c c−a a−b
 2  2  2
a b c
= + +
b−c c−a a−b
 
 ab ac bc 
 
+2  + + 
 (b − c)(c − a) (b − c)(a − b) (c − a)(a − b) 
| {z }
(−1)
 2
a b c
⇒ + + +2
b−c c−a a−b
 2  2  2
a b c
= + +
b−c c−a a−b

Como

 2
a b c
+ + +2≥2
b−c c−a a−b

entonces

a2 b2 c2
+ + ≥ 2.
(b − c)2 (c − a)2 (a − b)2
254 12. Soluciones

La igualdad ocurre si y sólo si

a b c
+ + = 0.
b−c c−a a−b
2

113. (IMO 1960). Resuelva

4x2
√ 2 < 2x + 9.
1 − 1 + 2x

Solución.

i) Calculamos el conjunto de restricciones:



1 + 2x ≥ 0 ∧ 1 − 1 + 2x 6= 0
x ≥ −1/2 ∧ x 6= 0;

luego tenemos el conjunto de números reales a donde pertenece el con-


junto solución

U = [−1/2, +∞i − {0}.

ii) Racionalizando el denominador:

√ 2
4x2 1 + 1 + 2x
√ 2 √ 2 < 2x + 9
1 − 1 + 2x 1 + 1 + 2x
√ 2
4x2 1 + 1 + 2x
⇔ < 2x + 9
(−2x)2
√ 2
4x2 1 + 1 + 2x
⇔ < 2x + 9
4x2

⇔ 1 + 2 1 + 2x + 1 + 2x < 2x + 9
255


⇔ 2 1 + 2x < 7
⇔ 4(1 + 2x) < 49
⇔ 8x < 45
45
⇔ x< ,
8
45
de donde x ∈ h−∞, i.
8
Luego

45 45
C.S. = h−∞, i ∩ U = [−1/2, i − {0}.
8 8
2

114. (IMO 1964). Si a, b, c son los lados de un triángulo, pruebe que

a2 (b + c − a) + b2 (a + c − b) + c2 (a + b − c) ≤ 3abc.

 Prueba. La desigualdad es equivalente a:

a2 b + a2 c + b2 a + b2 c + c2 a + c2 b ≤ 3abc + a3 + b3 + c3
⇔ 0 ≤ a3 − a2 (b + c) + abc + b3 − b2 (a + c) + abc + c3 − c2 (a + b) + abc
⇔ 0 ≤ a(a − b)(a − c) + b(b − a)(b − c) + c(c − a)(c − b)
⇔ a(a − b)(a − c) + b(b − a)(b − c) + c(c − a)(c − b) ≥ 0,

la cual es verdadera, pues es la desigualdad de Schur. 2

115. (IMO 1975). Considere las sucesiones (x1 , x2 , . . . , xn , . . .) y (y1 , y2, . . . , yn , . . .)


tales que x1 ≤ x2 ≤ · · · ≤ xn , y1 ≤ y2 ≤ · · · ≤ yn , y (z1 , z2 , . . . , zn ) es una
permutación de (y1 , y2 , . . . , yn ), pruebe que

(x1 − y1 )2 + · · · + (xn − yn )2 ≤ (x1 − z1 )2 + · · · + (xn − zn )2 .


256 12. Soluciones

 Prueba. La desigualdad es equivalente a:

x21 + · · · + x2n + y12 + · · · + yn2 − 2(x1 y1 + · · · + xn yn )


≤ x21 + · · · + x2n + z12 + · · · + zn2 − 2(x1 z1 + · · · + xn zn )
⇔ x1 z1 + · · · + xn zn ≤ x1 y1 + · · · + xn yn (∗)

pues y12 + · · · + yn2 = z12 + · · · + zn2 .


La desigualdad (∗) es verdadera, pues ya fue demostrada en el capı́tulo de
Reordenamientos. 2

116. (IMO 1978). Sean x1 , x2 , . . . , xn números enteros diferentes, pruebe que

x1 x2 xn 1 1 1
2
+ 2 +···+ 2 ≥ + +···+ .
1 2 n 1 2 n

 Prueba. Sea (a1 , a2 , . . . , an ) unapermutación de (x1 , x2 , . . . , xn ) con a1 ≤


1 1 1
a2 ≤ · · · ≤ an y sea (b1 , b2 , . . . , bn ) = 2
, 2
, . . . , 2 , donde se observa
n (n − 1) 1
1
que bi = , i = 1, 2, . . . , n.
(n + 1 − i)2
Consideremos la permutación (c1 , c2 , . . . , cn ) de (a1 , a2 , . . . , an ) donde ci =
xn+1−i , i = 1, 2, . . . , n.
Luego

x1 x2 xn
+ + · · · + ≥ c1 b1 + c2 b2 + · · · + cn bn
12 22 n2
≥ an b1 + an−1 b2 + · · · + a1 bn
= a1 bn + a2 bn−1 + · · · + an b1
a1 a1 an
= 2 + 2 +···+ 2
1 2 n

como a1 ≥ 1, a2 ≥ 2, . . . , an ≥ n entonces

a1 a1 an 1 2 n
+ + · · · + ≥ + + · · · +
12 22 n2 12 22 n2
1 1 1
= + +···+
1 2 n
257

de donde se obtiene

x1 x2 xn 1 1 1
2
+ 2 +···+ 2 ≥ + +···+ .
1 2 n 1 2 n
2

117. (IMO 1997). Sean x1 , x2 , . . . , xn números reales que satisfacen

n+1
|x1 + x2 + · · · + xn | = 1 y |xi | ≤ , para todo i = 1, 2, . . . , n,
2

pruebe que existe la permutación (y1 , y2 , . . . , yn ) de (x1 , x2 , . . . , xn ) tal que

n+1
|y1 + 2y2 + · · · + nyn | ≤ .
2

 Prueba. Sin pérdida de generalidad tomamos x1 ≤ x2 ≤ · · · ≤ xn y como


1 < 2 < · · · < n entonces por el teorema de reordenamientos se tiene que

1x + 2x2 + · · · + nxn ≥ nx1 + (n − 1)x2 + · · · + 1xn


| 1 {z } | {z }
M N

⇒ M + N = (n + 1) (x1 + x2 + · · · + xn )
⇒ |M + N| = (n + 1) |x1 + x2 + · · · + xn | = (n + 1) · 1 = n + 1
⇒ |M + N| = n + 1

de donde M + N = n + 1 o M + N = −(n + 1).

n+1
i) Si M + N = n + 1 ⇒ N < <M y
2
(n + 1)
ii) Si M + N = −(n + 1) ⇒ N < − < M.
2
Gráficamente

n+1
 n+1
N − 2 M N 2 M
258 12. Soluciones

 
(n + 1) n + 1
Si tomaramos el primer caso, entonces N ∈ − , y en el segundo
  2 2
(n + 1) n + 1
caso M ∈ − , , es decir M o N se encuentran en el intervalo
 2 2
(n + 1) n + 1 (n + 1) n+1
− , y además N ≤ − < ≤ M.
2 2 2 2
Sean (y1 , y2 , . . . , yn ) una permutación de (x1 , x2 , . . . , xn ) tal que P = 1y1 +
2y2 + · · · + nyn .
(n + 1)
Tomamos el máximo valor de P ≤ − y también tomamos i tal que
2
y1 ≤ y2 ≤ · · · ≤ yn y yi > yi+1 y consideramos

Q = 1y1 + 2y2 + · · · + (i − 1)yi−1 + (i + 1)yi + iyi+1 + · · · + nyn

entonces

Q − P = iyi+1 + (i + 1)yi − (iyi + (i + 1)yi+1 ) = yi − yi+1 > 0

⇒ Q − P > 0 de donde Q > P


n+1 n+1
y desde que |yi | ≤ , |yi+1 | ≤ , tenemos que
2 2
 
n+1 n+1
yi − yi+1 ≤ +
2 2
⇔ yi − yi+1 ≤ n + 1
⇔ Q−P ≤n+1
(n + 1) n+1
⇔ Q≤P +n+1≤− +n+1=
2 2
n+1
⇔ Q≤ ,
2
   
n+1 n+1 n+1
de otro lado Q ≥ − , entonces − ≤Q≤ .
2 2 2
n+1
∴ |Q| ≤ .
2
2
259

118. (IMO 2004). Sea n un número entero positivo ≥ 3 y sean t1 , t2 , . . . , tn números


reales positivos tales que
 
2 1 1 1
n + 1 > (t1 + t2 + · · · + tn ) + +···+ ,
t1 t2 tn

pruebe que ti , tj , tk son los lados de un triángulo para todo i, j, k con 1 ≤ i <
j < k ≤ n.

 Prueba. Basta demostrar sin pérdida de generalidad que t1 < t2 + t3 por


la simetrı́a de la desigualdad.
Desarrollamos:
 
1 1 1
(t1 + t2 + · · · + tn ) + +···+
t1 t2 tn
X  ti tj  X  ti tj 
= n+ + =n+ +
i6=j
tj ti 1≤i<j≤n
tj ti

X  
t1 t2 t1 t3 ti tj
= n+ + + + + +
t2 t1 t3 t1 tj ti
1≤i<j≤n
(i, j) 6= (1, 2)
(i, j) 6= (1, 3)
  X  
1 1 t2 + t3 ti tj
= n + t1 + + + +
t2 t3 t1 tj ti
1≤i<j≤n
(i, j) 6= (1, 2)
(i, j) 6= (1, 3)
√  
2t1 2 t2 t3 n(n − 1)
= n+ √ + +2 −2
t2 t3 t1 2

t1
Haciendo √ = a, tenemos
t2 t3
 
1 1 1 2
(t1 + t2 + · · · + tn ) + +···+ ≥ n + 2a + + n2 − n − 4
t1 t2 tn a
260 12. Soluciones

como
 
2 1 1 1
n + 1 > (t1 + t2 + · · · + tn ) + +···+
t1 t2 tn

por transitividad

2
n2 + 1 > n + 2a + + n2 − n − 4
a
2
⇔ 2a + − 5 < 0; a > 0
a
⇔ 2a2 − 5a + 2 < 0
⇔ (2a − 1)(a − 2) < 0
1
⇔ <a<2
2
1 t1
⇔ <√ <2
2 t2 t3

⇒ t1 < 2 t2 t3

Como 2 t2 t3 ≤ t2 + t3 ,

∴ t1 < t2 + t3 .

119. (IMO 2000). Sean a, b, c números reales positivos tales que abc = 1, pruebe
que
   
1 1 1
a−1+ b−1+ c−1+ ≤ 1.
b c a

 Prueba. La desigualdad es equivalente a:

(ab − b + 1)(bc − c + 1)(ca − a + 1) ≤ abc


⇔ (ab − b + 1)(bc − c + 1)(ca − a + 1) ≤ 1 (12.21)
261

Como abc = 1, podemos hacer el cambio:

x y z
a= , b = , c = , x, y, z > 0
y z x

en efecto la desigualdad (12.21) se transforma en:

x y  y z  z x 
− +1 − +1 − +1 ≤1
z z x x y y
⇔ (x − y + z)(y − z + x)(z − x + y) ≤ xyz (12.22)

Haciendo u = x − y + z, v = y − z + x, w = z − x + y

⇒ u + v = 2x, u + w = 2z, v + w = 2y

en efecto la desigualdad (12.22) es equivalente a:

   
u+v v+w u+w
u·v·w ≤
2 2 2
⇔ 8u · v · w ≤ (u + v)(v + w)(u + w) (12.23)

Si x, y, z son lados de un triángulo, entonces u, v, w son positivos, entonces la


desigualdad 8u · v · w ≤ (u + v)(v + w)(u + w) se verifica.
En seguida veamos cuando

i) Uno de ellos es negativo.


Supongamos que u < 0
entonces

8u · v · w ≤ (u + v)(v + w)(u + w)

se verifica pues

8u · v · w < 0 ∧ (u + v)(v + w)(u + w) > 0

es decir el miembro de la izquierda es negativo y el de la derecha es


positivo, verificando ası́ la desigualdad (12.23).
262 12. Soluciones

ii) Dos de ellos son negativos.


Supongamos que u < 0 y v < 0
en efecto sabemos que u + v = 2x < 0, esto contradice pues x > 0.
iii) Los tres son negativos.
Si u, v, w < 0 entonces

u + v = 2x < 0
u + w = 2z < 0
v + w = 2y < 0

esto nuevamente se contradice, no es posible este caso.

120. Pruebe que

n
X nX
|i − j| |xi − xj | = |xi − xj |
i,j=1
2 i,j

si x1 ≤ x2 ≤ · · · ≤ xn .

 Prueba.

n
X X
|i − j| |xi − xj | = |(i − j)(xi − xj )|
i,j=1 i,j

n

Xn X X n Xn

= (i − j)(x i − x )
j = (i − j)(xi − xj );
| {z }
i=1 j=1 i=1 j=1
(+)

(pues x1 ≤ x2 ≤ · · · ≤ xn )
n X
X n
= (ixi − ixj − jxi + jxj )
i=1 j=1
n X
X n n X
X n n X
X n n X
X n
= ixi − ixj − jxi + jxj
i=1 j=1 i=1 j=1 i=1 j=1 i=1 j=1
263

n
X n
X n
X n
X n
X n
X
= nixi − i xj − xi j+ njxj
i=1 i=1 j=1 i=1 j=1 j=1
n
X n
X n
X n
n(n + 1) n(n + 1) X
= nixi − xj − xi · + nixi
i=1
2 j=1 i=1
2 i=1
| {z }
n
X n
X
= 2nixi − n(n + 1) xi
i=1 i=1
Xn
= n (2i − (n + 1)) xi
i=1

efectuando la sumatoria

n
nX X
|xi − xj | = n |xi − xj |
2 i,j=1 i<j
X
=n (xi − xj )
1≤i<j≤n

= n [(xn − x1 ) + (xn − x2 ) + · · · + (xn − xn−1 )]


+n [(xn−1 − x1 ) + (xn−1 − x2 ) + · · · + (xn−1 − xn−2 )]
+···
+n [(x3 − x1 ) + (x3 − x2 )]
+n [(x2 − x1 )]
= n [(n − 1)xn + (n − 3)xn−1 + (n − 5)xn−2 + · · · + (1 − n)x1 ]
n
X
=n (2i − (n + 1)) xi ,
i=1

n
X nX
∴ |i − j| |xi − xj | = |xi − xj |.
i,j=1
2 i,j

2
264 12. Soluciones

121. Pruebe que

n
X n2 (n2 − 1)
(i − j)2 = .
i,j=1
6

 Prueba.

n
X n
X 
(i − j)2 = i2 − 2ij + j 2
i,j=1 i,j=1

n
X n
X n
X
= i2 − 2 ij + j2
i,j=1 i,j=1 i,j=1

X n
n X n X
X n n X
X n
= i2 − 2 ij + j2
i=1 j=1 i=1 j=1 i=1 j=1

n
X n
X Xn
2 n(n + 1)
= n·i −2 i· +n j2
i=1 i=1
2 j=1

n(n + 1)(2n + 1) n(n + 1) n(n + 1)


= n· −2· ·
6 2 2
n(n + 1)(2n + 1)
+n ·
6
n2 (n + 1)(2n + 1) n2 (n + 1)2
= −
3 2
   
2 2n + 1 n + 1 2 4n + 2 − 3n − 3
= n (n + 1) − = n (n + 1)
3 2 6
n2 (n + 1)(n − 1)
=
6
n2 (n2 − 1)
= .
6

2
265

122. (IMO 2003). Sea n un número entero ≥ 2 y los números reales x1 , x2 , . . . , xn


con x1 ≤ x2 ≤ · · · ≤ xn , pruebe que

n
!2 n
X 2 2 X
|xi − xj | ≤ n −1 (xi − xj )2 .
i,j=1
3 i,j=1

 Prueba. Para probar esta desigualdad utilizaremos resultados anteriores


como:
X nX
a) |i − j| |xi − xj | = |xi − xj |,
i,j
2 i,j
X n2 (n2 − 1)
b) (i − j)2 = ,
i,j
6

aplicamos la desigualdad de Cauchy-Schwarz

!2
X X X
|i − j| |xi − xj | ≤ (i − j)2 · (xi − xj )2
i,j i,j i,j
| {z }
!2
nX n2 (n2 − 1) X
⇔ |xi − xj | ≤ · (xi − xj )2
2 i,j 6 i,j

n2 P 2 n2 (n2 − 1) X
⇔ i,j |xi − xj | ≤ · (xi − xj )2
4 6 i,j
P 2 2 2 X
⇔ i,j |xi − xj | ≤ (n − 1) (xi − xj )2 .
3 i,j

123. (IMO 2006). Determine el menor valor de M para que la desigualdad


ab(a2 − b2 ) + bc(b2 − c2 ) + ca(c2 − a2 ) ≤ M a2 + b2 + c2 2

se verifique para cualesquiera a, b, c números reales.


266 12. Soluciones

 Prueba. Factorizamos el primer miembro de la desigualdad


ab(a2 − b2 ) + bc(b2 − c2 ) + ca(c2 − a2 ) = |(a − b)(b − c)(c − a)(a + b + c)|

Haciendo

a − b = x, b − c = y, c − a = z ⇒ x + y + z = 0

y además haciendo α = a + b + c

⇒ α2 = a2 + b2 + c2 + 2ab + 2ac + 2bc,


x2 = a2 + b2 − 2ab,
y 2 = b2 + c2 − 2bc,
z 2 = c2 + a2 − 2ca,

sumando tenemos:

α2 + x2 + y 2 + z 2 = 3 (a2 + b2 + c2 )
2
2 2 2 2 (α2 + x2 + y 2 + z 2 )
⇒ (a + b + c ) = .
9

Reemplazando tenemos una desigualdad equivalente


!
2
(α2 + x2 + y 2 + z 2 )
|αxyz| ≤ M
9
2
⇔ 9|αxyz| ≤ M (α2 + x2 + y 2 + z 2 ) ,

como

x + y + z = 0 ⇒ −z = x + y
⇒ |z| = |x + y|,
267

entonces

9|αxyz| = 9|αxy(x + y)|,

luego tenemos la desigualdad


2
9|αxy(x + y)| ≤ M α2 + x2 + y 2 + z 2 ,

(x + y)2
como xy ≤
4
9
⇒ 9|αxy(x + y)| ≤ |α||x + y|3.
4

9 2
Luego debemos encontrar la relación |α||(x + y)3| ≤ M (α2 + x2 + y 2 + z 2 ) .
4
Aplicando MA ≥ MG:

2α2 + (x + y)2 + (x + y)2 + (x + y)2 p


≥ 4 2α2(x + y)6
4
 2 4
2α + 3(x + y)2
⇔ ≥ 2α2 · (x + y)6
4
2
(2α2 + 3(x + y)2) √
⇔ ≥ 2|α||(x + y)3 |
42
 2 2
2α + 3(x + y)2 √
⇔ ≥ 4 2|α||(x + y)3 |
2
 2
3 √
⇔ α2 + (x + y)2 ≥ 4 2|α||(x + y)3 |
2
 2
9 3 9 2 3 2
⇔ |α||(x + y) | ≤ √ α + (x + y)
4 16 2 2
√  2
9 2 3
= α2 + (x + y)2 .
32 2
268 12. Soluciones

Pero

(x + y)2 ≤ 2(x2 + y 2)
⇔ 3(x + y)2 ≤ 2(x2 + y 2) + 2 (x + y)2
| {z }
⇔ 3(x + y)2 ≤ 2(x2 + y 2) + 2z 2
3
⇔ (x + y)2 ≤ (x2 + y 2 + z 2 )
2
3
⇔ α2 + (x + y)2 ≤ α2 + x2 + y 2 + z 2
2
 2
2 3 2 2
⇔ α + (x + y) ≤ (α2 + x2 + y 2 + z 2 )
2
√  2 √
9 2 2 3 2 9 2 2 2
⇔ α + (x + y) ≤ (α + x2 + y 2 + z 2 )
32 2 32

9 9 2 2 2
⇒ |α||(x + y)3 | ≤ (α + x2 + y 2 + z 2 ) ,
4 32

9 2
entonces M ≥
32

9 2
de donde el menor valor de M es y esto ocurre si y sólo si (a, b, c) =
  32
3 3
1 + √ , 1, 1 − √ . 2
2 2

124. (IMO 1983). Sean a, b, c los lados de un triángulo, pruebe que

a2 b(a − b) + b2 c(b − c) + c2 a(c − a) ≥ 0.

 Prueba. Haciendo


a + b − c = 2y


a = x + y, b = y + z, c = z + x ⇒ a + c − b = 2x



b + c − a = 2z
269

como a, b, c son lados de un triángulo entonces x, y, z > 0.


Reemplazando en la desigualdad, tenemos otra equivalente:

(x + y)2(y + z)(x − z) + (y + z)2 (z + x)(y − x) + (z + x)2 (x + y)(z − y) ≥ 0

efectuando tenemos:

zx3 + xy 3 + yz 3 ≥ xyz 2 + yzx2 + zxy 2


⇔ zx3 + xy 3 + yz 3 ≥ xyz(z + x + y)
x2 y 2 z 2
⇔ + + ≥ x + y + z.
y z x

Para probar este desigualdad utilizaremos la desigualdad de Titu

x2 y 2 z 2 (x + y + z)2
+ + ≥ =x+y+z
y z x y+z+x

de donde

x2 y 2 z 2
+ + ≥ x + y + z.
y z x
2

125. Sean a, b, c, d, e números reales positivos tales que abcde = 1, pruebe que

a + abc b + bcd c + cde d + dea


+ + +
1 + ab + abcd 1 + bc + bcde 1 + cd + cdea 1 + de + deab
e + eab 10
+ ≥ .
1 + ea + eabc 3

 Prueba. Como abcde = 1, entonces podemos hacer el cambio:

x y z w t
a= , b= , c= , d= , e=
y z w t x
270 12. Soluciones

en efecto

x x 1 1
+ +
a + abc y w y w
= x x =
1 + ab + abcd 1+ + 1 1 1
z t + +
x z t

también es conveniente otro cambio de variable, es decir

1 1 1 1 1
= m, = n, = p, = q, =r
x y z w t

luego

a + abc n+q
= ; similarmente
1 + ab + abcd m+p+r
b + bcd p+r
= ;
1 + bc + bcde n+q+t
c + cde q+m
= ;
1 + cd + cdea p+n+r
d + dea r+n
= ;
1 + de + deab m+p+q
e + eab m+p
= ;
1 + ea + eabc n+q+r

entonces basta demostrar la desigualdad

n+q p+r q+m r+n m+p 10


+ + + + ≥
m+p+r n+q+t p+n+r m+p+q n+q+r 3
     
n+q p+r q+m
⇔ +1 + +1 + +1
m+p+r n+q+t p+n+r
   
r+n m+p 10
+ +1 + +1 ≥ +5
m+p+q n+q+r 3
271

m+n+p+q+r m+n+p+q+r m+n+p+q+r


⇔ + +
m+p+r n+q+t p+n+r
m+n+p+q+r m+n+p+q+r 25
+ + ≥
m+p+q n+q+r 3
⇔ 3(m + n + p + q + r)
 
1 1 1 1 1
+ + + + ≥ 25
m+p+r n+q+t p+n+r m+p+q n+q+r

Haciendo

m + p + r = a′ , n + q + t = b′ , p + n + r = c′ ,
m + p + q = d′ , n + q + r = e′ ,

la desigualdad es equivalente a:
 
′ ′ ′ ′ 1 ′1 1 1 1
(a + b + c + d + e ) + + + + ≥ 25
a′ b′ c′ d′ e′
a′ + b′ + c′ + d′ + e′ 5
⇔ ≥ ,
5 1 1 1 1 1
+ + + +
a′ b′ c′ d′ e′

esta desigualdad es verdadera, pues MA ≥ MH. 2

126. Sean a, b, c que satisfacen a2 + b2 + c2 = 1, halle el máximo valor de

P = (a − b)(a − c)(b − c)(a + b + c).

 Prueba. Como

" #2
2 2
3(a b2 + c}2 )
| + {z = [2(a − b)2 + 2(a − c)(b − c) + (a + b + c)2 ]
1

≥ 8(a − c)(b − c) (2(a − b)2 + (a + b + c)2 )



≥ 16 2(a − c)(b − c)(a − b)(a + b + c)
272 12. Soluciones


⇔ 32 ≥ 16 2 (a − c)(b − c)(a − b)(a + b + c)
| {z }
P
9
⇔ P ≤ √ .
16 2

9 2
∴ el máximo de P es . 2
32

127. (IMO 2008).

(a) Demuestre que

x2 x2 x2
+ + ≥1 (12.24)
(x − 1)2 (x − 1)2 (x − 1)2

para todos los números reales x, y, z, distintos de 1, con xyz = 1.


(b) Demuestre que existen infinitas ternas de números racionales x, y, z, dis-
tintos de 1, con xyz = 1 para los cuales la expresión (12.24) es una
igualdad.

 Prueba.

(a) Haciendo x − 1 = a, y − 1 = b, z − 1 = c, tenemos que x = a + 1, y =


b + 1, z = c + 1 y como xyz = 1 entonces (a + 1)(b + 1)(c + 1) = 1. De
donde abc + ab + ac + bc + a + b + c = 0.
Luego la desigualdad es equivalente a

(a + 1)2 (b + 1)2 (c + 1)2


+ + ≥1
a2 b2 c2

con abc + ab + ac + bc + a + b + c = 0.
Efectuando tenemos

b2 c2 (a + 1)2 + a2 c2 (b + 1)2 + a2 b2 (c + 1)2 ≥ a2 b2 c2


⇔ 2a2 b2 c2 + 2ab2 c2 + 2ba2 c2 + 2ca2 b2 + a2 b2 + a2 c2 + b2 c2 ≥ 0
⇔ 2a2 b2 c2 + 2abc(bc + ac + ab) + a2 b2 + a2 c2 + b2 c2 ≥ 0. (12.25)
273

Además, como

abc + a + b + c = −(ab + ac + bc) (12.26)

elevando al cuadrado

a2 b2 c2 + (a + b + c)2 + 2abc(a + b + c)
= a2 b2 + a2 c2 + b2 c2 + 2abc(a + b + c)
⇔ a2 b2 c2 + (a + b + c)2 = a2 b2 + a2 c2 + b2 c2 ,

reemplazando en (12.25)

2a2 b2 c2 + 2abc(−abc − (a + b + c)) + a2 b2 c2 + (a + b + c)2 ≥ 0


⇔ a2 b2 c2 − 2abc(a + b + c) + (a + b + c)2 ≥ 0
⇔ (a + b + c − abc)2 ≥ 0.

(b) La igualdad ocurre si y sólo si

a + b + c − abc = 0
⇔ a + b + c = abc,

reemplazando en (12.26) tenemos que

2(a + b + c) + ab + ac + bc = 0
⇔ 2(x + y + z − 3) + (x − 1)(y − 1) + (x − 1)(z − 1)
+(y − 1)(z − 1) = 0
⇔ 2(x + y + z) − 6 + xy − (x + y) + 1 + xz − (x + z) + 1
+yz − (y + z) + 1 = 0
⇔ xy + xz + yz = 3
1 1
⇔ xy + + = 3, pues xyz = 1
y x
 
2 1
⇔ yx + − 3 x + 1 = 0,
y
274 12. Soluciones

como x es un número racional, entonces es necesario que su discriminante


sea cuadrado perfecto, en efecto
 2
1 9y 2 − 6y + 1
∆ = −3 − 4y = − 4y
y y2
−4y 3 + 9y 2 − 6y + 1 (y − 1)2 (1 − 4y)
= = ,
y2 y2

luego para que sea cuadrado perfecto basta que 1 − 4y = k 2 , con k ∈ Q,


1 − k2 2(k − 1)
luego y = , reemplazando en la ecuación se obtiene x = ∨
4 (1 + k)2
−2(1 + k)
x= , y de xyz = 1, se encuentra el valor de z.
(1 − k)2
Por lo tanto existen infinitas ternas como
 
2(k − 1) 1 − k 2 −2(1 + k)
(x, y, z) = , , ∨
(1 + k)2 4 (1 − k)2
 
−2(1 + k) 1 − k 2 2(k − 1)
(x, y, z) = , , .
(1 − k)2 4 (1 + k)2

2
BIBLIOGRAFÍA

[1] Elon Lages Lima: Análisis Real vol. 1.


IMPA.

[2] Mircea Becheanu; Bogdan Enescu: Balkan Mathematical


Olympiads 1984-2006.
GIL Publishing Home.

[3] Plam van Thuân; Lé Vi: Batdang Thuc.


Suy Ivân & Khán Phá

[4] Titu Andreescu; Dorin Andrica: 360 Problems for Mathematical


Contests.
GIL Publishing Home.

[5] Andrei Negut: Problems for the Mathematical Olympiads.


GIL Publishing Home.

[6] Radmila Bulajich; José Gómez; Rogelio Valdez: Cuaderno de


Olimpiadas de Matemática.
Instituto de Matemáticas UNAM.

[7] Titu Andreescu; Vasile Cirtoaje; Gabriel Dospinescu; Mircea Las-


cu: Old and New Inequalities.
GIL Publishing Home.

[8] Vasile Cirtoaje: Algebric Inequalities Old and New Methods.


GIL Publishing Home.

275
276 BIBLIOGRAFÍA

[9] Pham Kim Hung: Secrets in Inequalities, Volumen 1 - Basic Inequal-


ities.
GIL Publishing Home.

[10] Tran Phuing: Diamonds in Mathematical Inequalities.


Hanoi Publishing House.

[11] Titu Andreescu: I Seminario de Educación Matemática Iberoamer-


icana con Énfasis en Resolución de Problemas (SEMI 2005).
Cartagena de Indias, Colombia, Setiembre 2005.

También podría gustarte